Sie sind auf Seite 1von 88

31.

Surgery- Orthopedic Surgery and


Neurosurgery

1
2
Regarding acute osteomyelitis in children, which of the following statements are true:

1. Usually affects the diaphysis of a long bone


2. Can be confirmed in the early stages by x-ray examination
3. Can be cured by oral antibiotics for 7 days as an out patient
4. It is most often a staphylococcal infection

1,2,3 only
1 only
2,4 only
4 only
None of the above

Explanation
The correct answer is choice D

Osteomyelitis refers to an infection of the bone or the bone marrow. It affects mainly the metaphysis of
long bones commonly the distal femur and proximal tibia. The mode of infection is usually through the
haematogenous route, although there can be direct spread from a superficial wound infection.

The metaphysis NOT diaphysis (Choice B) of long bones is most affected probably due to the sluggish
circulation in the metaphyseal capillary loop.

The commonest infective agent in all age groups is Staphylococcus aureus, although other Gram-positive
cocci such as Streptococcus pneumonia or Streptococcus pyogens have been implicated. E. coli and
Bacteroides Spp. is a common cause in newborns. Haemophilus influenza is found in children less than 4 years
of age. Salmonella Spp. Is implicated in sickle cell anemia.

The cardinal symptoms and signs of osteomyelitis are pain, fever, inflammation and acute tenderness.
Newborn and young infants usually present with failure to thrive, irritability and asymmetric movement of
the limbs. Young children present with inability to support weight, fever, bone pain, swelling, redness and
guarding of the affected joint.

Definitive diagnosis is usually made by x-ray of the affected bone. In the first two weeks, the only changes seen on x-ray
is usually soft tissue swelling. Periosteal elevation and new bone formation becomes visible by the end of the second
week. Neighbouring bones may display some sclerotic changes. Ultrasound scan provides a means of visualizing sub-
periosteal pus collection and assisting ultrasound guided needle aspiration.

Other supportive laboratory investigations include; Full Blood Count showing leucocytosis, raised ESR and positive blood
cultures in about 50% of cases.

Differential diagnoses include septic arthritis, cellulites, rheumatic fever, Gauchers disease and sickle-cell disease.

The key aims of treatment are the selection of the appropriate antibiotic, prescription of an adequate dose and treatment
for a sufficient length of time to achieve cure. Supportive treatment includes provision of analgesia and fluid balance.

Antibiotic therapy may be required for up to 3 months in osteomyelitis. Anti-Staphylococcal agents include flucloxacillin
and fusidic acid. It is important to start antibiotics only after blood culture samples have been taken. If there is evidence of
pus collection, then the abscess should be drained. Poorly treated acute osteomyelitis may become chronic.

3
A 33 year old woman presents at your clinic complaining of attacks of "terrible dizziness" that she
experienced while taking her two young children on a merry-go-round several days ago, and over the
days that followed. She denies experiencing anything of the sort when she was younger. She is a non-
smoker and denies consuming any alcohol prior to any of her episodes of "dizziness", but she report a
history of allergic rhinitis, which she treats with antihistamines. Before you proceed, you decide to review
the various sensations that patients call "dizziness" and the conditions that can lead to them.

Which of the following statements is accurate regarding "dizziness"?

hypotension causes decreased volume of endolymph fluid, leading to vertigo


central vertigo includes vertigo resulting from neuritis of cranial nerve VIII
peripheral vertigo tends to be more severe than central vertigo
peripheral vertigo generally manifests with vertical nystagmus
disequilibrium typically presents with horizontal nystagmus

Explanation

The correct answer is choice C.

The word "dizziness" could mean disequilibrium, presyncope, or vertigo, and the etiology can involve any
of the systems listed. Disequilibrium is the sensation that one is off-balance. It is not connected with
nausea and can result of falls, often in a specific direction. Presyncope is lightheadedness or the feeling
that one is about to faint, while vertigo refers to a sensation of spinning or other movement and often is
associated with nausea and vomiting and nystagmus. Thus, it is important to question the patient carefully
as to the exact sensations she's experienced and how they related to her time on the merry-go-round.

Disruptions of the central nervous system can lead to disequilibrium and a feeling of discoordination and an
unsteady gait. Such disruptions are associated with the cerebellar system, not the vestibular apparatus,
and thus do not present with nystagmus (choice E).

Cardiovascular disturbances, hypotension for instance, can cause presyncope, but it does not lead directly
to vertigo (choice A).

Vertigo often results from conflicting inputs from vestibular, proprioceptive, and visual systems. Thus,
disruptions of the vestibular branch of cranial nerve VIII can lead to vertigo, as can disturbances in the
vestibular apparatus itself. If the problem is involves the brain, the term central vertigo is used, while
peripheral vertigo refers to a disturbance involving the vestibular apparatus (labyrinths) of the inner ear, or
cranial nerve VIII (choice B). Upper respiratory infection (URI) can lead to labyrinthitis, inflammation of the
eustachian tube, or middle ear inflammation (otitis media), all of which can cause peripheral vertigo.

Compared to central vertigo, peripheral vertigo manifests with symptoms that are of shorter duration, but
with an abrupt onset and more severity (choice C). Associated nystagmus tends to be only horizontal, not
vertical (choice D).

Physical examination of such a patient should include basic tests such as otoscopy to reveal bulging of the
tympanic membrane, blood pressure testing lying down vs. standing to reveal orthostatic intolerance, a full
cranial nerve exam, and a Romberg test. A Romberg test involves balance and coordination with eyes
closed. Positive Romberg means that patient cannot maintain balance with eyes closed and further testing
may include electronystagmography (ENG) to explore any vestibular deficits. However, if a positive
Romberg test is the only abnormality on physical examination related to balance, it indicates good
cerebellar function.

4
A 13-year-old girl appears at your office at 5:05 PM for a 3:30 PM appointment scheduled for the day
before. Her mother tells you that the girl has been limping for a couple of weeks and has much knee pain.
She has been afebrile, does not recall being hit in the knee or leg, and has not had any illnesses recently.
She has difficulty "moving her leg inward."

Given the late hour and that the workup will be done in the emergency department, you impress the
pediatric emergency department staff by telling them that the most likely diagnosis is which one of the
following:

She twisted the leg trying to be on time for the appointment yesterday
Septic arthritis of the hip
Septic arthritis of the knee
Aseptic necrosis of the hip
Slipped capital femoral epiphysis

Explanation

The correct answer is Choice E

Slipped capital femoral epiphysis is also known as slipped upper femoral epiphysis (SUFE), a condition that
occurs when the epiphysis of the femoral head is displaced postero-inferiorly.This is mostly due to
weakness in the growth plate. It is most common in adolescents aged 10 -15 years, affecting male children
more than female. The cause of SUFE is unknown, however, it is associated with obesity,
endocrinopathies(hypothyroidism, acromegaly, hypogonadism, hypopituitarism), renal failure and
following radiation therapy in children with leukemia or lymphoma.

Diagnosis is usually made from a careful history and physical examination.Children present with a limp or
hip pain which may be referred to the knee.There is restriction of abduction and internal rotation of the
affected hip. SUFE is a strong differential of knee pain in children. Clinical suspicion can be confirmed using
plain radiographs, anteroposterior and lateral views. The mainstay of treatment is surgery.

Aseptic necrosis of the hip (Choice D) is a long-term complication of untreated or poorly treated slipped
capital femoral epiphysis, due to impaired blood supply to the head of femur.

Septic arthritis(SA) is a suppurative inflammation occurring within a joint space. The disease is common in
infancy and childhood. it is usually monoarticular, commonly affecting the hip(Choice B) or the knee
(Choice C), although it can affect any joint. SA is a clinical emergency as it can destroy a joint in under 24
hours. consider SA in any acutely inflamed joint. The most common organism isolated from culture of joint
aspirate is Staphylococcus aureus. Other common organisms include; Streptococcus species, Pseudomonas
aeruginosa, pneumococci, Neisseria meningitidis, Escherichia coli, Klebsiella and Enterobacter species.
Treatment entails the use of empirical broad spectrum IV antibiotics until culture and sensitivity results
are known.

The patient has limped for a couple of weeks ruling out Choice A.

A 34 year old labourer presents to you wish severe back and right leg pain after lifting some heavy pieces
of wood. The pain radiates form his low back down the posterior lateral aspect of the leg into the lateral
foot. The pain is associated with a sensation of swelling in the lateral aspect of his right foot. Straight leg

5
raising is sensation of swelling in the lateral aspect of his right foot. Straight leg rasing is limited to 40
degrees. Laseque sign is positive but the bowstring sign is negative.

What additional signs would you expect to find in this patient?

hyper-reflexia with clonus on both legs but more so on the right


a suspected disassociated sensory loss from T12 to S1
wasting of the right quadriceps muscle with an absent right knee reflex
an absent right ankle reflex
weakness of ankle dorsiflesion and reduced sensation over the right hallaux

Explanation
The correct answer is choice D

A spinal disc herniation is a medical condition affecting the spine, in which a tear in the outer, fibrous ring
(annulus fibrosus) of an intervertebral disc (discus intervertebralis) allows the soft, central portion (nucleus
pulposus) to bulge out. Tears are almost always posterior-ipsilateral in nature owing to the presence of the
posterior longitudinal ligament in the spinal canal. This tear in the disc ring may result in the release of
inflammatory chemical mediators which may directly cause severe pain, even in the absence of nerve root
compression. This is the rationale for the use of anti-inflammatory treatments for pain associated with disc
herniation, protrusion, bulge, or disc tear.

Symptoms of a herniated disc can vary depending on the location of the herniation and the types of soft
tissue that become involved. They can range from little or no pain if the disc is the only tissue injured, to
severe and unrelenting neck or low back pain that will radiate into the regions served by affected nerve
roots that are irritated or impinged by the herniated material. Often, herniated discs are not diagnosed
immediately, as the patients come with undefined pains in the thighs, knees or feet. Other symptoms may
include sensory changes such as numbness, tingling, muscular weakness, paralysis, paresthesia, and
affection of reflexes. If the herniated disc is in the lumbar region the patient may also experience sciatica
due to irritation of one of the nerve roots of the sciatic nerve. Unlike a pulsating pain or pain that comes
and goes, which can be caused by muscle spasm, pain from a herniated disc is usually continuous or at
least is continuous in a specific position of the body.75,000.

The Straight leg raise, also called Lasègue's sign is a test done during the physical examination to
determine whether a patient with low back pain has an underlying herniated disk, mostly located at L5
(fifth lumbar spinal nerve), S1 (the first sacral spinal nerve) or S2 (the second sacral spinal nerve). With
the patient lying down on his/her back on an examination table/or exam floor, the examiner lifts the
patient's leg while the knee is straight. The straight leg raise test is positive if pain in the sciatic
distribution is reproduced between 30 and 70 degrees passive flexion of the straight leg."

The ankle jerk reflex, also known as the Achilles reflex, occurs when the Achilles tendon is tapped while
the foot is dorsi-flexed. A positive result would be the jerking of the foot towards its plantar surface. The
Achilles reflex checks if the S1 and S2 nerve roots are intact and could be indicative of sciatic nerve
pathology. This reflex is usually absent in disk herniations at the L5-S1 level.

hyper-reflexia with clonus on both legs but more so on the right (choice A) is consistent with an upper
motor neuron lesion. An upper motor neuron lesion is a lesion of the neural pathway above the anterior
horn cell or motor nuclei of the cranial nerves.

a suspected disassociated sensory loss from T12 to S1 (choice B) is not consistent with a herniated disc.
Dissociated sensory loss is a pattern of neurological damage caused by a lesion to a single tract in the
spinal cord which involves selective loss of fine touch and proprioception without loss of pain and
temperature, or vice-versa. Dissociated sensory loss always suggests a focal lesion within the spinal cord

6
or brainstem. A disassociated sensory loss from T12 to S1 would be consistent with Tabes dorsalis or
Multiple sclerosis.

wasting of the right quadriceps muscle with an absent right knee reflex (choice C) is consistent with
damage to the L4 nerve root. It is inconsistent with a positive straight leg raise test and sciatic symptoms.

weakness of ankle dorsiflexion and reduced sensation over the right hallaux (choice E) is consistent with
damage to the L5 nerve root. It is inconsistent with a positive straight leg raise test and sciatic symptoms.

A 25 year old man falls while skateboarding and strikes the left side of his head against a concrete
retaining wall. On physical examination only a minor scalp abrasion is present at the site of the impact,
with minimal bleeding that stops after a few minutes. He is initially alert following this accident, but then
becomes unconscious 30 minutes later. A head CT scan reveals a convex, lens-shaped area of hemorrhage
centered over the left parietal region.

These events are most likely to be associated with damage to which of the following parts of the
intracranial vasculature?

Bridging veins
Cavernous sinus
Great vein of Galen
Posterior inferior cerebellar artery
Middle meningeal artery

Explanation

The correct answer is Choice E.

Epidural hematomas develop between the dura mater and the skull bone and is often the result of head injury. The
bleeding is rapid due to a tear in the high pressure, middle meningeal artery. As the hematoma expands, the dura is
stripped from the inside of the skull, trapping the blood in the newly made space. It is seen as a biconvex, or
lentiform (lens-like), lesion on a non-contrast head CT—the hematoma is confined by the tightly attached dura
membrane and is unable to follow the brain’s curvature as a result.

A subdural hematoma occurs when the small, bridging veins that run on the surface of the brain tear and cause
bleeding (choice A). The hematoma collects between the dura mater (which adheres to the skull) and the arachnoid
mater (which envelopes the brain). In essence, the hematoma is located between two membranes and is seen as a
crescent-shaped lesion on a non-contrast head CT—the blood is free to follow alongside the conclave curvature of
the brain’s surface. This compression can cause an increase in intracranial pressure which may produce papilodema.

Symptoms for subdural and epidural hematomas are similar, except that epidural signs occur rapidly (within hours)
compared to subdural (within days to weeks). Another difference is that a lucid period of consciousness occurs
frequently in epidural hematomas whereas a slow change in mental status happens in subdural hematomas.

Symptoms of epidural hemorrhage include:

 Confusion, disorientation
 Abnormal posturing
 Loss of consciousness (often preceded with lucid period)
 Headache (severe)

7
 Weakness and/or numbness of arms, legs, face
 Vertigo
 Nausea or vomiting
 Anisocoria

The most common injury of the cavernous sinus (choice B) is thrombosis which usually occurs after an external
facial injury that lead to a clot in the facial vein. Common symptoms are proptosis, droopy eyelid, the inability to
move the eye in a particular direction or vision loss. This type of thrombosis is rarely fatal and can be treated with
antibiotics. The illness can become more serious, however, if it is left untreated and spreads to the dural veinous
sinuses.

The most common injury to the great vein of Galen (choice C) is thrombosis and a form of stroke that presents with
consciousness problems, headaches, nausea, visual defects, fatigue, disturbance of eye movements and pupillary
reflexes. Risk factors include oral contraceptives, pregnancy and postpartum.

Injury to the posterior inferior cerebellar artery (choice D) leads to an infarction of the medulla oblongata that
is called the Wallenberg syndrome. The patient presents with dysarthria, dysphonia, vertigo, nystagmus, ataxia,
sensory loss and ipsilateral Horner's Syndrome (ptosis, miosis, anhidrosis). The sensory loss consists of a loss of
pain and temperature sensation on the contralateral side of the body and ipsilateral side of the face. This crossed
sensory deficit is diagnostic for the syndrome.

A 15 year-old African-American boy is brought to your clinic, complaining of pain in the medial aspect of
his left knee. He denies trauma, and both he and his parents confirm that he has not suffered from a
fever. On physical examination, you note that the boy is obese, with a BMI of 32. While the range of
motion in both knees is normal, when asked to relax, he holds the left hip in external rotation and you find
that internal rotation of this joint is limited.

8
Which of the following is the most likely diagnosis?

Toxic synovitis
Legg-Calvé-Perthes
Medial collateral ligament (knee) sprain
Chronic Slipped capital femoral epiphysis
Osgood-Schlatter disease

Explanation

The correct answer is choice D.

Slipped capital femoral epiphysis (SCFE) is one of several conditions that can affect the hip joint during
childhood, and as with all hip condition, often pain is referred to the knee. While SCFE is fairly rare,
incidence is much higher in males compared to females and is influenced by ethnicity, with African-
Americans having 3.94 times the incidence of Caucasian children. The condition also depends on age,
presenting most frequently during adolescence. Obesity is an important risk factor, and in the case of
children younger than 10, endocrine disorders, particularly hypothyroidism, are associated as well.
Additionally, unexplained geographical and climate associations have been noted, with presentations
occurring more often during summer at latitudes higher than 40 degrees and during winter at lower
latitudes, in North America.

Toxic synovitis (choice A), also known as transient synovitis, is the most common cause of hip pain in
children. It is caused by a viral infection that sometimes (but not always) settles in the hip joint. The virus
leads to inflammation at the hip joint, which makes walking painful. Usually, just one hip is affected,
although the infection may can spread to the other hip.

Presenting most commonly in children between the ages of 4-8, Legg-Calvé-Perthes disease (choice B) is a
degenerative condition of the hip joint, resulting from idiopathic avascular necrosis of the the epiphysis of
the head of the femur. It is a fairly rare disease, with approximately 5.5 cases occurring per 100,000
children, annually. Typically, patients have a history of pain in association with weight-bearing activity,
including walking. Clinically, the most significant finding is decreased range of motion of the affected joint.
Often, pain is referred to the knee, while muscles of the thigh and leg may atrophy, exacerbating the
effects on movement. Additionally, osteoarthritis may develop, as a result of the rubbing between the
degenerating femoral head and the acetabulum.

The most common cause of medial collateral ligament sprain (choice C) is a sudden impact to the lateral
aspect of the knee joint, which is not part of this child's history. The trauma causes the lateral side of the
knee to collapse inward toward the midline of the body. The inside of the knee (where the medial collateral
ligament is located) then widens and opens up. This, in turn, stretches the ligament, resulting in the injury.
Osgood-Schlatter disease (choice B) is a condition of pain and inflammation of the tibial tubercle
apophysis, with knee pain localized over the tubercle. It is most common in adolescent athletes, especially
those involved in jumping sports.

A 52 year old woman presents with double vision and bulging of the right eye. Her symptoms have
progressively worsened over the past 18 months.

Physical exam shows proptosis of the right globe and thickening of the temporal bone. There is 3rd and
4th cranial nerve palsies on the right side.

What is the most likely diagnosis?

9
Angioma of the orbit
Optic nerve glioma
Temporal bone sarcoma
Sphenoid wing meningioma
Skull metastases

Explanation

The correct answer is Choice D.

10
A sphenoid wing meningioma is a benign tumor located near the sphenoid bone that originates from the
dura mater. Meningiomas constitute about 15% of all CNS neoplasms; they are more common in females
and in patients over the age of 50. Meningiomas of the anterior skull base constitute 40% of all intracranial
meningiomas; about half occur in the sphenoid wing while the other half in the tuberculum sella. Sphenoid
wing meningiomas are the most common among basal meningiomas. These meningiomas are associated
with hyperostosis of the sphenoid ridge. This tumor may expand into the wall of the cavernous sinus
(medially), into the orbit (anteriorly) and into the temporal bone (laterally).

Tumors growing in the inner wing often cause damage to the optic nerve that lead to a decrease in visual
acuity, a progressive loss of color vision, visual defects and an afferent pupillary defect. As the tumor
continues to grow and push on the optic nerve, the nerve atrophies causing blindness. Proptosis may be
due to bony hyperostosis and invasion of the meningioma in the orbit or cavernous sinus. Diplopia is
present due to orbital/cavernous sinus invasion or due to a herniated bud into the posterior fossa where
the 3rd and 4th cranial nerves are adherent to the tumor.

Among the answers, only a sphenoid wing meningioma would give proptosis, visual impairment and
hyperostosis of the temporal bone.

Hemangiomas are the most common intraorbital tumors found in adults (choice A). These benign lesions
are slow growing and manifest as a painless, progressive proptosis. Visual impairment, diplopia and
extraocular muscle weakness can result from the angioma compressing intraorbital structures. Most
patients that present with clinically significant cavernous hemangiomas are middle-aged.

Optic gliomas (choice B) are rare, slow-growing, benign tumors that occur in children, usually before the
age of 20. There is a strong association between optic glioma and neurofibromatosis type 1. It can affect
one or both of the optic nerves or the optic chiasm. Symptoms are due to tumor compression on the optic
nerve and nearby structures and include: involuntary eyeball movement, proptosis, squinting and vision
loss.

Temporal bone sarcomas (choice C) are exceptionally rare neoplasms and tend to occur in children and

11
young adults. Symptoms include headache, diplopia, visual defects and hyperostosis.

Skull metastases (choice E) are rare and usually arise from a primary neoplasm of the prostate, breast,
lung, head/ neck or lymphoma. The cardinal sign of metastatic skull base invasion is cranial neuropathy
and is typically sudden in onset. Although skull base metastases are often painless, localized pain at the
site of tumor invasion may be a symptom. Hyperostosis may be a sign of metastatic disease, especial
prostate carcinoma.

An 11-year-old boy has complained of pain in the region of the right upper thigh for 2 weeks. This pain is
worse at night and is relieved by aspirin. On physical examination no abnormal findings are noted. A
radiograph of his right leg reveals a small lucent area in the proximal femoral cortex surrounded by
sclerotic bone.

Which of the following is the most likely diagnosis?

Enchondroma
Multiple myeloma
Eosinophilic granuloma
Osteoid osteoma
Osteochondroma

Explanation

The correct answer is choice D

Osteoid Osteoma is a benign bone tumor originating from osteoblastic cells and affects older children and
adolescents. It is said not to be a true tumor because it involves just slight elevation of a sclerotic cortex
that encloses a radiolucent nidus. Due to the very slight elevation of the cortex, it will not be noticed on
physical examination except on a radiograph. It is painful and characteristically known to be relieved by
administration of aspirin.

The diagnosis can not be enchondroma because, enchondroma is symptomless except when it results in a
pathological fracture or swelling from expansion of the cortex.

Multiple myeloma shows numerous areas of punched out rarefactions on X-ray and the bone cortex is
unaffected, whereas, in the child above, the cortex is slightly elevated on radiograph. Multiple myeloma is
usually sited in the marrows of the skull, vertebrae, pelvis, ribs and proximal parts of the femora and
humera, the patient have pains over these affected bones and response to aspirin is not in the same way-
so it is not what our patient has. Notably it is seen in adults over the age of 40, our patient is just 11.

Eosinophilic granuloma is a disorder with a variety of tumor-like lesions which arises from the Langerhand-
type histiocytes. It presents in many ways similar to that of the index case, except that radiology shows
punched out radiolucent lesion. There is also a permeative lesion which invades and destroys the entire
cortex unlike in the patient above where there is just a small lucent region.

The patient above is also not suffering osteochondroma because it involves benign proliferation of

12
cartilage, occurs mostly on the feet, and sometimes shoulder and pelvic girdles.

A 35 year old intoxicated man presents to the hospital with a deep laceration to his right distal forearm
after putting his hand though a window. On examination, he has no sensation to his little finger and the
ulnar aspect of his ring finger. Power on abduction and adduction of all fingers is markedly decreased. He
is unable to flex the distal joint of his little finger. When he flexes the joints at the finger bases and tries to
keep the interphalangeal joints straight, he cannot do so with the fourth and fifth digits but does well with
digits 2 and 3. When he flexes his wrist, his hand deviates radially. The examination is otherwise normal,
and by the time you examine him he is sober enough to cooperate with your instructions and seems to
undertand them adequately.

What structures have been damaged?

ulnar nerve
flexor carpi ulnaris and flexor digitorum profundus muscles
median nerve
radial nerve
brachioradialis

Explanation

The correct answer is choice A.

The ulnar nerve is susceptible to damage along the distal ulna and enters Guyon's canal at the ulnar aspect
of the wrist. Ulnar nerve entrapment at the wrist is less common than median nerve entrapment at the
wrist (carpal tunnel syndrome), but it does occur. Here, the injury is likely to be a cutting of the nerve and
not an entrapment.

The ulnar nerve provides sensation to the palmar aspects of digit 5 and the ulnar aspect of digit 4, also the
parts of the palm that are proximal to those structures. It also provides all motor innervation for digit
abduction and adduction, except the thumb. Finally, it innervates lumbricals for digits 4 and 5 but not 2
and 3. Absence of ulnar nerve innervation but presence of median nerve innervation produces a
characteristic claw-hand appearance, with digits 4 and 5 hyperextended at their bases and flexed at their
interphalangal joints (because the lumbricals do not work). An examining physician may have missed that
appearance while trying to bandage up a laceration or may have assumed that the patient is protecting his
hand from pain.

If a tendon is cut or torn competely, then a bulge of the freed muscle will be evident when the patient tries
to use that muscle. No such bulge was found.

The radial nerve has no motor function in the intrinsic hand muscles and, except for the brachioradialis,
innervates no muscles other than extensors. No sensory or extensor loss was found attributable to that
nerve. The brachioradialis flexes the elbow and is strongest when the forearm is in mid-pronation. No
weakness was found.

A 29 year old male is working in his garden one weekend and develops low back pain and right leg pain.
The pain is very severe and has persisted for 2 weeks before he comes to see you. The pain radiates to

13
the top of his foot and is associated with numbness over the great toe, first web space and the dorsum on
the foot. The medial hamstring muscle deep tendon reflex is absent on the right and 2+ on the left.

Where is the most likely location of his lesion?

conus medullaris in the L4 segment


L5 nerve root
Right sacrospinus ligament
Right hip joint
Posterior tibia nerve

Explanation

The correct answer is choice B

Radicular pain, or radiculitis, is pain "radiated" along the dermatome (sensory distribution) of a nerve due
to inflammation or other irritation of the nerve root (radiculopathy) at its connection to the spinal column.
A common form of radiculitis is sciatica – radicular pain that radiates along the sciatic nerve from the lower
spine to the lower back, gluteal muscles, back of the upper thigh, calf, and foot as often secondary to
nerve root irritation from a spinal disc herniation or from osteophytes in the lumbar region of the spine.

The symptoms of radiculitis may change from one patient to the next, depending on which nerve root is
affected. For example, if the L5 nerve root (in the L5-S1 lumbar region) is compromised, a classic
symptom is numbness in the big toe. Patients suffering from L5 radiculitis may have difficulty finding a
comfortable position.

There are also a variety of surgeries that can be employed to treat severe cases of radicular pain,
depending on the underlying condition that the surgery addresses. To treat a disc herniation, which may
cause persistent radiating pain, a microdiscectomy surgery is usually performed. This is a minimally
invasive approach that removes the portion of the disc that presses against the nerve root. The surgery
has a high success rate, minimal healing time (typically the patient will go home on the same day as the
surgery), and usually provides immediate relief of the sciatica and other symptoms caused by a herniated
disc. This surgery may be recommended after several weeks of non-surgical treatment, or even earlier if
the pain and other sciatica symptoms are severe.

Cauda equina syndrome in the L4 segment (choice A) is a serious neurologic condition in which there is
acute loss of function of the lumbar plexus . It would manifest as muscle weakness (often paraplegia),
sphincter weaknesses causing urinary retention and post-void residual incontinence as assessed by
catheterizing after the patient has urinated. Also, there may be decreased anal tone; sexual dysfunction;
saddle anesthesia; bilateral leg pain and weakness; and bilateral absence of ankle reflexes.

Right sacrospinus ligament (choice C) is a thin, triangular ligament attached by its apex to the ischial
spine, and medially, by its broad base, to the lateral margins of the sacrum and coccyx, in front of the
sacrotuberous ligament with which its fibers are intermingled. Together with the sacrotuberous ligament, it
converts the greater sciatic notch into the greater sciatic foramen and the lesser sciatic notch into the
lesser sciatic foramen. The pudendal vessels and nerve pass behind the sacrospinous ligament directly
medially and inferiorly to the ischial spine. The inferior gluteal artery, from a branch of the internal iliac
artery, pass behind the sciatic nerve and the sacrospinous ligament and is left uncovered in a small
opening above the top of the sacrospinous ligament. A lesion at this site would not explain L5 radiculular
back pain.

Right hip joint (choice D) would not explain the signs and symptoms of L5 radiculular back pain. Many back
and spine problems can cause symptoms around the buttocks and hip. The most common problems that

14
refer pain to the hip region are herniated discs and sciatica.

Posterior tibia nerve (choice E) is a branch of the sciatic nerve that passes through the popliteal fossa to
pass below the arch of soleus. Tibial nerve dysfunction is a form of peripheral neuropathyperipheral
neuropathy. Symptoms from this lesion would include changes in sensation at the bottom of foot such as
burning and numbness. Furthermore, there would be weakness of the knee or foot leading to difficulty with
walking.

A 13-year-old boy complains of pain persisting in his left leg for 3 weeks. On physical examination his temperature is 37.9
C. A radiograph of the leg reveals a mass in the diaphyseal region of the left femur with overlying cortical erosion and soft
tissue extension. A bone biopsy is performed and the lesion on microscopic examination shows numerous small round
blue cells.

Which of the following neoplasms is he most likely to have?

Ewing sarcoma
Medulloblastoma
Neuroblastoma
Chondroblastoma
Osteoblastoma

Click on image to Zoom

Explanation

The correct answer is Choice A

Ewing’s sarcoma is a malignant primary bone tumour that usually occurs in the diaphyses of long bones. It
is commoner in male children, occurring mostly between the ages of 4 – 15 years and rare in adults older
than 30 years of age. Common sites are the femur, humerus, tibia, pelvis and ribs.

Clinically, Ewing’s sarcoma may mimic an infective process. It presents with fever, localized swelling and
redness. The earliest symptom is pain which is initially intermittent, but gradually becomes more intense.
Other findings include weight loss, mild anaemia, leucocytosis, elevated ESR and increased serum lactic
dehydrogenase levels

Radiological findings include a typical lamellated or ‘onion skin’ periosteal reaction with destruction of the
medulla and periosteal elevation of the cortex of the affected bone. Histology is required to confirm the
diagnosis before commencing treatment. Microscopically, the tumour is seen as densely packed uniform
small cells arranged in sheets cells with scant cytoplasm showing a single oval or round nucleus without
prominent nucleoli. Treatment usually involves a combination of radiotherapy and cytotoxic chemotherapy.

Medulloblastoma (Choice B) is the most common malignant intracranial tumour of childhood, arising in the
embryonic tissue of the cerebellar vermis. 70% occurs before 8 years of age.

15
Neuroblastoma (Choice C) is an APUD (Amine Precursor Uptake Decarboxylase) tumour derived from
neural crest cell tissue. Common sites include Adrenal medulla (50%), Abdominal sympathetic ganglia
(25%), Chest (20%), Pelvis (5%), Neck (5%). 50% of tumours are found before the age of 2 years and
90% before the age of 9 years.

Chondroblastoma (Choice D) is a rare benign tumour arising from chondroblast in epiphyses of long bones.

Osteoblastoma (Choice E) is typically found in short flat bones e.g. in the hands and vertebrae. It is
common between the ages of 10 – 13 years.

A 45 year woman presents for annual checkup. She has no significant medical history except osetoarthritis
of the knees. She is an avid tennis player and has been experiencing increasing point tenderness in her
left elbow over the past month. She also mentions that her wrist hurts when she grips a cup of coffee in
the morning. The following maneuver in the diagram is performed.

The maneuver shown below is used to elicit a sign of which one of the following musculoskeletal
problems?

Bicipital tendonitis
Radial nerve entrapment syndrome
Lateral epicondylitis
Medial epicondylitis
Olecranon bursitis

Click on image to Zoom

Explanation

The correct answer is choice C

Lateral epicondylitis, also known as tennis elbow is a tendonosis of the extensor tendon origin of the
forearm. Pain in the elbow can be elicited by resisted wrist extension as seen in the diagram. It is
commonly associated with playing tennis and other racquet sports.

The pathophysiology of lateral epicondylitis is degenerative. Non-inflammatory, chronic degenerative


changes of the origin of the extensor carpi radialis brevis (ECRB) muscle. The extensor digiti minimi also
has a small origin site medial to the elbow which can be affected by this condition. The muscle involves the
extension of the fifth digit and some extension of the wrist allowing for adaption to “snap” or flick the wrist
– usually associated with a racquet swing. Most often, the extensor muscles become painful due to tendon
breakdown from over extension. Improper form or movement allows for power in a swing to rotate through
and around the wrist – creating a moment on that joint instead of the elbow joint or rotator cuff. This
moment causes pressure to build impact forces to act on the tendon causing irritation and inflammation.

Symptoms of lateral epicondylitis include pain on the outer part of elbow and point tenderness over the
lateral epicondyle – a prominent part of the bone on the outside of the. Gripping and movements of the
wrist hurt, especially wrist extension and lifting movements. Activities that use the muscles that extend the
wrist (e.g. pouring a pitcher or gallon of milk, lifting with the palm down) are characteristically painful.

The strongest risk factor for lateral epicondylitis is age. The peak incidence is between 30 to 60 years of
age. Rest is the tennis player's treatment of choice when the pain first appears; the rest allows the tiny
tears in the tendon attachment to heal. In recalcitrant cases surgery may be indicated. Most techniques
aim to release the strain on the extensor carpi radialis brevis muscle, remove degenerative tissue and

16
promote healing. Non-specific palliative treatments include:

 Physical Therapy- most important part of the treatment. it includes various modalities for
preventing and treating tennis elbow.
 Non-steroidal anti-inflammatory drugs (NSAIDs): ibuprofen, naproxen or aspirin
 Heat or ice
 A counter-force brace or "tennis elbow strap" to reduce strain at the elbow epicondyle, to limit
pain provocation and to protect against further damage.

Bicipital tendonitis (choice A) is tendonitis of the biceps insertion and can be tested with pain associated
with resisted supination and elbow flexion.

Radial nerve entrapment syndrome (choice B) can occur with lateral epicondylitis, however, the phyisical
exam manuever seen in the diagram is not for this pathology.

Medial epicondylitis (choice D) results in medial elbow pain and can be elicited with forced wrist flexion

Olecranon bursitis (choice E) is a collection of fluid in the olecranon bursa that can be inflamed.

A 16-year-old boy has noted pain in his left knee after each hockey practice session for the past month.
The pain has been increasing in intensity and is not relieved with NSAIDs, ice, rest, or elevation. On
examination there is tenderness to palpation of his left knee, with reduced range of motion. Anterior
drawer sign is negative. A plain film radiograph of the left leg is taken.

Which of the following neoplasms is he most likely to have?

Ewing sarcoma
Osteosarcoma
Chondrosarcoma
Multiple myeloma
Metastatic seminoma

Click on image to Zoom

Explanation

The correct answer is choice B

Osteosarcoma is an aggressive cancerous neoplasm arising from primitive transformed cells of


mesenchymal origin that exhibit osteoblastic differentiation and produce malignant osteoid. It is the most
common histological form of primary bone cancer. Osteosarcoma is the eighth most common form of
childhood cancer, comprising 2.4% of all malignancies in pediatric patients, and approximately 20% of all

17
bone cancers.

There is a preference for origination in the metaphyseal region of tubular long bones, with 42% occurring
in the femur, 19% in the tibia, and 10% in the humerus. About 8% of all cases occur in the skull and jaw,
and another 8% in the pelvis. The tumour may be localised at the end of the long bone. Most often it
affects the upper end of tibia or humerus, or lower end of femur. Osteosarcoma tends to affect regions
around the knee in 60% of cases, 15% around the hip, 10% at the shoulder, and 8% in the jaw. The
tumor is solid, hard, irregular ("fir-tree," "moth-eaten" or "sun-burst" appearance on X-ray examination)
due to the tumor spicules of calcified bone radiating in right angles. These right angles form what is known
as Codman's triangle. Surrounding tissues are infiltrated.

The most commonly affected bones are the proximal humerus, the distal radius, the distal femur, and the
tibia, following the basic premise "far from the elbow, close to the knee". Other sites include the ribs, the
mandible, the spine, and the pelvis. Rarely, osteosarcoma may arise from soft-tissues (extraskeletal
osteosarcoma). Metastasis of tumors involving the limb bones is very common, usually to the lungs. The
tumor causes a great deal of pain, and can even lead to fracture of the affected bone. As with human
osteosarcoma, bone biopsy is the definitive method to reach a final diagnosis. Osteosarcoma should be
differentiated from other bone tumours and a range of other lesions, such as osteomyelitis. Differential
diagnosis of the osteosarcoma of the skull in particular includes, among others, chondrosarcoma and the
multilobular tumour of bone.

Microscopically, the characteristic feature of osteosarcoma is presence of osteoid (bone formation) within
the tumour. Tumor cells are very pleomorphic (anaplastic), some are giant, numerous atypical mitoses.
These cells produce osteoid describing irregular trabeculae (amorphous, eosinophilic/pink) with or without
central calcification (hematoxylinophilic/blue, granular) - tumor bone. Tumor cells are included in the
osteoid matrix. Depending on the features of the tumour cells present (whether they resemble bone cells,
cartilage cells or fibroblast cells), the tumour can be subclassified. Osteosarcomas may exhibit
multinucleated osteoclast-like giant cells.

Family physicians and orthopedists rarely see a malignant bone tumor (most bone tumors are benign).
Thus, many patients are initially misdiagnosed with cysts or muscle problems, and some are sent straight
to physical therapy without an x-ray. The route to osteosarcoma diagnosis usually begins with an x-ray,
continues with a combination of scans (CT scan, PET scan, bone scan, MRI) and ends with a surgical
biopsy. Films are suggestive, but bone biopsy is the only definitive method to determine whether a tumor
is malignant or benign.

Complete radical surgical en bloc resection is the treatment of choice in osteosarcoma. Although about
90% of patients are able to have limb-salvage surgery, complications, such as infection, prosthetic
loosening and non-union, or local tumor recurrence may cause the need for further surgery or amputation.

Ewing sarcoma (choice A) is a malignant round-cell tumor. It is a rare disease in which cancer cells are
found in the bone or in soft tissue. The most common areas in which it occurs are the pelvis, the femur,
the humerus, and the ribs. On conventional radiographs, the most common osseous presentation is a
permeative lytic lesion with periosteal reaction. The classic description of lamellated or "onion skin" type
periosteal reaction is often associated with this lesion.

Chondrosarcoma (choice C) is a cartilage-based tumor and is in a category of cancers called sarcomas.


About 25% of primary bone cancers are chondrosarcomas. This disease can affect people of any age,
although it is more common among older people than among children. The most characteristic imaging
findings are on CT, where detection of a chondroid matrix is present.

Multiple myeloma (choice D) is a cancer of plasma cells, a type of white blood cell normally responsible for
the production of antibodies. Collections of abnormal cells accumulate in bones, where they cause bone
lesions, and in the bone marrow where they interfere with the production of normal blood cells. Most cases
of myeloma also feature the production of a paraprotein, an abnormal antibody that can cause kidney
problems and interferes with the production of normal antibodies leading to immunodeficiency.
Hypercalcemia are often encountered.

The peak age of onset of multiple myeloma is 65 to 70 years of age. The workup of suspected multiple
myeloma includes a skeletal survey. This is a series of X-rays of the skull, axial skeleton and proximal long

18
bones. Myeloma activity sometimes appear as "lytic lesions" (with local disappearance of normal bone due
to resorption), and on the skull X-ray as "punched-out lesions" (pepper pot skull).

Metastatic seminoma (choice E) is a germ cell tumor of the testis. It is one of the most treatable and
curable cancers, with survival >95% in the early stages. The average age of diagnosis is 40 years. This is
about 5 to 10 years older than men with other germ cell tumors of the testes. In most cases they produce
masses that are readily felt on testicular self-examination. Testicular pain is reported in up to one fifth of
cases. Low back pain may occur after metastasis to the retroperitoneum

A 16-year-old boy has noted pain in his left knee after each hockey practice session for the past month.
The pain has been increasing in intensity and is not relieved with NSAIDs, ice, rest, or elevation. On
examination there is tenderness to palpation of his left knee, with reduced range of motion. Anterior
drawer sign is negative. A plain film radiograph of the left leg is taken.

Which of the following neoplasms is he most likely to have?

Ewing sarcoma
Osteosarcoma
Chondrosarcoma
Multiple myeloma
Metastatic seminoma

Click on image to Zoom

Explanation

The correct answer is choice B

Osteosarcoma is an aggressive cancerous neoplasm arising from primitive transformed cells of


mesenchymal origin that exhibit osteoblastic differentiation and produce malignant osteoid. It is the most
common histological form of primary bone cancer. Osteosarcoma is the eighth most common form of
childhood cancer, comprising 2.4% of all malignancies in pediatric patients, and approximately 20% of all
bone cancers.

There is a preference for origination in the metaphyseal region of tubular long bones, with 42% occurring
in the femur, 19% in the tibia, and 10% in the humerus. About 8% of all cases occur in the skull and jaw,
and another 8% in the pelvis. The tumour may be localised at the end of the long bone. Most often it
affects the upper end of tibia or humerus, or lower end of femur. Osteosarcoma tends to affect regions
around the knee in 60% of cases, 15% around the hip, 10% at the shoulder, and 8% in the jaw. The
tumor is solid, hard, irregular ("fir-tree," "moth-eaten" or "sun-burst" appearance on X-ray examination)
due to the tumor spicules of calcified bone radiating in right angles. These right angles form what is known
as Codman's triangle. Surrounding tissues are infiltrated.

The most commonly affected bones are the proximal humerus, the distal radius, the distal femur, and the
tibia, following the basic premise "far from the elbow, close to the knee". Other sites include the ribs, the

19
mandible, the spine, and the pelvis. Rarely, osteosarcoma may arise from soft-tissues (extraskeletal
osteosarcoma). Metastasis of tumors involving the limb bones is very common, usually to the lungs. The
tumor causes a great deal of pain, and can even lead to fracture of the affected bone. As with human
osteosarcoma, bone biopsy is the definitive method to reach a final diagnosis. Osteosarcoma should be
differentiated from other bone tumours and a range of other lesions, such as osteomyelitis. Differential
diagnosis of the osteosarcoma of the skull in particular includes, among others, chondrosarcoma and the
multilobular tumour of bone.

Microscopically, the characteristic feature of osteosarcoma is presence of osteoid (bone formation) within
the tumour. Tumor cells are very pleomorphic (anaplastic), some are giant, numerous atypical mitoses.
These cells produce osteoid describing irregular trabeculae (amorphous, eosinophilic/pink) with or without
central calcification (hematoxylinophilic/blue, granular) - tumor bone. Tumor cells are included in the
osteoid matrix. Depending on the features of the tumour cells present (whether they resemble bone cells,
cartilage cells or fibroblast cells), the tumour can be subclassified. Osteosarcomas may exhibit
multinucleated osteoclast-like giant cells.

Family physicians and orthopedists rarely see a malignant bone tumor (most bone tumors are benign).
Thus, many patients are initially misdiagnosed with cysts or muscle problems, and some are sent straight
to physical therapy without an x-ray. The route to osteosarcoma diagnosis usually begins with an x-ray,
continues with a combination of scans (CT scan, PET scan, bone scan, MRI) and ends with a surgical
biopsy. Films are suggestive, but bone biopsy is the only definitive method to determine whether a tumor
is malignant or benign.

Complete radical surgical en bloc resection is the treatment of choice in osteosarcoma. Although about
90% of patients are able to have limb-salvage surgery, complications, such as infection, prosthetic
loosening and non-union, or local tumor recurrence may cause the need for further surgery or amputation.

Ewing sarcoma (choice A) is a malignant round-cell tumor. It is a rare disease in which cancer cells are
found in the bone or in soft tissue. The most common areas in which it occurs are the pelvis, the femur,
the humerus, and the ribs. On conventional radiographs, the most common osseous presentation is a
permeative lytic lesion with periosteal reaction. The classic description of lamellated or "onion skin" type
periosteal reaction is often associated with this lesion.

Chondrosarcoma (choice C) is a cartilage-based tumor and is in a category of cancers called sarcomas.


About 25% of primary bone cancers are chondrosarcomas. This disease can affect people of any age,
although it is more common among older people than among children. The most characteristic imaging
findings are on CT, where detection of a chondroid matrix is present.

Multiple myeloma (choice D) is a cancer of plasma cells, a type of white blood cell normally responsible for
the production of antibodies. Collections of abnormal cells accumulate in bones, where they cause bone
lesions, and in the bone marrow where they interfere with the production of normal blood cells. Most cases
of myeloma also feature the production of a paraprotein, an abnormal antibody that can cause kidney
problems and interferes with the production of normal antibodies leading to immunodeficiency.
Hypercalcemia are often encountered.

The peak age of onset of multiple myeloma is 65 to 70 years of age. The workup of suspected multiple
myeloma includes a skeletal survey. This is a series of X-rays of the skull, axial skeleton and proximal long
bones. Myeloma activity sometimes appear as "lytic lesions" (with local disappearance of normal bone due
to resorption), and on the skull X-ray as "punched-out lesions" (pepper pot skull).

Metastatic seminoma (choice E) is a germ cell tumor of the testis. It is one of the most treatable and
curable cancers, with survival >95% in the early stages. The average age of diagnosis is 40 years. This is
about 5 to 10 years older than men with other germ cell tumors of the testes. In most cases they produce
masses that are readily felt on testicular self-examination. Testicular pain is reported in up to one fifth of
cases. Low back pain may occur after metastasis to the retroperitoneum

A 23 year old male was involved in a snowmobile accident and sustained a closed, displaced humeral shaft
fracture. His neurovascular status is normal. You reduce the fracture an place him in a splint. Prior to
discharge, he states he has numbness over the dorsum of his hand and he is unable to extend his

20
metacarpophalangeal joints.

Which of the following would you suspect to be the cause?

ulnar nerve was damaged at the time of the fracture


radial nerve was damaged at the time of the fracture
radial nerve is trapped in the fracture site
ulnar nerve is compressed by hematoma
median nerve is trapped in fracture site

Explanation

The correct answer is choice C

A humerus fracture can be classified by the location of the humerus involved: the upper end, the shaft, or
the lower end. Certain lesions are commonly associated with fractures to specific areas of the humerus. At
the upper end, the surgical neck of the humerus and anatomical neck of humerus can both be involved,
though fractures of the surgical neck are more common. The axillary nerve can be damaged in fractures of
this type. Mid-shaft fractures may damage the radial nerve, which traverses the lateral aspect of the
humerus closely associated with the radial groove. The median nerve is vulnerable to damage in the
supracondylar area, and the ulnar nerve is vulnerable near the medial epicondyle, around which it curves
to enter the forearm.

The radial nerve is a nerve in the human body that supplies the upper limb. It supplies the triceps brachii
muscle of the arm, as well as all 12 muscles in the posterior osteofascial compartment of the forearm, as
well as the associated joints and overlying skin.

It originates from the posterior cord of the brachial plexus with roots from C5, C6, C7, C8 & T1.

The radial nerve and its branches supply the dorsal muscles, such as triceps brachii, the extrinsic extensors
of the wrist and hands, and the cutaneous nerve supply to most of the back of the hand. (The ulnar nerve
cutaneously innervates the back of the little finger and adjacent half of the ring finger). The radial nerve
divides into a deep branch (which becomes the posterior interosseous nerve), and continues as the
superficial branch. The superficial branch of the radial nerve provides sensory innervation to much of the
back of the hand, including the web of skin between the thumb and index finger.

ulnar nerve was damaged at the time of the fracture (choice A) would be consistent with damage to the
medial epicondyle. The medial epicondyle of the humerus is larger and more prominent than the lateral
epicondyle. It gives attachment to the ulnar collateral ligament of the elbow-joint, to the Pronator teres,
and to a common tendon of origin of some of the flexor muscles of the forearm. The ulnar nerve runs in a
groove on the back of this epicondyle.

axillary nerve was damaged at the time of the fracture (choice B) would be consistent with damage to the
upper end of the hummerus such as the surgical neck of the humerus.

ulnar nerve is compressed by hematoma (choice D) would not present with numbness over the dorsum of
the hand or inability to extend metacarpophalangeal joints. The ulnar nerve can be trapped or pinched as it
proceeds from the brachial plexus to the fingertips. One common cause is cubital tunnel syndrome. In this
syndrome, a tunnel on the medial side of the elbow traps the nerve. Pinching of the ulnar nerve often
causes paraesthesiae (tingling) in the fourth and fifth digits. In severe cases, surgery is performed to move
the nerve.

median nerve is trapped in fracture site (choice E) would be consistent with damage in the supracondylar

21
area. A supracondylar fracture is a fracture of the distal humerus just above the epicondyles. While
relatively rare in adults it is one of the most common fractures to occur in children. Typically, the child
presents with history of a falling on an outstretched hand followed by pain, swelling and inability to move
the affected elbow. On examination, there is an unusual prominence of olecranon process.

A 68 year old retired cleaner presents to clinic with pains in her 'hand joints'. She is finding simple tasks
like opening buttons increasingly difficult. Her pains tend to get worse as the day progresses. Her hands
do get stiff particularly after use. She has never noticed her hands to be swollen but has noticed little
'swellings on the ends of her fingers' over the last few months and stiffness on a regular basis. Clinically,
you detect synovitis mainly in her DIP joints with sparing of the MCP and PIP joints bilaterally.

What would be recommended first line pharmacological treatment for this patient?

Ibuprophen
Hydrocodone
Methotrexate
Acetaminophen
Sulfasalazine Click on image to Zoom

Explanation

The correct answer is choice D

This patient has osteoarthritis of her hand. The "swellings" of her fingers at the distal interphalangeal joints
are Heberden's nodes. Primary treatment of osteoarthritis begins with NSAIDS or acetaminophen.

Osteoarthritis (OA) also known as degenerative arthritis or degenerative joint disease, is a group of
mechanical abnormalities involving degradation of joints, including articular cartilage and subchondral
bone. Symptoms may include joint pain, tenderness, stiffness, locking, and sometimes an effusion. A
variety of causes—hereditary, developmental, metabolic, and mechanical—may initiate processes leading
to loss of cartilage. When bone surfaces become less well protected by cartilage, bone may be exposed and
damaged. As a result of decreased movement secondary to pain, regional muscles may atrophy, and
ligaments may become more lax.

The main symptom is pain, causing loss of ability and often stiffness. "Pain" is generally described as a
sharp ache, or a burning sensation in the associate muscles and tendons. OA can cause a crackling noise
(called "crepitus") when the affected joint is moved or touched, and patients may experience muscle
spasm and contractions in the tendons. Occasionally, the joints may also be filled with fluid. Humid and
cold weather increases the pain in many patients. OA commonly affects the hands, feet, spine, and the
large weight bearing joints, such as the hips and knees, although in theory, any joint in the body can be
affected. As OA progresses, the affected joints appear larger, are stiff and painful, and usually feel worse,
the more they are used throughout the day, thus distinguishing it from rheumatoid arthritis.

In smaller joints, such as at the fingers, hard bony enlargements, called Heberden's nodes (on the distal
interphalangeal joints) and/or Bouchard's nodes (on the proximal interphalangeal joints), may form, and
though they are not necessarily painful, they do limit the movement of the fingers significantly. OA at the
toes leads to the formation of bunions, rendering them red or swollen. Some people notice these physical
changes before they experience any pain.

OA is the most common cause of joint effusion, sometimes called water on the knee in lay terms, an

22
accumulation of excess fluid in or around the knee joint. Osteoarthritis progresses in stages:

 joint space begins to narrow and osteophytes form


 joint space disappears as cartilage wears away and bone rubs on bone in the joint
 subchondral cysts appear
 bone tries to repair itself and there is bone remodeling

Diagnosis is made with reasonable certainty based on history and clinical examination. X-rays may confirm
the diagnosis. The typical changes seen on X-ray include: joint space narrowing, subchondral sclerosis
(increased bony formation around the joint), subchondral cyst formation, and osteophytes. Plain films may
not correlate with the findings on physical examination or with the degree of pain. Usually other imaging
techniques are not necessary to clinically diagnose osteoarthritis.

OA is the most common form of arthritis and the leading cause of chronic disability in the United States.
Lifestyle modification (such as weight loss and exercise) and analgesics are the mainstay of treatment.
Acetaminophen / paracetamol (choice D) is used first line and NSAIDS (choice A) are only recommended
as add on therapy if pain relief is not sufficient. This is due to the relative greater safety of acetaminophen.
If pain becomes debilitating joint replacement surgery may be used to improve the quality of life.

Hydrocodone (choice B) is an opiate pain medicine that can be used if primary NSAIDs or acetaminophen
do not work. It is not the first line treatment for OA.

Methotrexate (choice C) and Sulfasalazine (choice E) are drugs for rheumatoid arthritis and not indicated
for primary osteoarthritis

Review

Rheumatoid arthritis (RA) is usually a disease of insidious onset, although it can be abrupt. The diagnosis
typically is made when 4 of 7 qualifying criteria established by the American Rheumatism Association are
met. These qualifying criteria are as follows:

 Morning joint stiffness lasting longer than 1 hour before maximal improvement
 Arthritis involving 3 or more joints
 Arthritis of the hand, particularly involvement of the proximal interphalangeal (PIP) joints,
metacarpophalangeal (MCP) joints, or wrist joints
 Bilateral involvement of joint areas (ie, both wrists, symmetric PIP and MCP joints)
 Positive serum rheumatoid factor (RF)
 Rheumatoid nodules
 Radiographic evidence of RA

Other contributing history includes the following:

 General malaise
 Weakness
 Fever of undetermined etiology
 Weight loss
 Myalgias
 Tendonitis
 Bursitis

A 60 year old man develops seizures involving his right side, and complains of increasingly severe
headaches. Examination reveals a mild right hemiparesis and subtle dysphasia. The general physical

23
examination is normal. The past medical history is negative apart from non-insulin dependent diabetes
mellitus. A contrast CT scan shows a solitary left fronto-temporal lesion with irregular enhancement
peripheral, low density center, surrounding edema, and moderate mass effect.

Which of the following is the single most likely diagnosis?

ependymoma
a solitary cerebral metastasis
a cerebral infarction
astrocytoma
craniopharyngioma

Explanation

The correct answer is choice D.

Gliomas are classified as ependymomas, astrocytomas, oligodendrogliomas, and mixed gliomas.


Glioblastoma multiforme (GM) is high grade, the most common astrocytoma, and also the astrocytoma
with the worst prognosis. It is an aggressive tumour that often involves memory loss and deterioration
various other mental functions, and can present with headaches, paralysis, and seizures. In MRI, they
appear as an irregularly-shaped, ring-enhancing lesions. GM is extremely difficult to resect entirely with
surgery, to eliminate with radiation, and because of the blood-brain barrier very few chemotherapeutic
agents can be delivered to it. Other astrocytomas vary in grade and based on this as well as location,
prognosis can vary.

Ependymoma (choice A) develops from ependymal cells and accounts for approximately 30 percent of
primary CNS tumors in patients under 3 years of age, while craniopharyngioma (choice E) is a pituitary
tumour that occurs mostly in the pediatric population.

Since the general physical examination of this man is normal, there is no reason to suspect non-CNS
cancer resulting in cerebral metastasis (choice B), while cerebral infarction (choice C) would not be
associated with progressively-worsening headaches or the mass that was identified on CT.

A 55 year old female complains of numbness and discomfort in her right hand, which is wakening her from
sleep. Similar symptoms occur while steering the car or writing. You suspect carpal tunnel syndrome.

Which of the following would lead you to consider alternative diagnosis?

absence of sensory loss on testing with pinprick


absence of thenar muscle wasting
absence of weakness of abductor pollicis brevis
presence of rheumatoid arthritic changes in the hands
wasting of the thenar, hypothenar and interosseous muscles

Explanation

24
The correct answer is choice E

Carpal Tunnel Syndrome (CTS) is associated by symptoms and signs, which are caused by compression of
the median nerve travelling through the carpal tunnel. Carpal Tunnel Syndrome affects the hands since it is
an upper limb neuropathy that results in motor and sensory disturbance of the median nerve.

The carpal tunnel is an anatomical compartment located at the base of the wrist. Nine flexor tendons and
the median nerve pass through the carpal tunnel that is surrounded on three sides by the carpal bones
that form an arch. The nerve and the tendons provide function, feeling, and movement to some of the
fingers. The finger and wrist flexor muscles including their tendons originate in the forearm at the medial
epicondyle of the elbow joint and attach to the Metaphalangeal (MP), Proximal Interphalangeal (PIP), and
Distal Interphalangeal bones of the fingers and thumb (BSI). The carpal tunnel is approximately as wide as
the thumb and its boundary lies at the distal wrist skin crease and extends up the arm towards the elbow
for approximately 3 cm.

The median nerve can be compressed by a decrease in the size of the canal, an increase in the size of the
contents (such as the swelling of lubrication tissue around the flexor tendons), or both. Simply flexing the
wrist to 90 degrees will decrease the size of the canal. The median nerve innervates the “LOAF” muscles of
the thenar eminence; the first and second lumbricals, opponens pollicus, abductor pollicus brevis and part
of flexor pollicus brevis. This nerve supplies sensation to the thumb, index, middle and half of the fourth
finger. It’s important to note that sensation over the thenar eminence — the palm just proximal to the
thumb — should be spared in carpal tunnel syndrome, as the palmar cutaneous sensory branch comes off
before the carpal tunnel. If this sensory branch is involved, the median nerve may be compressed more
proximally between the heads of the pronator teres.

This condition affects individuals by causing pain, paresthesias, and sometimes weakness in the median
nerve distribution. Those diagnosed with Carpal Tunnel Syndrome may experience pain, numbness and
tingling sensations in the arm, which may extend to the shoulder and neck area; these feelings are more
prevalent at night due to various sleeping positions. To aid in the prevention of Carpal Tunnel Syndrome,
stretching exercises of the wrist, hand, and fingers have been used to combat against the pain and
numbness caused by repetitive actions. Other than using recommended stretches and exercises, useful
treatments for CTS include use of night splints, corticosteroid injections and ultimately surgery.

Hypothenar refers to a group of three muscles of the palm that control the motion of the little finger. The
muscles of hypothenar eminence are from lateral to medial:

 Opponens digiti minimi


 Flexor digiti minimi
 Abductor digiti minimi

The intrinsic muscles of hand can be remembered using the mnemonic, "A OF A OF A" for, Abductor pollicis
brevis, Opponens pollicis, Flexor pollicis brevis, Adductor pollicis (thenar muslces) and Opponens digiti
minimi, Flexor digiti minimi, Abductor digiti minimi (Hypothenar muscles). Hypothenar atrophy (choice E)
is associated with the lesion of the ulnar nerve, which supplies the three hypothenar muscles.

absence of sensory loss (choice A) over the thenar eminence would strongly suggest carpel tunnel
syndrome. Sensation over the thenar eminence — the palm just proximal to the thumb — should be spared
in carpal tunnel syndrome, as the palmar cutaneous sensory branch comes off before the carpal tunnel.

thenar muscle wasting (choice B) would strongly suggest carpel tunnel syndrome. The median nerve
innervates the “LOAF” muscles of the thenar eminence; the first and second lumbricals, opponens pollicus,
abductor pollicus brevis and part of flexor pollicus brevis. The thenar muscles atrophy when the median
nerve is compressed.

weakness of abductor pollicis brevis (choice C) would strongly suggest carpel tunnel syndrome. The
median nerve innverates the abductor pollicis brevis.

presence of rheumatoid arthritic changes in the hands (choice D) would strongly suggest carpel tunnel
syndrome. Rheumatoid arthritis causes inflammation of the flexor tendons, leading to an increase in the

25
size of the carpel tunnel contents.

A 6 year old boy presents to pediatric clinic with a limp for the past 6 months. According to his mother, he
has been experiencing an aching right hip and knee pain usually after 4 hours of movement, which she
originally attributed to growing pains. On examination, you notice reduced range of motion at the right hip
joint and an antalgic gait. There is mild atrophy of the right thigh muscle compared to the left. An AP
radiograph of the pelvis is taken.

What is the cause of the patient's limp?

Legg-Calve-Perthes' disease
Septic arthritis
Slipped capital femoral epiphysis
Juvenile rheumatoid arthritis
Osteosarcoma

Click on image to Zoom


Explanation

The correct answer is choice A

Legg–Calvé–Perthes syndrome is a degenerative disease of the hip joint, where growth/loss of bone mass
leads to some degree of collapse of the hip joint and to deformity of the ball of the femur and the surface
of the hip socket. The disease is characterized by idiopathic avascular osteonecrosis of the capital femoral
epiphysis of the femoral head leading to an interruption of the blood supply of the head of the femur close
to the hip joint. For example, a child may be 6 years old chronologically but may have grown to 4 years old
in terms of bone maturity. The child may then engage in activity appropriate for a child of 6 but may not
yet have the bone strength of an older child, leading to flattening or fracture of the hip joint.

Common symptoms include hip, knee, or groin pain, exacerbated by hip/leg movement. The pain feels like
a tooth ache, possibly severe. There is a reduced range of motion at the hip joint and a painful or antalgic
gait. There may be atrophy of thigh muscles from disuse and an inequality of leg length. In some cases,
some activity can cause severe irritation or inflammation of the damaged area including standing, walking,
running, kneeling, or stooping repeatedly for an extended period of time.

The first signs are complaints of soreness from the child, which are often dismissed as growing pains, and
limping or other guarding of the joint, particularly when tired. The pain is usually in the hip, but can also
be felt in the knee as referred pain. In some cases, pain is felt in the unaffected hip and leg, due to the
child favoring the injured side and placing the majority of their weight on the "good" leg. It is
predominantly a disease of boys (4:1 ratio). Whereas Perthes is generally diagnosed between 5 and 12
years of age, it has been diagnosed as early as 2 years of age. Typically the disease is only seen in one
hip, but bilateral Perthes is seen in about 8-10% of children diagnosed. Onset of pain may be up to 4 hours
after inactivity. Knee pain is felt in the back of the knee rather than in the front, not unlike a localized
charley horse. This lasts for an hour or so and returns nightly on inactivity.

X-Rays of the hip confirm the diagnosis. X-rays usually demonstrates a flattened and later fragmented
head of femur. A bone scan or MRI may be useful in making the diagnosis in those cases where x-rays are
inconclusive. Neither bone scan or MRI offer any additional useful information beyond that of x-rays in an
established case. If MRI or bone scans are necessary a positive diagnosis relies upon patchy areas of
vascularity to the capital femoral epiphysis (the developing femoral head).

26
The goal of treatment is to avoid severe degenerative arthritis. Orthopedic assessment is crucial. Younger
children have a better prognosis than older children. Treatment has traditionally centered on removing
pressure from the joint until the disease has run its course. Options include traction (to separate the femur
from the pelvis and reduce wear) braces (often for several months, with an average of 18 months) to
restore range of motion, physiotherapy, and surgical intervention when necessary because of permanent
joint damage.

Septic arthritis (choice B) is the purulent invasion of a joint by an infectious agent which produces arthritis.
Septic arthritis should be considered whenever one is assessing a patient with joint pain. Usually only one
joint is affected (monoarthritis) however in seeding arthritis, several joints can be affected simultaneously;
this is especially the case when the infection is caused by staphylococcus or gonococcus bacteria. The
absence of signs of infection makes septic arthritis unlikely.

Slipped capital femoral epiphysis (choice C)is a fracture through the physis (the growth plate), which
results in slippage of the overlying epiphysis. Often this condition will present in obese prepubescent
males, especially young black males, and sometimes females with an insidious onset of thigh or knee pain
with a painful limp. Hip motion will be limited, particularly internal rotation. Slipped capital femoral
epiphysis is a common cause of hip and knee pain in children aged 10–17. It is the most common hip
disorder in adolescence.

Juvenile rheumatoid arthritis (choice D) is the most common form of persistent arthritis in children.
Symptoms of JRA are often non-specific initially, and include lethargy, reduced physical activity, and poor
appetite. The first manifestation, particularly in young children, may be limping. Children may also become
quite ill, presenting with flu-like symptoms that persist. The cardinal clinical feature is persistent swelling of
the affected joint(s), which commonly include the knee, ankle, wrist and small joints of the hands and feet.
Swelling may be difficult to detect clinically, especially for joints such as those of the spine, sacroiliac
joints, shoulder, hip and jaw, where imaging techniques such as ultrasound or MRI are very useful. Pain is
an important feature of JRA, but young children may have difficulty in communicating this symptom.
Morning stiffness that improves later in the day is a common feature. Late effects of arthritis include joint
contracture (stiff, bent joint) and joint damage. Children with JRA vary in the degree to which they are
affected by particular symptoms.

Osteosarcoma (choice E) is the eighth most common form of childhood cancer, comprising 2.4% of all
malignancies in pediatric patients, and approximately 20% of all bone cancers.

Many patients first complain of pain that may be worse at night, and may have been occurring for some
time. If the tumor is large, it can appear as a swelling. The affected bone is not as strong as normal bones
and may fracture with minor trauma (a pathological fracture). The tumour may be localised at the end of
the long bone. Most often it affects the upper end of tibia or humerus, or lower end of femur.
Osteosarcoma tends to affect regions around the knee in 60% of cases, 15% around the hip, 10% at the
shoulder, and 8% in the jaw. The tumor is solid, hard, irregular ("fir-tree," "moth-eaten" or "sun-burst"
appearance on X-ray examination) due to the tumor spicules of calcified bone radiating in right angles.
These right angles form what is known as Codman's triangle. Surrounding tissues are infiltrated.

An 80 year old woman fell down the steps leading to the entrance of her house and sustained a right hip
fracture. Following her hip surgery, she was discharged to a nursing home. She has been unable to
mobilize since her hip operation. A month later, she dies suddenly.

Which of the following is most likely to be the immediate cause of death found at autopsy?

Cerebrovascular accident
Carcinomatosis
Pulmonary Embolism
Bronchopneumonia
Congestive Heart Failure

27
Explanation

The correct answer is Choice D.

Mortality rates following a hip fracture among elderly patients at one month is 9%, this rise to 19% in 3
months, and up to 30% in 1 year. The cause of death, as determined on autopsy from a series of patients
following surgery after a fracture hip include: Bronchopneumonia, 35.1%; Congestive heart failure, 16.2%;
Left ventricular failure, 10.8%; Carcinomatosis, 8.1%; Pulmonary embolism, 8.1%; and Cerebrovascular
accident, 2.7%.

A recently adopted two-year-old black girl, who was born prematurely, is having difficulty walking. On
physical examination, the child is below the fifth percentile for height and weight. Psychomotor and
speech development are delayed. Skeletal inspection is significant for enlargement of the costal cartilages,
thickened wrists bilaterally, and severe bowing (45 degrees) of the tibias bilaterally.

Which one of the following would predispose this child to developing rickets?

Formula fed since birth.


Daily treatment with phenytoin
Excessive consumption of starch.
Childhood obesity.
all of the above are true

Explanation

The correct answer is Choice B

The presentation above is that of a neurotic condition and it is most likely the child has been on
anticonvulsants over time. Anticonculsants like phenobarbitone and phenytoin are known to accelerate
metabolism of calcidiol, severely reducing its availability which may lead to insufficiency and subsequently,
rickets.

Formula fed children are adequately supplied with calcium as most baby formula are fortified with
adequate calcium and vitamin D. A formula fed child is not prone to developing rickets.

Spinach has very high calcium content. Excessive consumption of spinach is less likely to predispose to
rickets, though the oxalates in spinach binds to calcium reducing its availability; about 5% is still available
for absorption.

Childhood obesity is the most unlikely factor that will predispose the child to rickets except there is a
coexisting diseased condition like liver or renal disease.

An adolescent female who is a cheerleader comes to you with a painful bump below her right knee. There
is relief of symptoms with rest or restriction of activities. She denies fever or trauma. On exam, there
is pain and edema of the tibial tubercle. The pain is reproduced by extending the knee against resistance,
stressing the quadriceps, and squatting with the knee in full flexion. Knee joint examination is

28
normal. There is some evidence of mild quadriceps atrophy.

Which of the following is most likely diagnosis?

Legg-Calve-Perthes disease
Osteoid osteoma
Osgood schlatter disease
Osteochondritis dissecans
Osteomyelitits of the tibial tubercle

Explanation

The correct answer is choice C

This a condition whereby there is repetitive tensile stress on insertion of patellar tendon over the tibial
tuberosity. This causes minor avulsion at the site and subsequent inflammatory reaction. The condition is
most common in adolescent athletes, especially jumping sports. Cheerleader is usually involved in jumping
activity.

Legg- Calve Perthes (choice A) disease is a self limited Avascular necrosis of femoral head that presents at
4-10 years of age.

Osteoid Osteoma (choice B) is benign tumor that can produce a disabling back pain.

Osteochondritis dissecans (choice D) is a condition in which a portion of subchondral bone undergoes


avascular necrosis. The cause is usually unknown, but it is probably traumatic.

Osteomyelititis of the tibial tubercle (choice E) is an infection of the bone and is often preceded by signs of
systemic disease and/or general sepsis for several days. The female in this case denies fever and trauma.

A 20 year old hockey player presents to the ER with a painful shoulder. X-ray radiographs of the shoulder
is taken. He states it came out of joint when he was body checked on the ice and his coach pulled it back
into place. The pain has subsided but he still complains of numbness of the arm and inability to abduct
the arm from its current position.

Assuming his shoulder was dislocated, what is the most likely direction of dislocation?

29
superior
anterior
posterior
inferior
medial

Click on image to Zoom


Explanation

The correct answer is choice B

A dislocated shoulder occurs when the humerus separates from the scapula at the glenohumeral joint. The
shoulder joint has the greatest range of motion of any joint in the body and as a result is particularly
susceptible to dislocation and subluxation. Approximately half of major joint dislocations seen in
emergency departments are of the shoulder. Partial dislocation of the shoulder is referred to as
subluxation.

Over 95% of shoulder dislocation cases are anterior. Most anterior dislocations are sub-coracoid. Sub-
glenoid; subclavicular; and, very rarely, intrathoracic or retroperitoneal dislocations may occur. It can
result in damage to the axillary artery.

Symptoms of shoulder dislocation include: significant pain, which can sometimes be felt past the shoulder,
along the arm; inability to move the arm from its current position, particularly in positions with the arm
reaching away from the body and with the top of the arm twisted toward the back; numbness of the arm;
no bone in the side of the shoulder showing shoulder has become dislocated.

Prompt professional medical treatment should be sought for any suspected dislocation injury. Usually, a
dislocated shoulder is kept in its current position by use of a splint or sling. Emergent treatment is focused
on returning the shoulder to its normal position via processes known as reduction. Normally, closed
reduction, in which several methods are used to manipulate the bone and joint from the outside, is used. A
variety of techniques exist, but some are preferred due to fewer complications or easier execution. In
cases where closed reduction is not successful, surgical open reduction may be needed. Following
reduction, xrays is often used to ensure that the reduction was successful and there are no fractures. The
arm should be kept in a sling or immobilizer for several days, preferably until orthopedic consultation.

superior (choice A) dislocation of the shoulder joint is rare to nonexistent.

posterior (choice C) dislocation are occasionally due to electrocution or seizure and may be caused by
strength imbalance of the rotator cuff muscles. Posterior dislocations often go unnoticed, especially in an
elderly patient and in the unconscious trauma patient.

inferior (choice D) dislocation is the least likely form, occurring in less than 1% of all shoulder dislocation
cases. This condition is also called luxatio erecta because the arm appears to be permanently held upward
or behind the head. It is caused by a hyper abduction of the arm that forces the humeral head against the
acromion. Inferior dislocations have a high complication rate as many vascular, neurological, tendon, and

30
ligament injuries are likely to occur from this kind of dislocation.

medial (choice E) dislocation of the shoulder joint is rare to nonexistent.

A 78 year-old woman has experienced several episodes in which she lost her balance during the last 5
months. She also has noticed tinnitus in the left ear, some facial numbness, changes in the taste of
several foods, and dry eyes though she tears at random times throughout the day. On physical
examination she is found to have a marked decrease in hearing on the left, a Rinne test indicating air
conduction better than bone conduction, some sensory defect associated with cranial nerve VII, and a
positive Romberg test. A brain MRI scan reveales a solitary, discrete, 4 cm mass located in the region of
the left cerebellopontine angle. She exhibits no deficits in memory or other mental functions.

The most likely diagnosis is which of the following:

craniopharyngioma
glioblastoma multiforme
acoustic neuroma
Arnold-Chiari malformation
ependymoma

Explanation

The correct answer is choice C.

The presentation and imaging suggests a diagnosis of acoustic neuroma, a benign tumour that develops
from Schwann cells of the cranial nerve VIII in the cerebellopontine angle. Although it also is known as
vestibular schwannoma, acoustic neuroma can involve either the cochlear or vestibular branch of the
nerve. Thus, it may present with symptoms related either to sensorineural hearing loss, or balance, or
both. In this case, the tumour is causing hearing deficits and tinnitus (ringing in the ear) as well as
vestibular effects.

A Romberg test, which involves balance and coordination with eyes closed, is part of a standard
neurological examination. Positive Romberg means that patient cannot maintain balance with eyes closed
and further testing may include electronystagmography (ENG) to explore any vestibular deficits. However,
if a positive Romberg test is the only abnormality on physical examination related to balance, it indicates
good cerebellar function.

Arnold-Chiari malformation (choice D) is herniation of part of the cerebellum through the foramen
magnum of the skull. This can be either congenital or acquired and can be asymptomatic for years, then
present with symptoms. If this were connected with the balance deficit, however, the patient would have
trouble with balance and coordination with eyes open as well as closed. Furthermore, the identification of
a mass is reason enough to eliminate this choice.

Craniopharyngioma (choice A) is a pituitary tumour that occurs mostly in the pediatric population.
Ependymoma (choice E) develops from ependymal cells and accounts for approximately 30 percent of
primary CNS tumors in patients under 3 years of age.

Glioblastoma multiforme (choice B) is an aggressive tumour that often involves memory loss and
deterioration various other mental functions. In MRI, they appear as an irregularly-shaped, ring-
enhancing lesions. It has a poor prognosis as it is extremely difficult to resect entirely with surgery, to
eliminate with radiation, and because of the blood-brain barrier very few chemotherapeutic agents can be

31
delivered to it. Fortunately, this woman has no symptoms, signs, or findings associated with it.

Acoustic neuroma is a fairly common condition. With the increased use of MRI and high resolution CT
scanning in recent years, many small, asymptomatic acoustic neuromas has been elucidated as incidental
findings. In such cases, only regular monitoring may be required. In the case of this patient who has
symptoms, surgical removal or destruction of the tumour with radiation are the best treatment options. It
is a slow-growing tumour and the prognosis is very good.

A 14-year-old girl is noted to have an odd, twisted appearance to her back while she is out swimming with her friends.
She is tall and thin. A radiograph reveals an abnormal lateral bowing of the spine, with 20 degrees of lateral curvature in
the mid-thoracic region.

Which of the following is most likely to produce these findings?

Asymmetric cartilage growth of vertebral body end


plates
Multiple osteochondromas of the vertebral bodies
Vitamin D deficiency with rickets
A disorder of procollagen synthesis with multiple
compressed fractures
Trauma related bone abnormalities

Click on image to Zoom

Explanation

The correct answer is Choice A

Scoliosis is a deformity resulting from lateral curvature in the frontal plane of the spine. Structural
scoliosis results from disproportionate growth and rotation of the vertebral body causing a prominence
in the back from rib asymmetry.

Causes of scoliosis include the following:

 Idiopathic; most cases are of unknown cause


 Congenital; e.g spina bifida, hemivertebra, VACTERL (vetebral, anorectal, cardiac, tracheo-oesophageal,
renal and radial limb anomalies)
 Secondary; e.g connective tissue disorders (such as Marfan's syndrome), neuromuscular imbalance
(resulting from poliomyelitis, cerebral palsy, muscular dystrophy) or postural from leg length discrepancy

32
85% of cases are late onset Idiopathic scoliosis which affect mostly girls 10 - 14 years of age during
pubertal growth spurt.

Diagnosis is made from physical examination of the back with the patient standing erect and then bent
forward. X-rays can be used to measure the severity of spinal curvature.

A 31-year-old male recreational basketball player suffered an inversion injury to his ankle. When initially
seen shortly after the injury, he was unable to fully bear weight on the affected leg and had point
tenderness over the tip of his lateral malleolus. Swelling was noted over the lateral aspect of his ankle.
Radiographs obtained at the time of injury showed no fracture or widening of the joint space. The ankle
was immobilized, ice was applied, and he was told to use crutches and come back in 3 days. At this
follow-up visit, examination reveals swelling and ecchymosis over the lateral ankle. The anterior drawer
sign is weakly positive when compared with the uninjured ankle. He is now able to partially bear weight
on the injured ankle and has discarded his crutches.

What is the next appropriate step in management?

Apply heat to the ankle


Immobilize the ankle in a walking cast for 6 weeks
Begin range-of-motion and strengthening ankle exercises
Repeat ankle radiographs
Steroid injections

Explanation

The correct answer is choice C

A sprained ankle, also known as an ankle sprain or ankle ligament injury, is a common medical
condition where one or more of the ligaments of the ankle is torn or partially torn. The anterior
talofibular ligament is one of the most commonly involved ligaments. Sprains to the lateral aspect of the
ankle account for 85% of ankle sprains.

Sprains happen when the foot is rolled or turned beyond motions that are considered normal for the
ankle. When an ankle is placed on uneven surface or experiences a large force when landing the
ligaments can be stretched into an abnormal position. The ligaments of the ankle hold the ankle bones
and joint in position. They protect the ankle joint from abnormal movements -especially twisting,
turning, and rolling of the foot.

Ankle sprains are classified as grade 1, 2, and 3. Some of the most common causes of ankle injuries
are: lack of conditioning, lack of warming up and stretching properly, previous history of an ankle
sprain, inadequate shoes, and uneven ground. Depending on the amount of damage or the number of
ligaments that are damaged, each sprain is classified from mild to severe. The amount of force that is
placed on the ankle helps classify which grade of sprain is suffered. A mild sprain or one that causes
slight stretching with minimal damage to the fibers in the ligament is considered a Grade 1 sprain.
When there is some tearing of the ligament and the ankle joint moves in abnormal ways, it is noted to
be a Grade 2 sprain. The final classification is Grade 3 sprain and includes severe injuries. These are
ones where complete tears of a ligament and the presence of instability is experienced. Usually bruising
will occur around the ankle but this can be prevented by putting ice on it and not putting it into hot
water.

It is important to either rule out a fracture clinically or radiologically. A sudden movement or twist often
when the foot rolls in can overstretch the supporting ligaments, causing ligament tears and bleeding

33
around the joint. This is known as a sprain. This type of injury occurs most frequently in activities that
require running, skipping, jumping and change of direction (such as basketball, netball, football/soccer).
Some people are particularly prone to recurring ankle sprains.

For most ankle sprains, rest, ice, compression, and elevation (RICE) is often recommended.

In uncomplicated lateral ankle sprains, swelling of the soft tissue can be prevented with compression
around both malleoli, elevation of the injured ankle higher than the heart, and pain-free exercises.
Braces and crutches can be used to help alleviate the pain. Exercise immediately after a sprain however
may improve function and recovery.

Apply heat to the ankle (choice A) is not indicated for ankle sprains. For most ankle sprains, rest, ice,
compression, and elevation (RICE) is often recommended.

Immobilize the ankle in a walking cast for 6 weeks (choice B) is not indicated for ankle sprains in the
absence of fractures. Immobilization is not recommended even in severe grade 3 sprains.

Repeat ankle radiographs (choice D) is unnecessary for ankle sprains once a fracture has been ruled out
on initial radiographs. Serial imaging is not recommended for ankle sprains and can be followed
clinically.

Steroid injections (choice E) is not indicated as a treatment modality for ankle sprains. RICE (rest,
ice,compression and elevation) and NSAIDs for pain is sufficient.

A 40 year old female patient with inflammatory arthritis develops an acutely swollen and painful elbow
2 days following an intra-articular steroid injection. On physical examination, the elbow is warm to
touch and has pain with any range of motion.

What is the most appropriate initial course of action?

Joint aspiration for culture


Repeat steroid injection
Send serum urate and start colchicine
Increase oral steroid and methotrexate
Immobilise the joint

Explanation

The correct answer is choice A

Septic arthritis is the purulent invasion of a joint by an infectious agent which produces arthritis. It is
worth noting that people with artificial joints are more at risk than the general population but have
slightly different symptoms, are infected with different organisms and require different treatment. Septic
arthritis is considered a medical emergency. If untreated, it may destroy the joint in a period of days. The
infection may also spread to other parts of the body.

The usual etiology of septic arthritis is bacterial, but viral, mycobacterial, and fungal arthritis occur
occasionally. A broader term is "infectious arthritis", which describes arthritis caused by any infectious
organism. Septic/suppurative arthritis and "bacterial arthritis" are sometimes considered equivalent, but
there are exceptions. For example, Borrelia burgdorferi can cause infectious arthritis, but is not associated
with suppurative arthritis. Septic arthritis should be considered whenever one is assessing a patient with

34
joint pain. Usually only one joint is affected (monoarthritis) however in seeding arthritis, several joints
can be affected simultaneously; this is especially the case when the infection is caused by staphylococcus
or gonococcus bacteria.

The diagnosis of septic arthritic can be difficult as no test is able to completely rule out the possibility. A
number of factors should increase one's suspicion of the presence of an infection. In children these are:
fever > 38.5 C, non-weight-bearing, serum WBCs > 12 x 10^9, ESR > 40 mm/hr, CRP > 20 mg/dL, a
previous visit for the same. Diagnosis is by aspiration (giving a turbid, non-viscous fluid), Gram stain and
culture of fluid from the joint, as well as tell-tale signs in laboratory testing (such as a highly elevated
neutrophils (approx. 90%), ESR or CRP). The ESR and CRP are almost always raised on admission, CRP
being faster in diagnostics. The Gram stain can rule in the diagnosis of septic arthritis however cannot
exclude it.

Therapy is usually with intravenous antibiotics, analgesia and washout/aspiration of the joint to dryness.
In an infection of a prosthetic joint, a biofilm is often created on the surface of the prosthesis which is
resistant to antibiotics. Surgical debridement or arthrotomy is usually indicated in these cases. A
replacement prosthesis is usually not inserted at the time of removal to allow antibiotics to clear infection
of the region.

Patients in whom surgery is contraindicated may trial long-term antibiotic therapy.

Repeat steroid injection (choice B) as repeating a steroid injection in a setting of an infection is


contraindicated.

Send serum urate and start colchicine (choice C) is incorrect as it is assuming the patient may have gouty
arthritis, which is not supported in the patient’s history or physical exam.

Increase oral steroid and methotrexate (choice D) is incorrect as it does not address the primary concern
of ruling out an infection.

Immobilise the joint (choice E) is the next appropriate step in management after the joint is aspirated and
appropriate antibiotics are started.

A 67 year old female presents with difficulty walking and slurred speech. Examination reveals gait
ataxia, nystagmus and dysarthria. She was recently diagnosed with stage IV ovarian cancer and is about
to receive chemotherapy. Her brain CT scan and MRI were both normal.

What is the most likely diagnosis?

Conversion disorder
Malignancy associated with Guillain Barre syndrome
AIDS dementia complex (ADC)
Paraneoplastic cerebellar degeneration
Vitamin B12 deficiency associated with malnutrition

Explanation

The correct answer is Choice D.

Paraneoplastic cerebellar degeneration is a phenomenon associated with cancer, especially lung, ovarian

35
and breast. It is thought to be caused by an autoimmune reaction—an anti-neuronal antibody known as
anti-Yo—that is targeted against components of the central nervous system, specifically Purkinje cells and
large brain stem nuclei. One characteristic is cerebellar ataxia that is caused by Purkinji cell destruction in
the cerebellum; it is manifested by dysarthria, gait ataxia and nystagmus. Radiologic imaging may reveal
cerebellar atrophy. Many antibodies have been associated with this syndrome including anti-Yo, anti-Hu
and anti-Ri. A large number of patients who display anti-Yo anti-bodies typically have breast or ovarian
cancer.

Conversion disorder (choice A) is considered a psychiatric disorder where patients present with symptoms
(such as numbness, blindness, paralysis or fits) but where no neurological disorder is found. It is believed
that these problems arise in response to difficulties in the patient's life.

Several cases have associated Guillain Barre syndrome (GBS) with neoplastic diseases (choice
B). Paraneoplastic syndromes occur in 10–20% of patients with lung malignancies and occur most
frequently with small cell lung cancer. It remains unclear whether these are merely coincidental or
represent true paraneoplastic phenomena. The clinical features of GBS associated with oncological cases
do not appear to differ from those classically triggered by a preceding bacterial or viral infection. GBS is
an autoimmune disorder affecting the peripheral nervous system and is characterized by symmetrical
weakness that affect the lower limbs first which rapidly progresses in an ascending fashion. The weakness
progresses upward into the arms and face, usually over a period of a few hours to days. Most patients
require hospitalization and about 30% require a ventilator due to the paralysis of the diaphragm.

AIDS dementia complex (choice C) is a metabolic encephalopathy induced by HIV infection and fueled by
activation of brain macrophages and microglia. These cells are infected with HIV and secrete neurotoxins.
ADC typically occurs after years of HIV infection.

The essential features of ADC are:

 Disabling cognitive impairment (mental slowness, trouble with memory, poor concentration)
 Motor dysfunction (a loss of fine motor control leading to clumsiness, poor balance, tremors)
 Speech problems
 Behavioral change (apathy, lethargy, diminished emotional responses and spontaneity)
 Dementia - occurs when neurocognitive impairment in the patient is severe enough to interfere
with daily function.

Vitamin B12 deficiency (choice E) is commonly asymptomatic. Early symptoms include: fatigue, mental
slowness, poor concentration, memory impairment, irritability and depression. Sleep disturbances may
occur because B12 may be involved in the regulation of sleep. It can cause megaloblastic anemia (enlarged
blood corpuscles with characteristic neutrophil changes). Neurological signs of B 12 deficiency include
sensory disturbances (numbness, altered proprioception, impaired sense of smell), anorexia and ataxia.
In serious cases it can cause severe and irreversible damage to the nervous system, including subacute
degeneration of the spinal cord.

A 41 year-old woman presents at your clinic with vague headaches and what she describes as "patchy
vision". She is Canadian of northern European heritage and a non-smoker. She is the mother of two
healthy children, each born after a normal pregnancy. However, her family history is remarkable for a
sister who was diagnosed recently with multiple sclerosis and an aunt with macular degeneration. A
visual field test reveals bitemporal hemianopia.

Most likely, the diagnosis will be:

36
multiple sclerosis
pituitary adenoma
bilateral temporal arteritis
craniopharyngioma
macular degeneration

Explanation

The correct answer is choice B.

Bitemporal hemianopia (also called bitemporal hemianopsia) means that the patient is seeing only the
inner half of the visual field on each side, thus is blind to the outer, or temporal, fields. This results from
a lesion at the optic chiasm which most often is a pituitary tumour. Derived from adenohypophyseal
cells, tumours of the pituitary gland account for 15 percent of primary intracranial neoplasms. With an
incidence of 1 out of 10,000 annually, adenomas account for more than 90 percent of pituitary tumours,
which makes it the most likely etiology of this patient's symptoms.

Craniopharyngioma (choice D) is the most common pituitary tumour in the pediatric population. Thus, it
is far less likely in this case than pituitary adenoma. While the patient's family history should raise red
flags with respect to multiple sclerosis (MS, choice A) and macular degeneration (choice E), the very
specific visual field deficit here is the reason why the most likely choice has to involve a lesion at the
optic chiasm.

MS is a disease that occurs primarily in people of northern European origin and has a particularly high
incidence in Canada. Having a first degree relative with MS increases one's risk substantially. With such
a history and visual problems described as "patchy vision, it would be appropriate to send this woman
for an MRI, even if the lesion were not tracked down to the optic chiasm. MRI of the brain can reveal,
not only a pituitary tumour but changes resulting from MS as well. Thus, the consultations that will be
required include both neurosurgery and neurology. Ophthalmology also should evaluate this patient.
Macular degeneration presents with deficits in the center of the visual field, which should be
differentiated easily from bitemporal hemianopia, if the visual field testing is very thorough.

Temporal arteritis (choice C) is a vasculopathy resulting from inflammation of temporal arteries.


Typically, it causes headaches and also may cause bilateral vision loss, but not with the pattern that is
presented with this patient. Temporal arteritis can be unilateral or bilateral and typically is seen in
patients older than 50 years of age. It is included as a choice here to distract you with the word
"temporal."

A 25 year-old male is flown to the ER unconscious, after suffering a head injury while riding his motor
cycle in the mountains without wearing a helmet. Just after arriving, he regains consciousness as he is
taken for a CT scan, which is shown in the image. After consulting with the neurosurgeon, who agrees
that immediate surgery is necessary, you are introduced to the patient's girlfriend, who notes that the
patient, who is a law student, had been experiencing vague headaches and occasional dizziness for
several weeks prior to the accident.

You diagnose this patient with which of the following conditions?

37
subarachnoid hemorrhage
epidural hematoma
subdural hematoma
intra-parenchymal hemorrhage
meningioma

Click on image to Zoom

Explanation

The correct answer is choice B.

An epidural hematoma (also called extradural hematoma) is an accumulation of blood between the dura
matter and the bone. It can be acute, subacute, or chronic. In the case of an intracranial epidural
hematoma (as opposed to spinal) usually it is the temporoparietal region of the skull (66 percent of
cases), resulting from a tare in an artery, typically the middle meningeal artery. However, the condition
can result from injuries to a different artery, or to a vein. Generally, it occurs due to trauma and skull
fractures are present in 85-95 percent of cases occuring in adults. As the blood accumulates, the dura is
separated from the bone, causing severe headache, while subjecting the underlying brain tissue to
increased pressure. Because the dura is held very tightly to the bone at the suture lines, typically the
dura is pushed into an arc, spanning suture line to suture line, thus forming a lens shape, seen easily in
CT imaging (because adherence of the dural to the calvaria increases with age, intracranial epidural
hematoma is rare in individuals over 60 years of age, though it also is rare before age 2). If the
hematoma is big, is a shifting of the brain toward the other side of the skull is notable. Frequently,
patients suffering from an epidural hematoma wake up for a period, known as a lucid interval, following
the initial unconsciousness.

If the patient does experience a lucid interval, prognosis is much better if treatment is initiated before
he or she slips back into unconsciousness. In fact, for patients who are awake prior to surgery, a zero
percent mortality rate has been calculated, compared to 9 percent for obtunded patients and 20 percent
for comatose patients. In any case, intracranial epidural hematoma is considered to be the most serious
complication of head injury, and an extreme emergency. Present in 2 percent of head injuries, though
5-15 percent of fatal head injuries, the condition requires immediate diagnosis and surgical intervention.

Individuals suffering from a small epidural hematoma may require only conservative medical treatment,
but with close observation, since sudden, neurological deterioration may occur. Osmotic diuretics and
hyperventilation can be used, keeping the head elevated, in patients with elevated intracranial pressure.
Treatment of coagulapathies is appropriate in cases when epidural hematoma, either intracranial or
spinal, results for this reason rather than from trauma. Generally, however, definitive treatment centers
around surgery. Either craniotomy or laminectomy is conducted, allowing for evacuation of the
hematoma and coagulation of the bleeding sites. The dura is evaluated and can be tented to the bone.
If needed, epidural drains can be used for up to 24 hours. In certain cases, burr holes, negative
pressure drainage, and other less invasive techniques can be employed. Other techniques include closed
suction drain for thrombolytic evacuation, and endovascular embolization during the acute stage, to
minimize bleeding.

In a subarachnoid hemorrhage (choice A), the bleeding occurs in the subarachnoid space, which is to
say between the arachnoid membrane and the pia mater that surrounds the brain. The condition can
develop from head injury, or spontaneously, generally as a result of a ruptured cerebral aneurysm.

In subdural hematoma (choice C),the blood accumulates, just under the dural, between the dura and
arachnoid mater, generally from injury to the bridging veins. As opposed to an epidural hematoma

38
which forms a lens shape from suture line to suture line, an subdural hematoma continues spreading
out, and thus is seen as a crescent shape. Intra-parenchymal hemorrhage (choice D) is bleeding in the
brain tissue itself, resulting from a variety of conditions, including hemorrhagic stroke due to
hypertension.

Meningioma (choice E) refers to a category of tumours that arise contiguously to the meninges, and
thus are found at the surface of the brain. The etiology has not been elucidated, though trauma and
viruses have been suggested as possible causes. Incidence increases with age and symptoms can vary
widely, depending on the locations.

A previously healthy 31-year-old woman experiences a severe headache and loses consciousness within
an hour. She is rushed to the emergency department via ambulance. An emergency head CT scan
reveals extensive subarachnoid hemorrhage at the base of the brain. She is afebrile. A lumbar puncture
yields cerebrospinal fluid with many red blood cells, but no white blood cells. The CSF protein is slightly
increased, but the glucose is normal.

Which of the following is the most likely diagnosis?

Acute bacterial meningitis


Progressive multifocal leukoencephalitis

Traumatic head injury


Ruptured berry aneurism
Parkinson disease

Explanation

The correct answer is choice D.

Berry aneurysms are congenital in nature and result in a weakness of the blood vessel wall. Weak or
thinned parts of the cerebral vasculature are vulnerable to the increased hydrostatic pressure caused by
hypertension, and will bulge out. Some parts of the brain vasculature are inherently weak: particularly
areas along the Circle of Willis, where small communicating vessels link the main cerebral vessels. The
risk factors for developing berry aneurysms include any condition that causes hypertension, or any
condition that causes weakening of blood vessel walls.

Signs of an aneurysm that has not ruptured include (but are not limited to): headaches (severe),
diplopia, loss of vision, eye and neck pain. Signs of an aneurysm that has ruptured include: sudden
severe headache, confusion and/or stupor, seizures, sudden mood swings, dysarthria, ptosis, and
ataxia. Most cerebral aneurysms are unobserved until they have already ruptured, though if suspected,
diagnosed using angiography, MRI, CT scans, and cerebrospinal fluid analysis. Micro-vascular clipping
and occlusion are effective surgical procedures for treating a berry aneurysm.

The woman cannot have acute bacterial meningitis (choice A) because the LP did not reveal WBC's, and
she was afebrile.

Progressive multifocal leukoencephalitis (choice B) is a progressive illness in immunocompromised


patient.

39
Traumatic head injury (choice C) arises from an external force on a patient’s head.

Parkinson disease (choice E) is a slowly progressive degenerative disease of older persons and is
marked by a tremor at rest.

A 6 year-old boy presents at your clinic on account of pain in the right knee. His parents explain that he
limps during ambulation, and the pain seems to be focussed in the medial aspect of the knee. The child's
history is negative for any injury. On physical examination, you note decreased internal rotation of the
hip of the affected leg, as compared to the contralateral leg. A radiograph of the child's pelvis is shown.

Most likely, you will diagnosis the child which which of the following?

Köhler disease
Osgood-Schlatter disease
Legg-Calvé-Perthes
developmental dysplasia of the hip
slipped capital femoral epiphysis

Click on image to Zoom

Explanation

The correct answer is choice C.

Presenting most commonly in children between the ages of 4-8, Legg-Calvé-Perthes disease is a
degenerative condition of the hip joint, resulting from idiopathic avascular necrosis of the the epiphysis of
the head of the femur. In radiographs, the femoral head appears flattened, as it does in this image. It is a
fairly rare disease, with approximately 5.5 cases occurring per 100,000 children, annually. Typically,
patients have a history of pain in association with weight-bearing activity, including walking. Clinically, the
most significant finding is decreased range of motion of the affected joint. Often, pain is referred to the
knee, while muscles of the thigh and leg may atrophy, exacerbating the effects on movement.
Additionally, osteoarthritis may develop, as a result of the rubbing between the degenerating femoral
head and the acetabulum.

Treatment is based on the approach of reducing strain on the joint while increasing activity, as a
countermeasure to muscle atrophy. Orthotic devices and traction may be employed to remove pressure
by keeping the femur abducted and internally rotated, thus holding the femoral head well inside the
rounded part of the acetabulum. One such orthotic device is the Scottish Rite brace, which maintains the
abduction in the hip joint, though allowing the knee to move freely. Surgical options are available as well,
specifically femoral osteotomy (the drawback of which is shortening of the leg, leading to continued
limping) and innominate osteotomy. These procedures can help the femoral head to reossify and the
condition can end up being self-limiting and resolve as the child grows, if osteoarthritis and muscle can be
prevented. Exercise may be prescribed, the choice of which is extremely important. High impact activities
such as running and tumbling are to be avoided, while swimming and bicycling are to be encouraged,
since these activities work the muscles of the lower extremity without putting weight on the hip joint.

Although Köhler disease (choice A) involves avascular necrosis, this occurs in the navicular bone in the
foot, while Osgood-Schlatter disease (choice B) is a condition of pain and inflammation of the tibial
tubercle apophysis, with knee pain localized over the tubercle. It is most common in adolescent athletes,
especially those involved in jumping sports. Once known as "congenital dislocation of hip", developmental

40
dysplasia of the hip (choice D) can be elucidated during physical examination by way of the Barlow and
Ortoloni maneuvers. The condition involves both dysplasia and dislocation. However, this radiograph
reveals no such dislocation. In the case of a slipped capital femoral epiphysis (choice E), the radiograph
would show displacement of the femoral epiphysis. Additionally, this condition presents most frequently
between the ages of 10-16.

A 26 year-old Caucasian woman presents at your clinic, complaining of headaches and occasional
diplopia. She is a nonsmoker with no history of any major illness or surgery, and takes not medication.
Her family history includes a father with hyperlipidemia and coronary atherosclerosis and a mother with
mild knee problems due to obesity. On physical examination, you note that the patient is slightly
overweight, with a BMI of 26. Reflexes and Romberg test are normal. You note bilateral palsy of cranial
nerve VI, but no other focal signs. Additionally, funduscopic examination reveals a finding shown in the
image, but a CT and MRI of the brain, visual acuity, visual field, and color vision testing, and testing of
cerebrospinal fluid all return with no abnormal results.

The preferred term for the condition that this patient has, most likely, is:

pseudotumor cerebri
idiopathic intracranial hypertension
pituitary microadenoma
benign intracranial hypertension
pseudopapilledema

Click on image to Zoom

Explanation

The correct answer is choice B.

Headaches, together with bilateral or unilateral cranial nerve VI (abducens nerve) palsy and what appears
to be papilloedema, suggests elevated intracranial pressure. In such cases, imaging of the head, either CT
or MRI, is warranted prior to a lumbar puncture. When all other causes can be ruled out, a diagnosis of
idiopathic intracranial hypertension is in order. While this is considered a benign condition, patients
generally are not asymptomatic, but often present with headaches, and sometimes even diplopia, due to
the abducens nerve palsy. Recall that the abducens nerve gets is name, because it controls the lateral
rectus muscle, which abducts the eye, and thus if compromised the affected individual will see double
when gazing in certain directions. Although pseudotumor cerebri (choice A) and benign intracranial
hypertension (choice D) both are synonymous with idiopathic intracranial hypertension, the latter is the
preferred term in formal medical language.

Pituitary adenomas are categorized as microadenomas (choice C) if they are less than 10 mm in size, and
otherwise are termed macroadenomas. Very often, small microadenomas are asymptomatic, and studies
have revealed that most are stable and do not grow, especially those smaller than 4mm. Such studies
were made possible by improving availability and increasing resolution of brain imaging modalities such
as CT and MRI, which also have led to a rise in incidental findings of small pituitary tumours, sometimes
termed "incidentalomas". Based on these findings, it is estimated that as much as 10 percent of the
general population has such asymptomatic tumors. Thus, whether large enough to cause this woman's

41
symptoms, or extremely small so as to be stable and asymptomatic, any pituitary microadenoma would
have been detected easily in the brain imaging.

Pseudopapilledema (choice E) is a phenomenon in which the optic disc appears to swell, though it is
secondary to some underlying, usually benign, condition, rather than to elevated intracranial pressure.
While the latter tends to manifest with bilateral papilledema, pseudopapilledema can be either bilateral or
unilateral. If bilateral, often, it can take an ophthalmologist to distinguish pseudopapilledema from real
papilloedema, based on subtle features. In this case, however, since the patient is complaining of
headaches, it seems more likely that the intracranial pressure is indeed elevated.

A 23-year-old woman was diagnosed with systemic lupus erythematosus (SLE) at age 13 years when she
presented with polyarticular joint swelling. On a regimen of methotrexate and hydroxychloroquine, her
joint pain and swelling have been relatively well controlled. The patient’s fingers exhibit both boutonnière
and swan-neck deformities, but she is able to make a normal closed fist.

What will radiographs of the patient’s hands likely reveal?

Bone fragmentation
Carpal fusion
Erosions in the proximal joints
Marked subchondral cysts
Normal joints

Click on image to Zoom

Explanation

The correct answer is choice E

Arthritis is a group of conditions involving damage to the joints of the body.

The most common form, osteoarthritis (degenerative joint disease) is a result of trauma to the joint,
infection of the joint, or age. Other arthritis forms are rheumatoid arthritis, psoriatic arthritis, and related
autoimmune diseases in which the body attacks itself. Septic arthritis is caused by joint infection.

The major complaint by individuals who have arthritis is pain. Pain is often a constant and daily feature of
the disease. The pain may be localized to the back, neck, hip, knee or feet. The pain from arthritis occurs
due to inflammation that occurs around the joint, damage to the joint from disease, daily wear and tear of
joint, muscles strains caused by forceful movements against stiff, painful joints and fatigue. The most
important factor in treatment is to understand the disorder and find ways to overcome the obstacles which
prevent physical exercise.

Systemic lupus erythematosus, often abbreviated to SLE or lupus, is a chronic systemic autoimmune
disease (or autoimmuneconnective tissue disease) that can affect any part of the body. As occurs in other
autoimmune diseases, the immune system attacks the body’s cells and tissue, resulting in inflammation
and tissue damage. SLE most often harms the heart, joints, skin, lungs, blood vessels, liver, kidneys, and
nervous system. The course of the disease is unpredictable, with periods of illness (called flares)
alternating with remissions. The disease occurs nine times more often in women than in men, especially

42
between the ages of 15 and 50, and is more common in those of non-European descent

SLE is one of several diseases known as "the great imitators" because it often mimics or is mistaken for
other illnesses. Common initial and chronic complaints include fever, malaise, joint pains, myalgias,
fatigue, and temporary loss of cognitive abilities. Because they are so often seen with other diseases, these
signs and symptoms are not part of the diagnostic criteria for SLE. When occurring in conjunction with
other signs and symptoms (see below), however, they are considered suggestive.

The most commonly sought medical attention is for joint pain, with the small joints of the hand and wrist
usually affected, although all joints are at risk. Unlike rheumatoid arthritis, lupus arthritis is less disabling
and usually does not cause severe destruction of the joints and patients will often have normal
radiographic findings. Fewer than ten percent of people with lupus arthritis will develop deformities of the
hands and feet.

Bone fragmentation (choice A) is a feature of osteonecrosis and characterized by the presence of bone
destruction, joint destruction, and subluxation or dislocation and fragmentation. It is a common finding in
rheumatoid arthritis but not in lupus arthritis.

Carpal fusion (choice B) is the abnormal fusion of two or more wrist bones to make a single bone. Fusion is
often asymptomatic but may result in impaired wrist movement. Acquired carpal fusion is most commonly
seen in juvenile rheumatoid arthritis.

Erosions in the proximal joints (choice C) is a feature of rhuematoid arthritis. Erosions of the hands and
wrists involve all five MCP, PIP joints and interphalangeal joint of the thumb and the radiocarpal, inferior
radioulnar, pisiform-triquetral joints. Early signs of erosions show fusiform periarticular soft-tissue swelling,
resulting from effusion. The site of first erosion is classically seen in the base of proximal phalanx of 4th
finger

Marked subchondral cysts (choice D) is a feature of osteoarthritis. Osteoarthritis is caused by the


breakdown of cartilage in one or more joints. Osteoarthritis progresses in stages:

 joint space begins to narrow and osteophytes form


 joint space disappears as cartilage wears away and bone rubs on bone in the joint
 subchondral cysts appear

 bone tries to repair itself and there is bone remodeling

A 45 year-old white woman is brought to the emergency department with mental status changes
characterized by increasing confusion, somnolence, and lethargy. She was noted to drink large amounts of
water on occasion recently and was urinating frequently. She has a history of schizoaffective disorder and
mild mental retardation. Her medications include divalproex sodium, lithium, risperidone, benztropine
mesylate, quetiapine, and clonazepam . She is awake and alert. Physical examination reveals mild
tachycardia, with a heart rate of 105 beats/min. Her mouth is dry. Laboratory test results show a serum
sodium level of 147 mEq/L (normal 137-145) and a fasting blood glucose of 82 mg/dL (normal 65-110).
Serum osmolality is 304 milli-osmoles/kg (normal 275-299).

Which of the following tests would be most useful for the diagnosis?

water-deprivation test
measuring hemoglobin (Hb) A1c
measuring plasma vasopressin level
dexamethasone suppression test
administering desmopressin

43
Explanation

The correct answer is choice A.

Confusion, somnolence, and lethargy can arise in association with various conditions, and polydipsia is one
of them. Polydipsia is the consumption of large amounts of water. It can be primary, or psychogenic,
polydipsia such that drinking the water causes the frequent urination, or it can be an effect of diabetes
insipidus (DI). The latter causes hypernatremia which we see in this patient, though so it is only slightly
beyond the normal range, as is the corresponding increase in serum osmolarity. DI can be central or
nephrogenic, but the relation to polydipsia is the same; excessive excretion of urine, polyuria, makes the
patient thirsty. Thus, reducing the intake of fluid in DI has no effect on the polyuria, whereas reducing
intake does reduce urinary output in psychogenic polydipsia.

Psychogenic polydipsia may occur in association with psychotic conditions such as schizophrenia and
schizoaffective disorder. It also may be the result of the anticholinergic effects of the patient's medication,
specifically dry mouth, although to drink to such a large extent this etiology is less likely. However, lithium
is a notorious as a cause of nephrogenic diabetes insipidus, while the antipsychotic drug risperidone
actually is used to treat psychogenic polydipsia.

If the water-deprivation test (also known as a fluid-deprivation test) does not show a reduction in urinary
output, the next step is to distinguish between central and nephrogenic DI. This is achieved by
administering desmopressin (choice E), which is 1-desamino-8-D-arginine vasopressin, a modified form of
the body's arginine vasopressin (choice C). If urinary output still is not affected, it suggests that the cause
is central, specifically a deficiency of ADH from the pituitary gland. Otherwise, the DI is nephrogenic.

HbA1c (choice B) is an assay of glycated hemoglobin, used in the workup for diabetes mellitus, but this
patient has a normal glucose level and nothing is mentioned suggesting diabetes mellitus. A
dexamethasone suppression test (choice D) is used to determine whether Cushing syndrome is present,
and if so whether the cause is an ACTH-producing tumour in the pituitary, or an ectopic ACTH source.

A 40-year-old recreational tennis player injures his left ankle. He limps into your office 3 days later. He
has been applying ice and wearing an elastic wrap. His medical history is suggestive of an inversion ankle
injury. He reports that he continued to play for 15 minutes after the injury but stopped because of
increasing discomfort. Physical examination shows the lateral aspect of the ankle is swollen, with
ecchymosis below the lateral malleolus. Palpation reveals tenderness anterior to the lateral malleolus but
no tenderness over the base of the fifth metatarsal. Anterior drawer and talar tilt tests are negative. The
patient says he wants to play in a tournament in 2 months.

Which of the following approaches is the best treatment and is consistent with the Ottawa ankle rules?

44
X-ray the ankle. If no fracture, then use elastic wrap and crutches and no weight-bearing for four
weeks. If fracture, then immobilize and no weight-bearing for six weeks.

X-ray the ankle. If no fracture, then use air stirrup (air cast) and weight-bearing as tolerated for 4
weeks, then discontinue air stirrup and begin strengthening exercises. If fracture, then
immobilize and no weight-bearing for six weeks.

Do not X-ray. Use an air stirrup for two weeks with full weight-bearing, maintain passive range of
motion with daily exercises, then begin strengthening exercises three weeks after the injury.

Do not X-ray the ankle. Use an air stirrup and avoid weight-bearing for four weeks, then begin
passive range-of-motion and strengthening exercises.

Do not X-ray the ankle. Physical findings are consistent with a complete tear of the talofibular
ligament, and surgical repair is the only option for an athlete.

Explanation

The correct answer is choice C.

The Ottawa ankle rules help clinicians decide if an X-ray study is worth doing. X-rays are required only if there
is any pain in the malleolar zone and either bone tenderness along the distal 6 cm of the posterior edge of
either malleolus (medial or lateral), or inability to bear weight both immediately and ion clinical evaluation for
four steps. Remember that malleolar pain may be anywhere but the tenderness must be in the posterior
malleolus region, medial or lateral.

There is no tenderness at the posterior aspect of either malleolus. Also, the patient obviously can weight-bear
because he was able to play tennis after the injury. No X-ray is required.

There is evidence of a partial tear of the anterior talofibular ligament (note the location of tenderness and
swelling), but no ankle instability. Surgery is not necessary. A non-weight-bearing status for four weeks will
permit deconditioning. To avoid the formation of adhesions, passive range of motion needs to be preserved
beginning as soon as possible. (This principle is more important for the shoulder than for the ankle, but is
worthwhile nevertheless.

Other considerations regarding the Ottawa ankle rules include litigation issues (liability for a slip-and-fall
injury), impaired sensation so that pain and tenderness may be falsely negative, a previously injured ankle or
foot that may have internal fixation that can work loose or break, and mental-health issues that interfere with
disclosing pain or its absence as you examine the ankle. Do not follow the Ottawa ankle rules blindly.

In general, prescribe large doses of vitamin C after any ligament or tendon injury to make sure that collagen
repair proceeds quickly and with a strong result.

45
A 68 year-old woman presents at your clinic on account persistent headaches. Physical examination is
unrevealing, but CT scan of the head reveals a 2.5 cm spherical mass in the lateral aspect of the cerebral
hemisphere, along the junction of gray and white matter.

Which of the following types of lesion is most likely?

ependymoma
glioblastoma multiforme
metastatic carcinoma
acoustic neuroma
meningioma

Explanation

The correct answer is choice C.

The majority (approximately 70 percent) of intracranial tumours in adults are supratentorial (above the

46
tentorium), while the opposite is true in the case of the pediatric population. Metastases is the most
common type of mass lesion in the brain, followed by astrocytoma, then meningioma, then pituitary
tumours. Metastases often appear in spherical form and located the junction of cortical white and gray
matter.

Ependymoma (choice A) is very uncommon and can develop from ependymal cells throughout the central
nervous system. In children usually it is intracranial, arising in the ventricles, and accounting for
approximately 30% of primary CNS tumors in patients under 3 years of age.

Glioblastoma multiforme (choice B) is an aggressive tumour, a type of astrocytoma, that often involves
memory loss and deterioration various other mental functions. They are not spherical, but appear in MRI
as an irregularly-shaped, ring-enhancing lesion, which can cross the midline as a "butterfly lesion". It has a
poor prognosis as it is extremely difficult to resect entirely with surgery, to eliminate with radiation, and
because of the blood-brain barrier very few chemotherapeutic agents can be delivered to it.

Acoustic neuroma (choice D) is a fairly common, benign tumour that develops from Schwann cells of the
cranial nerve VIII in the cerebellopontine angle. Although it also is known as vestibular schwannoma,
acoustic neuroma can involve either the cochlear or vestibular branch of the nerve. Thus, it may present
with symptoms related either to sensorineural hearing loss, or balance, or both. With the increased use of
MRI and high resolution CT scanning in recent years, many small, asymptomatic acoustic neuromas has
been elucidated as incidental findings. In such cases, only regular monitoring may be required. Even when
symptomatic, however, this is a slow-growing tumour and the prognosis is very good. While meningioma
(choice E) can develop as a spherical lesion, they tend to develop at the surface of the brain, rather than at
the junction of gray and white matter.

A 21 year-old man presents in your clinic complaining of a "problem knee" which he aggravated while
snowboarding two weeks ago. This has happened several times before, but normally the knee feels better
after a few days. This time, it has not improved and when he shows it to you, you note swelling. You ask
the patient to sit on a table with his legs dangling over the edge. You then bend the patient's knee so that
it's flexed at a 90-degree angle. You then grasp the patient's foot and turn it inward and instruct the
patient to extend the affected leg until he feels pain. The patient experiences some pain when the leg
reaches about 30 degrees of flexion. As you rotate the patient's foot back into its normal position, this
alleviates the pain in the patient's knee.

The most likely diagnosis includes which of the following:

meniscus injury
osteochondritis dissecans
chondral fracture
osteoarthritis
synovial chondromatosis

Explanation

The correct answer is choice B.

Osteochondritis dissecans (OCD) is characterized by a focal area of injury to subchondral bone which leads
to necrosis. If the subchondral bone does not heal, sequelae include swelling, softening, partial
detachment, and finally complete separation of a bony fragment. Typically, it presents during the ages of
10-20, but it occurs in other age groups as well, with males being affected with more than double the
incidence as females. Most cases (85 percent) involve the medial femoral condyle (MFC) of the knee, while
the rest involve the lateral femoral condyle, with 70 percent of MFC lesions occurring in the

47
posterolaterally.

OCD is thought to develop in response to either trauma or ischemia and few symptoms and signs are
available to identify it. The Wilson test is performed by having the examiner flex the knee to 90 degrees
while rotating the tibia internally. If pain develops at 30 degrees of flexion and is relieved by external
rotation, this is considered a positive Wilson sign. With this result, the next step is to take plain X-ray film
which should reveal a bony fragment in the medial condyle of the femur. This should be followed up with
MRI which can elucidate any osteonecrosis, which is staged according to appearance in the imaging. Stage
1 means thickening of articular cartilage and low signal changes. Stage 1 means there is a beach of the
articular cartilage and low signal rim behind a fragment, suggesting fibrous attachment. Stage 3 means
that the articular cartilage is breached but with high signal changes behind the fragment as well as
underlying subchondral bone. Stage 4 means an osteochondral fragment is detached or very loose.

The special test performed for OCD is the wilson's test and was explained in the stem. In this test, pain is
worse on knee extension to 30 degrees flexion and pain relieved with external rotation of tibia.

Meniscus Injury (choice A) tends to result from twisting, and often repetitive movements involving the
knee. Provocative maneuvers include, not the Wilson test, but the McMurray, Steinmann test, and Apley
tests. Often, there is a clicking sound, resulting from a piece of meniscus being loose or dislodged.

Chondral fracture (choice C) is likely when persistent patellar tenderness pain is present that has resulted
from direct or indirect injury. In contrast, this patient had difficulty with his knee for quite some time,
characterized by brief periods of aggravation.

Osteoarthritis (choice D) is characterized by osteophytes visible on X-ray. It is a progressive condition that


is quite common, affecting 30 percent of people between the ages of 45-65 and 80 percent of those
beyond age 70. In young people, however, it is fairly rare.

Synovial chondromatosis (choice E) is a very rare condition characterized by the development of cartilage
within the synovial membranes of joints, bursae, or tendon sheaths. This is due to metaplasia of the
subsynovial connective tissue.

A 23 year old male was involved in a snowmobile accident and sustained a closed displaced humeral shaft
fracture. His neurovascular status is normal. You realign the fracture and place him in a spint. Prior to
discharge he states he has numbness over the dorsum of his hand and he is unable to extend his
metacarpophalangeal joints.

Which of the following would you suspect to be the cause?

ulnar nerve was damaged at the time of the fracture


radial nerve was damaged at the time of the
fracture
radial nerve is trapped in the fracture site
ulnar nerve is compressed by hematoma
median nerve is trapped in fracture site

Click on image to Zoom

Explanation

48
The correct answer is choice C

A humerus fracture can be classified by the location of the humerus involved: the upper end, the shaft, or
the lower end. Certain lesions are commonly associated with fractures to specific areas of the humerus. At
the upper end, the surgical neck of the humerus and anatomical neck of humerus can both be involved,
though fractures of the surgical neck are more common. The axillary nerve can be damaged in fractures of
this type. Mid-shaft fractures may damage the radial nerve, which traverses the lateral aspect of the
humerus closely associated with the radial groove. The median nerve is vulnerable to damage in the
supracondylar area, and the ulnar nerve is vulnerable near the medial epicondyle, around which it curves
to enter the forearm.

The radial nerve is a nerve in the human body that supplies the upper limb. It supplies the triceps brachii
muscle of the arm, as well as all 12 muscles in the posterior osteofascial compartment of the forearm, as
well as the associated joints and overlying skin.

It originates from the posterior cord of the brachial plexus with roots from C5, C6, C7, C8 & T1.

The radial nerve and its branches supply the dorsal muscles, such as triceps brachii, the extrinsic extensors
of the wrist and hands, and the cutaneous nerve supply to most of the back of the hand. (The ulnar nerve
cutaneously innervates the back of the little finger and adjacent half of the ring finger). The radial nerve
divides into a deep branch (which becomes the posterior interosseous nerve), and continues as the
superficial branch. The superficial branch of the radial nerve provides sensory innervation to much of the
back of the hand, including the web of skin between the thumb and index finger.

ulnar nerve was damaged at the time of the fracture (choice A) would be consistent with damage to the
medial epicondyle. The medial epicondyle of the humerus is larger and more prominent than the lateral
epicondyle. It gives attachment to the ulnar collateral ligament of the elbow-joint, to the Pronator teres,
and to a common tendon of origin of some of the flexor muscles of the forearm. The ulnar nerve runs in a
groove on the back of this epicondyle.

axillary nerve was damaged at the time of the fracture (choice B) would be consistent with damage to the
upper end of the hummerus such as the surgical neck of the humerus.

ulnar nerve is compressed by hematoma (choice D) would not present with numbness over the dorsum of
the hand or inability to extend metacarpophalangeal joints. The ulnar nerve can be trapped or pinched as it
proceeds from the brachial plexus to the fingertips. One common cause is cubital tunnel syndrome. In this
syndrome, a tunnel on the medial side of the elbow traps the nerve. Pinching of the ulnar nerve often
causes paraesthesiae (tingling) in the fourth and fifth digits. In severe cases, surgery is performed to move
the nerve.

median nerve is trapped in fracture site (choice E) would be consistent with damage in the supracondylar
area. A supracondylar fracture is a fracture of the distal humerus just above the epicondyles. While
relatively rare in adults it is one of the most common fractures to occur in children. Typically, the child
presents with history of a falling on an outstretched hand followed by pain, swelling and inability to move
the affected elbow. On examination, there is an unusual prominence of olecranon process.

A 43-year-old teacher presents for evaluation of diffuse joint pain. In addition to arthralgias, the patient
describes numbness and tingling in his fingers that is worse at night. His medical history is notable for
diabetes and hypertension. On examination his wrists, back, and knees are hyperextensible, and he has
marked pronation of the ankles. The patient’s wrists appear thickened without clear evidence of synovitis,
and Phalen’s maneuver is positive bilaterally. Radiographs of the hands are consistent with degenerative
osteoarthritis, and distal digital tufting is noted.

Which of the following is the most likely cause of this patient’s osteoarthritis?

49
Wilson’s disease
Ehlers-Danlos syndrome, type III
Ochronosis
Primary generalized osteoarthritis
Acromegaly

Explanation

The correct answer is choice E.

Acromegaly is a syndrome that results when the pituitary gland produces excess growth hormone (hGH)
after epiphyseal plate closure at puberty. A number of disorders may increase the pituitary's GH output,
although most commonly it involves a GH producing tumor called pituitary adenoma, derived from a
distinct type of cell (somatotrophs). Acromegaly most commonly affects adults in middle age, and can
result in severe disfigurement, serious complicating conditions, and premature death if unchecked.
Because of its insidious pathogenesis and slow progression, the disease is hard to diagnose in the early
stages and is frequently missed for many years, until changes in external features, especially of the face,
become noticeable.

In over 90 percent of acromegaly patients, the overproduction of growth hormones is caused by a benign
tumor of the pituitary gland, called an adenoma. These tumors produce excess growth hormones and, as
they expand, compress surrounding brain tissues, such as the optic nerves. This expansion causes the
headaches and visual disturbances that often accompany acromegaly. In addition, compression of the
surrounding normal pituitary tissue can alter production of other hormones, leading to changes in
menstruation and breast discharge in women and impotence in men because of reduced testosterone
production.

Patients afflicted with acromegaly have coarse facial features with swollen hands and feet. Larger rings,
gloves, shoes, and hats are needed. Overgrowth of the jawbone (mandible) can cause the jaw to protrude.
Cartilage in the voice box (larynx) may thicken, making the voice deep and husky. The ribs may thicken,
creating a barrel chest. Joint pain is common; after many years, crippling degenerative arthritis may occur
that is caused by thickening of bones and cartillage rather than the usual wear and tear of osteoarthritis.

Wilson’s disease (choice A) or hepatolenticular degeneration is an autosomal recessive genetic disorder in


which copper accumulates in tissues; this manifests as neurological or psychiatric symptoms and liver
disease. Although copper deposition in the joints can result in arthritic changes, the lack of hepatic or
neurological findings does not suggest Wilson’s disease is the source of this patient’s arthritis.

Ehlers-Danlos syndrome, type III (choice B) is a group of inherited connective tissue disorders, caused by
a defect in the synthesis of collagen (a protein in connective tissue). Physical findings will include
hypermobile joints and history of easy bruising and bleeding. Patient’s will develop osteomalacia and early
onset of osteoarthritis but

Ochronosis (choice C) is an autosomal recessive metabolic disorder that causes an excess of homogentisic
acid, resulting in adverse pigmentation, calcification, and inflammation of cartilaginous and related tissue
throughout the body. Pigmented cartilage may appear blue due to a light scattering phenomenon (Tyndall
effect), and to a lesser degree this may be true for skin with dermal deposition of this pigment.

The clinical features of this metabolic disorder are dark urine, pigmentation of the skin and arthritis.
Particularly helpful is the almost constant presence of a patch of pigmentation (gray to brown in color) in
the sclera, between the margin of the cornea and the outer or inner canthus. Early arthritis is the most
common presentation of ochronosis but patients will present at a very early age like most metabolic
disorders.

50
Primary generalized osteoarthritis (choice D) also known as degenerative arthritis or degenerative joint
disease, is a group of mechanical abnormalities involving degradation of joints, including articular cartilage
and subchondral bone. Symptoms may include joint pain, tenderness, stiffness, locking, and sometimes an
effusion. A variety of causes—hereditary, developmental, metabolic, and mechanical—may initiate
processes leading to loss of cartilage. When bone surfaces become less well protected by cartilage, bone
may be exposed and damaged. As a result of decreased movement secondary to pain, regional muscles
may atrophy, and ligaments may become more lax.

Osteoarthritis progresses in stages:

 joint space begins to narrow and osteophytes form


 joint space disappears as cartilage wears away and bone rubs on bone in the joint
 subchondral cysts appear
 bone tries to repair itself and there is bone remodeling

A 50 year old man has been imbibing martinis (shaken, not stirred) for several hours while at the
blackjack table. He wanders off and is found several minutes later lying down on the floor. Paramedics
arrive and discover a bruise on his posterior occiput; no other signs of trauma is found. He is lethargic
during his transport to the hospital but otherwise in stable condition; vital signs show: temperature 36.9
C., pulse 81/minute, respirations 20/minute, and blood pressure 115/80 mm Hg. On arrival, his blood
ethanol is 220 mg/dL. His mental status deteriorates and he becomes unresponsive. His right pupil is 8
mm and the left is 4 mm. A head CT scan reveals a collection of blood in the right subdural region.

Damage to which of the following intracranial vascular structures has most likely resulted in these
findings?

Middle meningeal artery


Cavernous sinus
Middle cerebral artery
Dural bridging vein
Great vein of Galen

Explanation

The correct answer is Choice D.

A subdural hematoma occurs when the small, bridging veins that run on the surface of the brain tear and cause
bleeding. The hematoma collects between the dura mater (which adheres to the skull) and the arachnoid mater
(which envelopes the brain). In essence, the hematoma is located between two membranes and is seen as a
crescent-shaped lesion on a non-contrast head CT—the blood is free to follow alongside the conclave curvature of
the brain’s surface. This compression can cause an increase in intracranial pressure which may produce papilodema.

Symptoms for subdural and epidural hematomas are similar, except that epidural signs occur rapidly (within hours)
compared to subdural (within days to weeks). Another difference is that a lucid period of consciousness occurs
frequently in epidural hematomas whereas a slow change in mental status happens in subdural hematomas. It is
difficult to obtain a proper neurologic assessment in patients that are intoxicated since the side effects (in this case
from alcohol) may mask a serious neurologic condition (in this case a subdural hematoma).

51
Alcohol level (mg/dl in a non-alcoholic patient):

 >100 Intoxicated
 >200 Lethargic
 >300 Comatose
 >500 Respiratory depression, death

Symptoms of subdural hemorrhage include:

 Confusion, disorientation
 Loss of consciousness
 Decreased memory
 Difficulty speaking or swallowing
 Difficulty walking
 Headache
 Seizures
 Weakness and/or numbness of arms, legs, face
 Irritability
 Vertigo
 Nausea or vomiting
 Personality changes
 Loss of appetite
 Blurred vision

Epidural hematomas develop rapidly due to a tear in the high pressure, middle meningeal artery (choice A) The
hematoma forms between the dura mater and the skull bone and is often the result of head injury. As the
hematoma expands, the dura is stripped from the inside of the skull, trapping the blood in the newly made space. It
is seen as a biconvex, or lentiform (lens-like), lesion on a non-contrast head CT—the hematoma is confined by the
tightly attached dura membrane and is unable to follow the brain’s curvature as a result.

52
The most common injury of the cavernous sinus (choice B) is thrombosis which usually occurs after an external
facial injury that lead to a clot in the facial vein. Common symptoms are proptosis, droopy eyelid, the inability to
move the eye in a particular direction or vision loss. This type of thrombosis is rarely fatal and can be treated with
antibiotics. The illness can become more serious, however, if it is left untreated and spreads to the dural veinous
sinuses.

Injury to the middle cerebral artery (choice C) would result in an intracerebral stroke and is caused by occlusion, not
trauma.

Symptoms include:

 Contralateral signs: hemiparesis, hemi-anesthesia, homonymous hemianopia


 Dominant hemisphere: dysphagia, aphasia
 Nondominant hemisphere: apraxia, sensory neglect

The most common injury to the great vein of Galen (choice E) is thrombosis and a form of stroke that presents with
consciousness problems, headaches, nausea, visual defects, fatigue, disturbance of eye movements and pupillary
reflexes. Risk factors include oral contraceptives, pregnancy and postpartum.

A 10 year-old girl is noted to have increasing facial distortion for the past 8 months from a lesion involving
her jaw. On physical examination she has a right mandibular mass. A biopsy is performed and on
microscopic examination reveals a monotonous pattern of small non-cleaved lymphocytes.

Infection with which of the following organisms is most likely to be associated with development of this
girl's mass lesion?

53
Adenovirus
Cytomegalovirus
Epstein-Barr virus
Hepatitis C virus
Herpes simplex virus
Human T-lymphotropic virus

Explanation

The correct answer is choice C.

The case presented in this question is one of Burkitt lymphoma, which is a subcategory of non-Hodgkin's
lymphoma (NHL). Not only does the histology of Burkitt lymphoma appear as a pattern of small non-
cleaved lymphocytes, but one of the most common locations where this condition occurs is in the jaw.

As of 2009, approximately 65,000 cases of NHL were diagnosed each year in the United States, and
approximately 7,500 in Canada, 80-90% of which are of B cell origin. This amounts to approximately 5%
of new cancers in men and 4% of new cancers in women. The incidence of NHL has been increasing in
recent decades. The 65,000 cases in the United States in 2009 represents an increase from approximately
56,000 cases diagnosed in 2001. From 1973 to 2001, the incidence increased 80%. NHL is one of a few
types of cancer that seems to result from an immunocompromised state. Thus, the sharp increase in
recent times may have to do with the increased use of immunosuppressive drugs as well as the emergence
of the HIV epidemic.

The World Health Organization (WHO) divides lymphocytic neoplasms into three broad categories:
Hodgkin's lymphoma, B-cell neoplasms, and T-cell and putative NK-cell neoplasms, the latter two
categories including both lymphomas and leukemias. B-cell neoplasms are further divided into precursor
and peripheral B-cell neoplasms. Diffuse large B-cell lymphoma (DLCL) is one of several types of the
peripheral category, but accounts for approximately 40% of NHLs in North America and Europe. Burkitt
lymphoma is a type of B-cell lymphoma.

Geography appears to play a role in the development of NHL and the type of NHL that dominates, with
Burkitt lymphoma being prominent in equatorial Africa (this is known as endemic Burkitt lymphoma), and
follicular lymphomas being more common in North America and Europe.

While incidence of Burkitt lymphoma is only 0.1 per 100,000 in the United States, in Africa the incidence is
57 to 76 times higher, at 5.7-7.6 per 100,000 and this may have to do with viral factors. In particular,
Burkitt lymphoma has been found to be associated with infection with Epstein-Barr virus (EBV, choice C).

Adenoviruses (choice A) affect the respiratory and gastrointestinal tracts along with the eyes and urinary
tract. They account for approximately 10 percent of acute respiratory infections in children and also cause
diarrhea, but there is no association with Burkitt lymphoma.

While cytomegalovirus (choice B) affects the oropharynx and accounts for many cases of infectious
mononucleosis (as EBV does), there is no known association between cytomegalovirus and Burkitt
lymphoma.

While hepatitis C virus (choice D) is prevalent in Africa and causes severe liver disease, there is no
association with Burkitt lymphoma

Herpes simplex virus (choice E) consists of two categories: HSV-1 and HSV-2. While these affect the
oropharynx along with the genitals, as in the case of the others, there is no association with Burkitt
lymphoma.

Although Burkitt lymphoma is highly aggressive, often it is responsive to chemotherapy. The regimen that

54
is used is rituximab together with EPOCH, which stands for etoposide, vincristine, doxorubicin,
cyclophosphamide, and prednisone, although regimens also include methotrexate, Ifosfamide, and
cytarabine.

The Human T-lymphotropic virus Type I (HTLV-1) (choice F) is a human RNA retrovirus that causes T-cell
leukemia and T-cell lymphoma in adults and may also be involved in certain demyelinating diseases,
including tropical spastic paraparesis.

A 63 year old man presents with a two month history of severe headaches, aggressive behavior and
weakness. Brain MRI reveals three dense, homogenously enhanced periventricular masses. Sterotactic
brain biopsy confirms a primary cerebral lymphoma.

All of the following statements regarding primary cerebral lymphoma are true except:

Macroscopically, it can be very difficult to differentiate it from malignant gliomas


Its clinical course can resemble that of glioblastoma multiforme
The EBV gene is found in immunocompromised patients only
Surgical resection is ineffective in almost all cases
Cranial irradiation and steroids produce partial response in the majority of patients

Explanation

The correct answer is Choice C.

Primary central nervous system lymphomas (PCNSL) are highly associated with Epstein-Barr virus (EBV)
infection and is found in > 90% among immunodeficient patients. Rare cases among immunocompetent
patients have shown an association with EBV infection (choice C). In the immunocompetent population,
PCNSLs typically appear in patients between the ages of 50-60.

Despite some characteristic conventional MRI findings, it may be difficult or even impossible to distinguish
cerebral lymphomas from glioblastoma multiforme. It is also difficult from a histopathological perspective
to distinguish a lymphoma from a high-grade glioma (choice A). In general, gliomas are more infiltrative
and show more nuclear pleomorphism. The clinical setting as well as cytomorphological features can be
useful in differentiating gliomas from PCNSL. Any doubt can usually be resolved by immuno-histochemical
studies.

A primary CNS lymphoma usually presents with signs of mass effect: seizures, headache, cranial nerve
findings, altered mental status, diplopia, dysphagia, vertigo, monocular vision loss, progressive dementia
and stupor. Systemic symptoms may include fever, night sweats or weight loss. Gliomas have similar
symptoms as PCNSLs (choice B) with a median survival time of less than a year with treatment. The
prognosis for PCNSL is somewhat better with a median survival time of 3.5 years with treatment.

Surgical resection shows no additional benefit usually because of the depth of the tumor (choice D).
Treatment with irradiation and corticosteroids typically only produces a partial response (choice E) and
tumors recur in more than 90% of patients.

55
A social worker refers a 2-year-old child to the physician because of suspected child abuse. On physical
examination, the child has blue sclerae, diminished hearing in both ears, and misshapen teeth. There are
no external contusions or lacerations of the skin. Radiographs of the child's extremities show recent and
healing long bone fractures. Which of the following is the most likely diagnosis?

Extensive trauma ("shaken child syndrome")


Osteogenesis imperfecta
Achondroplasia
Rickets
Osteopetrosis

Explanation

The correct answer is choice B

Osteogenesis imperfect, is a genetic malformation in which the child is born with, or prone to multiple
fractures that diminishes as the child gets older (the thin bone type). There is also mesenchymal defect in
osteoid and collagen formation which results in poor ossification and fibrous tissue formation. This poor
fibrous tissue formation exposes the choroid coat and so on physical examination the sclera appears blue.

Radiograph will reveal thin osteoporotic diaphysis with grossly mishappened (bent) bones. In Extensive
trauma that may result from child abuse, x-ray may or may not show multiple fractures, where it is
present, bone diaphysis will be normal with no osteoporotic lesions. Most importantly, the child will
present with external contusions or skin lacerations. Our patient here has no such external signs on
physical examination.

Achondroplasia results from a chromosomal defect that presents mainly as a disorder of cartilages. The
patient is a dwarf with well developed large cranial vault. Dwarfism is also seen in some cases of
Osteogenesis imperfecta but the distinguishing feature is the large size of the head in Achondroplasia and
absence of fractures.

In Rickets, the problem is mainly the softening of bone due to failure of calcification of osteoid to osseous
bone. This failure results from deficient calcium absorption following 1, 25-dihydroxycholecalciferol
deficiency. There are no fractures as described in our patient but presents instead with knocked knees
(Genu valgum) or bow legs (Genu Varus) as weight bearing increase in the child between the 6 months
and 2 years.

Osteopetrosis (Albers-Schonberg Disease) affects the long bones and the calvarium with sclerotic bones
on radiograph. The bones are brittle and as in Osteogenesis imperfecta, are also prone to fractures. But
there sclerae do not expose the blue choroid coat beneath. This is a major distinguishing sign on physical
examination.

A 14-year-old male is brought to the emergency room by ambulance after he was knocked unconscious by
a rough tackle on the football field. He was initially alert and interactive but as he was brought off the
field and into the sidelines, he began to have seizures. He arrives in the emergency room awake and
responsive, and vital signs are within normal limits. You order a CT scan of the head which shows a large,
actively bleeding epidural hematoma with compression of the surrounding structures and midline shift. As
you are transporting the boy to the operating room, he becomes unarousable.

Which one of the following acute vital sign changes would you expect to see in this patient?

56
tachycardia
hypotension
bradycardia
hypothermia
hyperthermia

Explanation

The correct answer is choice C.

This boy has suffered a traumatic epidural hematoma that is large enough to affect surrounding structures
and cause midline shift of the brain. One would expect further expansion of the bleed given the results of
the CT scan, causing an elevation of the intracranial pressure (ICP). Increased ICP can cause the Cushing
triad: hypertension, bradycardia, and irregular respirations.

Increased ICP can be caused by many factors, including trauma, tumor, meningitis, hydrocephalus, and
edema. Cerebral perfusion pressure (CPP) decreases as the ICP increases and can be remembered by the
equation:

Cerebral perfusion pressure (CPP) = mean arterial pressure (MAP) - intracranial pressure (ICP)

Usually the CPP is maintained by the body automatically, but this may fail if the ICP is acutely elevated to a
high enough pressure. This causes the resultant vital sign changes seen in the Cushing triad.
Papilledema, bulging fontanel (if present), altered mental status, and coma may also result and can
indicate possible compression of the brainstem and herniation. Acute elevation of ICP is an emergency and
requires acute medical and/or surgical intervention.

Tachycardia (choice A) is not a component of the Cushing triad. Rather, it is bradycardia that you may see
in the above vignette.

Hypotension (choice B) is not part of the triad.

Temperature changes (choices D and E) are not classically seen with acute ICP elevation.

A full-term male infant is delivered by forceps following premature rupture of the membranes experienced
by the mother several hours earlier. On physical examination, you note asymmetric gluteal folds and a
positive Ortoloni sign.

These findings are indicative of which of the following conditions?

Köhler disease
developmental dysplasia of the hip
Legg-Calvé-Perthes disease
slipped femoral epiphysis
Osgood-Schlatter disease

57
Explanation

The correct answer is choice B.

Known as "congenital dislocation of hip" as far back as the time of Hippocrates, developmental dysplasia of
the hip (DDH, choice D) involves dysplasia and dislocation, and can be elucidated during physical
examination of an infant by way of the Barlow and Ortoloni maneuvers during the first few months of life.
The Barlow maneuver is carried out by flexing the thigh and adducting the hip while stabilizing the pelvis
and applying posterior pressure. The Ortoloni maneuver (Ortoloni test) is performed, also by flexing the
thigh, but now abducting the hip with gentle traction. This will relocate the hip, if it has been dislocated
recently, and resistance to abduction constitutes a positive result. The originator of the latter test, Ortoloni,
was an early 20th century Italian pediatrician who evaluated, diagnose, and treated many children with
DDH. Additionally, the presence of asymmetric gluteal folds is suggestive of the condition.

Köhler disease (choice A) involves avascular necrosis in the navicular bone in the foot. Presenting most
commonly in children between the ages of 4-8, Legg-Calvé-Perthes disease (choice C) is a degenerative
condition of the hip joint, resulting from idiopathic avascular necrosis of the the epiphysis of the head of
the femur. It is a fairly rare disease, with approximately 5.5 cases occurring per 100,000 children,
annually. Typically, patients have a history of pain in association with weight-bearing activity, including
walking. Clinically, the most significant finding is decreased range of motion of the affected joint. Often,
pain is referred to the knee, while muscles of the thigh and leg may atrophy, exacerbating the effects on
movement. Additionally, osteoarthritis may develop, as a result of the rubbing between the degenerating
femoral head and the acetabulum.

Slipped capital femoral epiphysis (SCFE, choice D) is one of several conditions that can affect the hip joint
during childhood, and as with all hip condition, often pain is referred to the knee. While SCFE is fairly rare,
incidence is much higher in males compared to females and is influenced by ethnicity, with African-
Americans having 3.94 times the incidence of Caucasian children. The condition also depends on age,
presenting most frequently during adolescence. Obesity is an important risk factor, and in the case of
children younger than 10, endocrine disorders, particularly hypothyroidism, are associated as well.
Additionally, unexplained geographical and climate associations have been noted, with presentations
occurring more often during summer at latitudes higher than 40 degrees and during winter at lower
latitudes, in North America.

Osgood-Schlatter disease (choice E) is a condition of pain and inflammation of the tibial tubercle
apophysis, with knee pain localized over the tubercle. It is most common in adolescent athletes, especially
those involved in jumping sports.

A 22-year-old tennis player slips and falls awkwardly on her left wrist during a match. Plain radiographs
taken in the emergency department on the day of the injury show a nondisplaced fracture in the middle
third of the scaphoid bone.

All of the following statements regarding management methods for successful union after a nondisplaced
scaphoid fracture are correct except?

Closed reduction and percutaneous fixation offers a relatively short healing time of 8 to 12
weeks
Middle third scaphoid fractures have better prognosis for healing than proximal pole
fractures because of the vascular anatomy
Partial removal of the scaphoid is only used as a last resort in select conditions because of
resultant mechanical imbalances
Radiographs are the primary resource for assessing union within the first 12 weeks
The average postsurgical union rate after closed reduction for nondisplaced scaphoid
fractures is approximately 90%

58
Explanation

The correct answer is choice D

A scaphoid fracture is the most common type of wrist fracture. The scaphoid resides in the region of the
snuffbox and should be palpated after a wrist injury. Scaphoid fractures usually cause pain at the base of
the thumb accompanied by swelling in the same area.

The use of MRI, if available, is preferred over CT and can give one an immediate diagnosis. Bone
scintigraphy is also an effective method for diagnosis fracture which do not appear on Xray.

Clinically patients present with snuff box tenderness. Scaphoid fractures are often diagnosed by X-rays.
However not all fractures are apparent initially. Therefore people with tenderness over the scaphoid are
often casted for 7–10 days at which point a second set of X-rays is taken. If there was a hairline fracture,
healing will now be apparent. Even then a fracture may not be apparent. A CT Scan can then be used to
observe the snuff box area more precisely. Fractures can take between 6 and 12 weeks of casting. The
scaphoid has its own blood flow unlike other carpal bones. It receives its blood supply primarily from lateral
and distal branches of the radial artery. Blood flows from the top/distal end of the bone in a retrograde
fashion down to the proximal pole; if this blood flow is disrupted by a fracture, the bone may not heal.
Surgery is necessary at this point to mechanically mend the bone together.

Avascular necrosis (AVN) is a common complication of a scaphoid fracture. Risk of AVN depends on the
location of the fracture.

Fractures in the proximal 1/3 have a high incidence of AVN

Waist fractures in the middle 1/3 is the most frequent fracture site and has moderate risk of
AVN. Fractures in the distal 1/3 are rarely complicated by AVN.

Closed reduction and percutaneous fixation offers a relatively short healing time of 8 to 12 weeks (choice
A) is a correct statement of schapoid fractures.

Middle third scaphoid fractures have better prognosis for healing than proximal pole fractures because of
the vascular anatomy (choice B) is a correct statement of schapoid fractures. The scaphoid has its own
blood flow unlike other carpal bones. It receives its blood supply primarily from lateral and distal branches
of the radial artery.

Partial removal of the scaphoid is only used as a last resort in select conditions because of resultant
mechanical imbalance (choice C) is a correct statement of schapoid fractures.

The average postsurgical union rate after closed reduction for nondisplaced scaphoid fractures is
approximately 90% (choice E) is a correct statement of schapoid fractures. Non union can also occur from
undiagnosed or undertreated scaphoid fractures.

A 16 year-old boy is brought to your clinic because he is suffering from attacks that his mother describes
as "seizures", since he suffered a febrile seizure at age two and she believes his current condition is
related to this. He reports experiencing feelings of impending doom, that his heart is racing, difficulties

59
breathing and a sense that he is watching himself from the outside. An electroencephalogram (EEG)
conducted shows sharp waves in the anterior temporal and sphenoidal regions.

Most likely, this boy is experiencing which of the following?:

generalized tonic/clonic seizures


generalized absense seizures
simple partial seizures
complex partial seizures
panic attacks

Explanation

The correct answer is choice C.

Tachycardia and dyspnea certainly are principal features of panic attacks (choice E), which often may be
confused with seizures limited to the temporal lobe. The sharp waves on the EEG in this case indicate that
the boy is indeed interictal, which is to say in a period between episodes of some type of seizure.

Generalized tonic/clonic seizures (choice A), also known as grand mal seizures, are the most frequent type
and also the most familiar to most physicians. These consist of two phases: a tonic phase and a clonic
phase. The tonic phase manifests as stiffening in the arms and legs (hypertonia) accompanied by
hypoventillation, which may stop breathing completely for a period. If this happens, it causes cyanosis
visible in the nailbeds, face, and lips. The clonic phase features jerking of arms, legs, and face, while
breathing resumes, and lasts for no more than a minute. During the seizure the patient may become
incontinent and may bite the tongue. Care provided should be supportive to prevent injuries and turning
seizing individuals on their sides to prevent choking. The common practice of sticking on object into the
patient's mouth as a bite block is not recommended. During a post-ictal period that can last from a few
minutes to hours, the person may be confused, fatigued, and suffer from headaches.

Generalized absence seizures (choice B), also known as petit mal seizures, are characterized by an
individual simply staring into space as if daydreaming, with no tonic or clonic movements, although rapid
blinking or limited jerking movements may be involved. Typically, this condition presents between the ages
of 3 and 12 years.

A partial seizure also is called a focal seizure, because it involves a specific region of one hemisphere. This
type of seizure is categorized as simple partial (choice C) or complex partial (choice D). In the former case,
consciousness is not lost, while consciousness is lost or at least impaired in the latter case. If a partial
seizure develops into a generalized seizure, it is known as a partial seizure secondarily generalized.

Partial seizures can include motor or sensory symptoms. The latter case may involve vertigo, paresthesias,
visual or auditory effects, and also higher cortical symptoms related to emotion, or the phenomenon of
déjà vu. Depersonalization, the feeling that one is observing himself from the outside is characteristic of a
temporal lobe effect.

A 13 month-old infant is brought to the hospital on account of reduced appetite. His records show that she
was in the 70th weight percentile at 3 months of age and the 75th percentile for weight at 6 months,
while his length was in the 60th percentile both visits. Now he is in the 20th percentile for weight and the
30th for height. He is formula-fed with no indications of any sensitivity to the formula.

While he achieved early milestones of early infancy such as holding her head up at 2 months of age and
rolling by 7 months, his parents are concerned that he is barely crawling while some of his peers are

60
beginning to walk. On physical examination, you find generalized hypotonia, while laboratory results
include an absence of hexosaminidase A activity in the child's serum.

Other likely findings on physical examination include:

microcephaly
unusually-long extremities
cherry red spots on funduscopy
musty odor of the child's sweat
reduced pigmentation of hair

Explanation

The correct answer is choice C.

Neuromotor deterioration with failure to grow and gain weight can result from many conditions. However,
the absence of hexosaminidase A activity in the child's serum, suggests that Tay-Sachs disease is the
reason in the case of this child. In this condition, the lack of activity of hexosaminidase A enzyme results in
the accumulation of GM2 ganglioside in lysosomes of neurons. This, in turn is caused by a mutation in the
gene HEXA. For the disease to manifest, one needs to have a mutation in both copies of HEXA, since one
functional gene is sufficient for metabolism of GM2; thus, Tay-Sachs is a recessive disease. While HEXA a
is located on chromosome 15, the correct band is 15q23-q24, located in the long arm of the chromosome.
As part of the workup, chromosomal analysis is likely to reveal HEXA mutations on both chromosomes of
pair 15.

The funduscopic finding of "cherry-red" spots on the macula of each retina typically is associated with GM2
gangliosidosis.

In its classic infantile form, Tay-Sachs disease is fatal, usually by ages 2-4, since no treatment is available
to prevent the accumulation of GM2 ganglioside. Generally, macrocephaly (enlarged head, defined as being
above 97th percentile on head circumference), not microcephaly (choice A) is noticeable by about 18
months of age, though it may occur earlier, as in the case of this child. In the course of deteriorating,
seizures, difficulty swallowing, spasticity, dementia, and blindness set in. Death usually results from
pneumonia.

Incidence of the Tay-Sachs gene is high among Ashkenazic Jews. In North America, it used to be that 99
percent Tay-Sachs diagnoses were in Jewish children, but the success of genetic screening programs has
reduced the incidence. At the same time, other ethnic groups carry HEXA gene mutations of different types
so that, internationally, 1 of every 280 people is a carrier for a HEXA mutation. Since the non-Jewish
mutations were recognized more recently than the one that affects Ashkenazic Jews, the proportion of Tay-
Sachs diagnoses in non-Jews has increased. In North America, non-Jewish populations with a high
incidence of Tay-Sachs include Pennsylvania Dutch (Amish), Louisiana Cajuns, and French Canadians.
Thus, in the eastern St. Lawrence River Valley of Quebec, incidence of Tay-Sachs disease is similar to that
of Jewish populations.

Unusually-long extremities (choice B) is found in association with Marfan syndrome and with Klinefelter
syndrome. Indeed, medical historians have speculated that figures such as Abraham Lincoln and the
Egyptian pharaoh Akhenaton may have had Marfan syndrome. People with Marfan syndrome may be
diagnosed from infancy into adulthood. They are at risk for structural cardiovascular conditions such as
mitral valve prolapse, aortic root dilatation, and aortic dissection. Klinefelter syndrome results from the
presence of one or more extra X chromosomes in a male.

Musty odor (choice D) and reduced pigmentation of hair (choice E) occur in phenylketonuria (PKU), which

61
results from a deficient ability to metabolize phenylalanine to tyrosine. Classically, this is due to deficiency
in the enzyme phenylalanine hydroxylase (PAH). Although the precise mechanism is not known, the
resulting buildup of phenylalanine in the brain leads to mental retardation. If detected early, however, the
condition can be managed by way of a diet extremely low in phenylalanine, with tyrosine added in its
place, and patients can live a fairly normal life.

While there is an adult form of Tay-Sachs in which people have lived as long as 80 years, this is a very
different condition, in which HEXA activity, while low, is present nevertheless. This results in the disease
being chronic with symptoms appearing only during adulthood. Additionally, there is a late
infantile/juvenile form which is subacute, with onset between ages 2-10 years.

An 11-year-old little league pitcher presents to the hospital with a chief complaint of pain and numbness
in the medial aspect of his elbow on his throwing arm. He also complains of a “locking sensation” in his
elbow when pitching. Physical examination reveals mild crepitus, a positive Tinel’s sign in the cubital
tunnel, and decreased sensation in the fourth and fifth digits. Radiographic evaluation reveals an
olecranon osteophyte formation and a large posteromedial loose body.

Which of the following statements regarding this patient’s condition is correct?

There is no role for surgical excision of the olecranon osteophyte or loose body.
The valgus extension overload (VEO) test is a helpful diagnostic aid.
If left untreated, this patient will develop wrist extensor weakness.
This patient’s elbow pain will be worst during the follow-through phase of a pitch.
Medial ligamentous laxity decreases olecranon impingement within the olecranon
fossa.

Explanation

The correct answer is choice B

Injury to the medial collateral ligament of the elbow (MCL) can be a career-threatening injury for an
athlete without appropriate diagnosis and treatment. It has been considered separately from other athletic
injuries due to the unique constellation of pathology that results from repetitive overhead throwing. The
ligament can become stretched, frayed, or torn through the repetitive stress of the throwing motion. The
risk of injury to the throwing athlete's elbow ligament is thought to be extremely high as the amount of
stress through this structure approaches its ultimate tensile strength during a hard throw.

The anterior bundle of the MCL is the primary restraint to valgus stress from 20° to 120° of flexion, and tensile stresses
approaching the point of failure are generated during the acceleration phase of high velocity throwing.

MCL injuries, ulnar neuritis, valgus extension overload with osteophyte formation and posteromedial impingement, flexor
pronator strain, medial epicondyle pathology, and osteochondritis dissecans (OCD) of the capitellum have all been
described as sequelae of the overhead throwing motion. In addition, loose body formation, bony spur formation, and
capsular contracture can all be present in conjunction with these problems or as isolated entities.

Evaluation of medial stability is the cornerstone of the assessment of the overhead athlete with valgus extension overload.
This can be assessed by techniques such as the “milking maneuver” and the moving valgus stress test. The moving valgus
stress test: Starting with the arm in full flexion, the examiner applies a constant valgus force to the elbow and then quickly
extends the elbow. The patient experiences reproduction of his painful symptoms with an apprehension-like response in an
arc as the elbow passes from 120° of flexion to 70° of extension. The milking maneuver is performed by having the patient
reach under his injured arm with the opposite hand and grab the thumb of the injured arm. Continued pulling will place a
valgus stress on the elbow under examination. Of note, the clinician should palpate the MCL in approximately 60° of

62
flexion, to move the flexor pronator mass anterior to the fibers of the anterior band.

There is no role for surgical excision of the olecranon osteophyte or loose body (choice A) is an incorrect statement for this
type of injury. Not all pathology in the thrower's elbow is amenable to surgical treatment but osteophyte or loose body can
be excised.

If left untreated, this patient will develop wrist extensor weakness (choice C) is incorrect for excessive overhead
movement. Wrist extensor weakness would suggest radial nerve damage.

The ulnar nerve is a critical source of pathology and of potential arthroscopic complications. A history of ulnar neuritis is
suggested by paresthesias or dysesthesias in the distribution of the ulnar nerve, often radiating from the medial elbow into
the ulnar two digits. Intrinsic muscle hand weakness or clumsiness with fine motor movements may also belie ulnar
neuropathy. Of the utmost importance in all patients undergoing elbow arthroscopy is to evaluate the stability of the ulnar
nerve within the cubital tunnel.

This patient’s elbow pain will be worst during the follow-through phase of a pitch (choice D) for this type of injury is
incorrect. Medial elbow pain is the most common symptom in athletes who throw. Pain may be especially prominent during
the acceleration phase of the overhead throw.

Medial ligamentous laxity decreases olecranon impingement within the olecranon fossa (choice E) is incorrect. Laxity in the
medial ligament increases olecranon impingement within the olecranon fossa.

A 60 year old man has tenderness in the region distally between the tendons of the extensor pollicis
longus and extensor pollicis brevis (anatomic snuff box) after falling on the palm of his right hand. A
radiograph is taken of the hand to assess for fracture.

A fracture of which of the following carpal bones is most likely in this patient?

hamate
lunate
scaphoid
trapezium
triquetrum

Click on image to Zoom

Explanation

The correct answer is choice C

A Scaphoid fracture is the most common type of wrist fracture. The scaphoid resides in the region of the
snuffbox and should be palpated after a wrist injury. Scaphoid fractures usually cause pain at the base of
the thumb accompanied by swelling in the same area. The use of MRI, if available, is preferred over CT
and can give one an immediate diagnosis. Bone scintigraphy is also an effective method for diagnosis

63
fracture which do not appear on Xray.

Clinically patients present with snuff box tenderness. Scaphoid fractures are often diagnosed by X-rays.
However not all fractures are apparent initially. Therefore people with tenderness over the scaphoid are
often casted for 7–10 days at which point a second set of X-rays is taken. If there was a hairline fracture,
healing will now be apparent. Even then a fracture may not be apparent. A CT Scan can then be used to
observe the snuff box area more precisely. Fractures can take between 6 and 12 weeks of casting. The
scaphoid has its own blood flow unlike other carpal bones. It receives its blood supply primarily from lateral
and distal branches of the radial artery. Blood flows from the top/distal end of the bone in a retrograde
fashion down to the proximal pole; if this blood flow is disrupted by a fracture, the bone may not heal.
Surgery is necessary at this point to mechanically mend the bone together.

Avascular necrosis (AVN) is a common complication of a scaphoid fracture. Risk of AVN depends on the
location of the fracture.

 Fractures in the proximal 1/3 have a high incidence of AVN


 Waist fractures in the middle 1/3 is the most frequent fracture site and has moderate risk of AVN.
 Fractures in the distal 1/3 are rarely complicated by AVN.

Non union can also occur from undiagnosed or undertreated scaphoid fractures.

hamate (choice A) is a bone in the human hand that may be readily distinguished by its wedge-shaped
form, and the hook-like process which projects from its volar surface. It is situated at the medial and lower
angle of the carpus, with its base downward, resting on the fourth and fifth metacarpal bones, and its apex
directed upward and lateralward.

This is the bone most commonly fractured when an amateur golfer hits the ground hard with their golf club
on the downswing. The fracture is usually a hairline fracture, commonly missed on normal X-Rays.
Symptoms are pain aggravated by gripping, tenderness over the hamate and symptoms of irritation of the
ulnar nerve. This is characterized by numbness and weakness of the pinky finger with partial involvement
of the ring finger as well, the "ulnar 1½ fingers."

lunate (choice B) is a carpal bone in the human hand that may be distinguished by its deep concavity and
crescentic outline. The lunate carpal bone is situated between the lateral Scaphoid bone and medial
Triquetral bone. The lunate carpal bone straddles distally the bordering ulna and radius bones and
proximally to distal Trapezium and Trapezoid bone.

The lunate is the most frequently dislocated carpal bone.

trapezium (choice D) is distinguished by a deep groove on its palmar surface. It is situated at the radial
side of the carpus, between the scaphoid and the first metacarpal bone. Fractures of the trapezium do not
manifest as anatomic snuff box tenderness.

triquetrum (choice E) is located in the wrist on the medial side of the proximal row of the carpus between
the lunate and pisiform bones. It is on the ulnar side of the hand, but does not articulate with the ulna. It
connects with the pisiform, hamate, and lunate bones. It is the 3rd most commonly fractured carpal bone
but fractures of the triquetrum does not manifest as anatomic snuff box tenderness.

64
A 20 year-old presents in your family sports medicine clinic complaining of a "problem knee" which he
aggravated while snowboarding two weeks ago. This has happened several times before, but normally the
knee feels better after a few days. This time, it has not improved and when he shows it to you, you note
swelling. You ask the patient to sit on a table with his legs dangling over the edge. You then bend the
patient's knee so that it's flexed at a 90-degree angle. You then grasp the patient's foot and turn it inward
and instruct the patient to extend the affected leg until he feels pain. The patient experiences some pain
when the leg reaches about 30 degrees of flexion. As you rotate the patient's foot back into its normal
position, this alleviates the pain in the patient's knee.

The next step in the management of this patient is:

plain X-ray films


MRI
arthrography
referral to an orthopedic surgeon
conservative measures (rest, ice, observation)

Explanation

The correct answer is choice A.

65
The history and presentation, along with the positive Wilson test, suggests a diagnosis of osteochondritis
dissecans (OCD), which is characterized by a focal area of injury to subchondral bone which leads to
necrosis. If the subchondral bone does not heal, sequelae include swelling, softening, partial detachment,
and finally complete separation of a bony fragment. Typically, it presents during the ages of 10-20, but it
occurs in other age groups as well, with males being affected with more than double the incidence as
females. Most cases (85 percent) involve the medial femoral condyle (MFC) of the knee, while the rest
involve the lateral femoral condyle, with 70 percent of MFC lesions occurring in the posterolaterally.

OCD is thought to develop in response to either trauma or ischemia and few symptoms and signs are
available to identify it. The Wilson test is performed by having the examiner flex the knee to 90 degrees
while rotating the tibia internally. If pain develops at 30 degrees of flexion and is relieved by external
rotation, this is considered a positive Wilson sign. Given this patient's condition, conservative measures
(choice E) are not adequate as the severity of the condition needs to be assessed. Thus, the next step is to
take plain X-ray film (choice A) which should reveal features such as a bony fragment in the medial
condyle of the femur. While arthrography (choice C) was used in the diagnosis of OCD in the past, the
advent of MRI (choice B) renders it largely obsolete, although in certain cases it is useful in conjunction
with MRI as an MR arthrogram.

Thus, the plain X-ray, which should be done in AP, lateral, and tunnel views, should be followed up with
MRI which can elucidate any osteonecrosis, which is staged according to appearance in the imaging.

Stage 1 means thickening of articular cartilage and low signal changes. Stage 1 means there is a beach of
the articular cartilage and low signal rim behind a fragment, suggesting fibrous attachment. Stage 3 means
that the articular cartilage is breached but with high signal changes behind the fragment as well as
underlying subchondral bone. Stage 4 means an osteochondral fragment is detached or very loose.

While stages 1 and 2 represent a knee that is stable, if the condition proves to be in stage 3 or higher, at
that point surgical intervention will be required, making referral to orthopedics appropriate (choice D).

A 64-year-old man comes to clinic complaining of lower back pain. His history reveals that he is a chronic
alcoholic, and he just returned from a 5-year sojourn in a sub-Saharan African country where he
occasionally consumed unpasteurized goat milk. He has not lost weight, and has not experienced any
change in his bowel habits.

Which of the following is the least indicated in making a diagnosis in this patient?

Bone marrow biopsy


Digital rectal examination
Chest x-ray
Thyroid function test
Mantoux test

Explanation

The correct answer is choice D

Bone marrow biopsy (Choice A) aids the diagnosis of multiple myeloma, a plasma cell malignancy
characterized by production of large amounts of abnormal proteins, lytic bone lesions and hypercalcemia.
Its peak age of onset is 60-70 years, and may present as back pain.

66
Digital Rectal Exam (Choice B) aids in the diagnosis of prostate cancer. It develops in men aged 50 and
above, and commonly metastasizes to the lumbar vertebrae where it may cause lower back pain and
paraparesis.

Chest x-ray (Choice C) aids in the diagnosis of lung cancer, a malignancy he is at increased risk for since
he is a chronic smoker. Lung cancer commonly metastasizes to the vertebral column, and the patient may
present with back pain.

Thyroid function test (Choice D) aids in the diagnosis of hyperthyroidism. In this condition, increased bone
resorption may lead to pathologic fractures and bone pain. However, in the absence of any thyroid
symptoms, it is least likely to be the cause. Furthermore, age and sex make this condition less likely in the
patient.

Mantoux test (Choice E) aids in the diagnosis of tuberculosis. Given the patient’s history of consumption of
unpasteurized goat milk in a tropical country, he is at increased risk for abdominal tuberculosis. The
microbe may spread to the spine, where it may affect the lower thoracic or upper lumbar vertebra and
cause back pain.

A 9-year-old male with sickle cell disease presents to your emergency room with a fever and knee pain.
His dad says that he fell on the knee about a week ago. Initially the pain was severe, and over the past
week, it has decreased but persists. It is now reddened, and the boy refuses to walk on the knee. His
vital signs are as follows: temperature, 39°C; heart rate 120 beats/min; and blood pressure 110/72
mmHg. His left leg is generally erythematous with the knee being the area of maximal erythema. It is
warm, shiny, and exquisitely tender, and the boy refuses to actively move the joint. On passive range of
motion exercises, he screams in pain and is only able to tolerate minimal movement of the knee. Plain
radiographs are negative for any fracture. You admit him for intravenous antibiotics and pain control and
order magnetic resonance imaging of the extremity for the following morning.

Which bacterial pathogen is most likely responsible for this boy's symptoms?

group A Streptococcus
Salmonella
Pseudomonas
Enterobacter
Haemophilus influenzae

Explanation

The correct answer is choice B.

This boy's symptoms are concerning for osteomyelitis. He sustained an initial trauma to the knee that
likely spread directly to the bone. The boy has fever, tachycardia, and an erythematous, tender,
edematous knee with minimal passive or active range of motion. An inability to move the affected joint
should immediately be concerning for an osteomyelitis or septic joint. Osteomyelitis can be secondary to
direct inoculation, as in this vignette, or hematogenously spread.

Diagnosis of osteomyelitis is based on clinical suspicion and is best confirmed with magnetic resonance
imaging (MRI). Plain radiographs of the affected extremity are usually negative in the acute phase, with
abnormalities only becoming evident weeks after the infection has spread. If the site of infection is known,
MRI can generally confirm the diagnosis. If osteomyelitis is suspected but no site can be identified, a bone
scan can aid in localizing the infection. A blood culture and orthopedic surgery consultation should be

67
obtained. Labs may also be helpful in diagnosis, with white blood cell count, C-reactive protein, and
erythrocyte sedimentation rate commonly being elevated.

Common pathogens causing osteomyelitis depend on the age group and cause of the infection. In all types
of osteomyelitis, Staphylococcus aureus is a major pathogen. Hematogenous spread is commonly caused
by Enterobacter, group A Streptococcus, and Haemophilus influenzae. Directly inoculated osteomyelitis is
commonly caused by Pseudomonas, with a special subset of patients with sickle cell disease having
Salmonella as the most common cause. Diabetics are commonly affected by anaerobic bacteria and
immunocompromised patients by fungal elements and Mycobacteria.

Treatment for osteomyelitis is intravenous antibiotics aimed at the most likely bacterial pathogens. Blood
cultures and surgical debridement may aid in treatment by identifying a specific bacterial cause.

Group A Streptococcus and Haemophilus influenzae (choices A and E) are most frequently found in children
with osteomyelitis secondary to hematogenous spread of the bacteria.

Pseudomonas (choice C) may be found in directly inoculated osteomyelitis but is not the most common
bacteria in sickle cell patients.

Enterobacter (choice D) is common in both hematogenous and directly inoculated osteomyelitis but is not
the most common bacteria affecting sickle cell patients.

A 65-year-old woman reports difficulty walking. She can walk for one block, but pain “in the back of her
legs” forces her to stop and sit, which relieves the pain. She can walk for a long time in the grocery store,
but the pain returns with carrying groceries into her house. On examination, she has limitation in range of
motion in the cervical spine, marked Hayberden’s nodes and Bouchard’s nodes, and knee crepitus. Her
neurologic examination is unremarkable. Her distal pulses are detected with some effort.

Which of the following is the most likely cause of her symptoms?

Spinal stenosis
Vascular claudication
Knee osteoarthritis
Trochanteric bursitis
Lumbar radiculopathy

Explanation

The correct answer is choice A

Spinal stenosis is a medical condition in which the spinal canal narrows and compresses the spinal cord
and nerves. This is usually due to the common occurrence of spinal degeneration that occurs with aging.
It can also sometimes be caused by spinal disc herniation, osteoporosis or a tumor. In the cervical (neck)
and lumbar (low back) region it can be a congenital condition to varying degrees. Spinal stenosis may
affect the cervical, thoracic or lumbar spine. In some cases, it may be present in all three places in the
same patient. Lumbar spinal stenosis results in low back pain as well as pain or abnormal sensations in
the legs, thighs, feet or buttocks, or loss of bladder and bowel control.

Symptoms can be similar to those found with claudication giving rise to the term pseudoclaudication for
when they present in association with spinal stenosis. Spinal or neurogenic claudication is not due to lack

68
of blood supply, but rather it is caused by nerve root compression or stenosis of the spinal canal, usually
from a degenerative spine, most often at the "L4-L5" or "L5-S1" level. This may result from many factors,
including bulging disc, herniated disc or fragments from previously herniated discs (post-operative), scar
tissue from previous surgeries, osteophytes (bone spurs that jut out from the edge of a vertebra into the
foramen, the opening through which the nerve root passes). In most cases neurogenic claudication is
bilateral, i.e. on both sides, but it can also be present unilaterally.

Vascular claudication (choice B) refers to impairment in walking, or a "painful, aching, cramping,


uncomfortable, or tired feeling in the legs that occurs during walking and is relieved by rest". The
perceived level of pain from claudication can be mild to extremely severe. Claudication is most common
in the calves but it can also affect the feet, thighs, hips, buttocks, or arms.

Spinal or neurogenic claudication may be differentiated from arterial claudication based on activity and
position. In neurogenic claudication, positional changes lead to increased stenosis (narrowing) of the
spinal canal and compression of nerve roots and resultant lower extremity symptoms. Standing and
extension of the spine narrows the spinal canal diameter. Sitting and flexion of the spine increases spinal
canal diameter. A person with neurogenic claudication will have worsening of leg cramping with standing
erect or standing and walking. Symptoms may be relieved by sitting down (flexing the spine) or even by
walking while leaning over (flexion of the spine) a shopping cart

Knee osteoarthritis (choice C) is a degradation of the knee joint, including articular cartilage and
subchondral bone. Symptoms may include joint pain, tenderness, stiffness, locking, and sometimes an
effusion. A variety of causes—hereditary, developmental, metabolic, and mechanical—may initiate
processes leading to loss of cartilage. When bone surfaces become less well protected by cartilage, bone
may be exposed and damaged. As a result of decreased movement secondary to pain, regional muscles
may atrophy, and ligaments may become more lax. Osteoarthritis would not explain her claudication
symptoms.

Trochanteric bursitis (choice D) is the inflammation of one or more bursae (small sacs) of synovial fluid
leading to hip pain. The bursae rest at the points where internal functionaries, such as muscles and
tendons, slide across bone. Healthy bursae create a smooth, almost frictionless functional gliding surface
making normal movement painless. When bursitis occurs, however, movement relying upon the inflamed
bursa becomes difficult and painful. Moreover, movement of tendons and muscles over the inflamed
bursa aggravates its inflammation, perpetuating the problem.

Lumbar radiculopathy (choice E) is not a specific condition, but rather a description of a problem in which
one or more nerves are affected and do not work properly (a neuropathy). The emphasis is on the nerve
root . This can result in pain (radicular pain), weakness, numbness, or difficulty controlling specific
muscles. In a radiculopathy, the problem is at or near the root of the nerve, along the spine. However,
the pain or other symptoms may manifest in an extremity through a process called referred pain. For
example, an impingement in the lower back or lumbar-sacral spine can be manifested with symptoms in
the foot.

A 16 year old female fell while roller blading on her out stretched right hand. She is taken to the ER and
an x-ray of the wrist is taken. Upon inspection, there is significant angulation and deformity of the right
arm.

The proper reduction technique for this wrist fracture includes which of the following?

69
slight extension, full pronation, and full ulnar
deviation
slight flexion, full supination, and full radial
deviation
slight extension, full supination, and full radial
deviation
slight flexion, full pronation, and full ulnar deviaton
slight flexion, full pronation, and no deviation

Click on image to Zoom

Explanation

The correct answer is choice D

Reduction is a medical procedure to restore a fracture or dislocation to the correct alignment. When a bone
fractures, the fragments lose their alignment in the form of displacement or angulation. For the fractured
bone to heal without any deformity the bony fragments must be re-aligned to their normal anatomical
position. Orthopedic surgeons attempt to recreate the normal anatomy of the fractured bone by reduction.

Reduction could be by Closed or Open methods.

 Open reduction refers to the method wherein the fracture fragments are exposed surgically by
dissecting the tissues.
 Closed reduction refers to manipulation of the bone fragments without surgical exposure of the
fragments.

Once the fragments are reduced, the reduction is maintained by application of casts, traction or held by
implants which may in turn be external or internal.

A Colles' fracture, also Colles fracture, is a distal fracture of the radius in the forearm with dorsal
(posterior) displacement of the wrist. The fracture is sometimes referred to as a "dinner (or silver) fork
deformity" due to the shape of the resultant forearm. For a more detailed discussion see distal radius
fracture.

The term Colles fracture is classically used to describe a fracture at the distal end of the radius, at its
cortico-cancellous junction. However, now the term tends to be used loosely to describe any fracture of the
distal radius, with or without involvement of the ulna, that has dorsal displacement of the fracture
fragments.

The fracture is most commonly caused by people falling forward onto a hard surface and breaking their fall
with extended outstretched hands - falling with wrists flexed would lead to a Smith's fracture. It can also
be caused by overuse. It usually occurs about an inch or two proximal to the radio-carpal joint with
posterior and lateral displacement of the distal fragment resulting in the characteristic dinner fork like
deformity. Colles' fracture is a common fracture in people with osteoporosis, second only to vertebral
fractures.

Management depends on the severity of the fracture. An undisplaced fracture may be treated with a cast
alone.The cast is applied with the distal fragment in palmar flexion and ulnar deviation. A fracture with
mild angulation and displacement may require closed reduction. Significant angulation and deformity may
require an open reduction and internal fixation or external fixation. The volar forearm splint is best for

70
temporary immobilization of forearm, wrist and hand fractures, including Colles' fracture There are several
established instability criteria: dorsal tilt >20° communited fracture abruption of the ulnar styloid process
intraarticular displacement >1mm loss of radial height >2mm. A higher amount of instability criteria
increases the likelihood of operative treatmant.

slight extension, full pronation, and full ulnar deviation (choice A) would not restore the fractured bones to
original alignment. The distal fragment needs to be in palmar flexion when the cast is applied.

slight flexion, full supination, and full radial deviation (choice B) would not restore the fractured bones to
original alignment. Radial deviation would aggravate the fracture.

slight extension, full supination, and full radial deviation (choice C) would not restore the fractured bones
to original. Radial deviation would aggravate the fracture and the distal fragment needs to be in palmar
flexion

slight flexion, full pronation, and no deviation (choice E) would not restore the fractured bones to original
alignment.

A 65-year-old rheumatoid patient with a long history of intermittent neck pain is brought to the primary
care office because her family notes that she has become progressively less functional and is now
wheelchair-bound. Physical examination reveals globally decreased strength in her lower extremities,
hyperreflexia in both upper and lower extremities with an upgoing Babinski reflex, and a positive
Hoffman’s sign. Hip range of motion is painful.

What is the next step in this patient’s management?

Cervical spine radiographs


High-dose prednisone taper
Hip and pelvis radiographs
Lumbar spine radiographs
Physical therapy for strengthening and aquatherapy

Explanation

The correct answer is choice B

Spinal cord compression develops when the spinal cord is compressed by bone fragments from a vertebral
fracture, a tumor, abscess, ruptured intervertebral disc or other lesion. It is regarded as a medical
emergency independent of its cause, and requires swift diagnosis and treatment to prevent long-term
disability due to irreversible spinal cord injury.

Symptoms suggestive of cord compression are back pain, a dermatome of increased sensation, paralysis of
limbs below the level of compression, decreased sensation below the level of compression, urinary and
fecal incontinence and/or urinary retention. Lhermitte's sign (intermittent shooting electrical sensation) and
hyperreflexia may be present.

Diagnosis is by X-rays but preferably magnetic resonance imaging (MRI) of the whole spine. The most
common causes of cord compression are tumors, but abscesses and granulomas (e.g. in tuberculosis) are
equally capable of producing the syndrome. Tumors that commonly cause cord compression are lung

71
cancer (non-small cell type), breast cancer, prostate cancer, renal cell carcinoma, thyroid cancer,
lymphoma and multiple myeloma.

Dexamethasone (a potent glucocorticoid) in doses of 40-60 mg/day may reduce edema around the lesion
and protect the cord from injury. It may be given orally or intravenously for this indication.

Surgery is indicated in localised compression as long as there is some hope of regaining function. It is also
occasionally indicated in patients with little hope of regaining function but with uncontrolled pain.
Postoperative radiation is delivered within 2-3 weeks of surgical decompression. Emergency radiation
therapy is the mainstay of treatment for malignant spinal cord compression. It is very effective as pain
control and local disease control. Some tumors are highly sensitive to chemotherapy (e.g. lymphomas,
small-cell lung cancer) and may be treated with chemotherapy alone.

Once complete paralysis has been present for more than about 24 hours before treatment, the chances of
useful recovery are greatly diminished, although slow recovery, sometimes months after radiotherapy, is
well recognised. The median survival of patients with metastatic spinal cord compression is about 12
weeks, reflecting the generally advanced nature of the underlying malignant disease.

Cervical spine radiographs (choice A) should be ordered to assess the level of compression but not before
emergent steroids or external radiation is delivered to a compressed cord. An MRI of the spinal cord would
be the best possible study to order in a spinal cord compression.

Hip and pelvis radiographs (choice C) would not be appropriate. The patient has upper motor neuron signs
and upper extremity weakness suggesting that the lesion is at the level of the cerivical spine.

Lumbar spine radiographs (choice D) would not be appropriate. The patient has upper motor neuron signs
and upper extremity weakness suggesting that the lesion is at the level of the cerivical spine.

Physical therapy for strengthening and aquatherapy (choice E) is an inappropriate treatment modality for
spinal cord compression.

A 32-year-old man comes to you, because of a fever. He is pale and has several ecchymoses and
petechiae on his back and leg and several small axillary and inguinal nodes. The WBC is 114,000 /uL
(normal 4,100-10,900), Hgb 12.0 g/dL (normal 13.2-16.2), Hct 34.9 percent (normal 40-52). As you
expected, he is thrombycotopenic, his platelet count being 105,000/uL (normal 140,000-450,000), and he
must be worked up for leukemia.

Which of the following is true, regarding laboratory differentiation between different leukemia types?

the Philadelphia chromosome can be used to differenciate myelogenous leukemias from


lymphoid leukemias
translocation t(8;21) is common in chronic myelogenous leukemia
Auer bodies are a frequent feature of acute myeloid leukemia
Periodic acid-Schiff (PAS) is negative in M3 leukemia
myeloperoxidase (MPO) helps to distinguish between acute and chronic lymphoid
leukemias

Explanation

72
The correct answer is choice C.

A fever, together with anemia and an extremely high white blood cell count (WBC) occurs in various types
of leukemia. Anemia is a common effect, as is thrombocytopenia, because the proliferation of leukemic
cells in the bone marrow interferes with the production of other blood cells. While the platelet count is
within the normal range, the episodes of epistaxis suggest that it may have been dropping to lower
numbers, or that the current count is low for this patient.

Chronic leukemias are defined as proliferations of fairly mature white blood cells. Generally, these are
post-mitotic. In contrast, acute leukemias are characterized by a proliferation of immature white blood
cells. Auer bodies (or Auer rods) are javelin-shaped marks often visible on some of leukemic cells of acute
myelogenous leukemia (AML). Made of fused lysosomes that contain crystalline inclusions, they also are
rich peroxidase and other lysosomal enzymes and thus stain with the bluish dye, azure.

The Philadelphia chromosome (choice A) is a t(9;22) translocation that is present in leukemic cells in
approximately 95 percent of chronic myelogenous leukemia (CML). However, it also has been found to
occur in 25-30 percent of acute lymphoblastic leukemia in adults and sometimes in acute myelogenous
leukemia (AML) as well.

The t(8;21) translocation (choice B), is found in approximately 5-12 percent of patients with AML. Periodic
acid-Schiff (PAS) is sensitive to glycogen in cells and tends to be weakly positive in M3 leukemia (choice
D), which refers to acute promyleocytic leukemia.

Myeloperoxidase (MPO, choice E) is an enzyme present in the lysosomes of neutrophils. Stains that detect
it are useful in the diagnosis of myeloid leukemias, but they will not stain lymphoblasts. Although
immature myeloid blasts can be negative to MPO, lymphoblasts cannot be positive. Thus, a positive MPO
test is particularly useful would rule out acute lymphoblastic leukemia (ALL) when it already is known that
a patient has acute leukemia.

The most common laboratory findings in nutritional rickets are decreases in serum calcium, serum
phosphorus, calcidiol, calcitriol, and urinary calcium. Conversely, parathyroid hormone, alkaline
phosphatase, and urinary phosphorus levels are elevated.

Which one of the following laboratory test results would indicate nutritional rickets in a child?

Decrease alkaline phosphatase


Decreased serum calcium
Decreased urinary phosphorous
Decrease PTH
all of the above

Explanation

The correct answer is choice B

In nutritional rickets, the problem is mainly deficiency of 1, 25-dihydroxy vitamin D which results in
impaired intestinal absorption of calcium or an outright deficiency of calcium. Poor intestinal calcium
absorption logically leads to decreased serum calcium levels.

73
Low levels of serum calcium have a negative feedback effect on parathyroid hormone secretion in the
parathyroid glands (increased PTH secretion). Calcium sensing receptors located in the parathyroid glands
are activated when serum calcium level is low causing PTH secretion to increase and help normalize
calcium levels in the following ways;

Bone: PTH leads to bone resorption through osteoclasts and osteoblasts resulting in increase in the level of
alkaline phosphatase.

Intestine: PTH influences the intestinal absorption of calcium by mediating renal production of 1, 25-
dihydroxy vitamin D which needed for calcium absorption.

Kidney: PTH further enhances urinary excretion of phosphorus so concentration of phosphorus in the urine
is increased in an attempt to increase Calcium-Phosphorus ratio.

A 43 year-old woman, gravida 2, para 1, presents at your clinic with vague headaches and difficulties with
her vision. Additionally, she reports that her last menstrual period was more than 7 weeks ago. A
pregnancy test is negative, but you find bitemporal hemianopia on her visual field test which is confirmed
after more thorough testing by an ophthalmologist. The patient then is sent for an MRI which reveals a 12
mm pituitary tumour.

Most likely, this tumour is of which of the following types:

somatotroph
lactotroph
gonadotroph
corticotroph
thyrotroph

Explanation

The correct answer is choice B.

Derived from adenohypophyseal cells, tumours of the pituitary gland account for 15 percent of primary
intracranial neoplasms. With an incidence of 1 out of 10,000 annually, adenomas account for more than 90
percent of pituitary tumours. Pituitary adenomas are categorized as microadenomas if they are less than
10 mm in size, or macroadenomas if they are 10 mm or bigger, as this one is. It is estimated that as much
as 10 percent of the general population has asymptomatic pituitary tumours, which of course tend to be
microadenomas.

In this case, the bitemporal hemianopia (hemianopsia) indicates that the tumour is compressing the optic
chiasm, and this was corroborated by the MRI. There are five categories of pituitary adenoma.
Somatotrophs (choice A) secrete growth hormone, and thus can cause acromegaly. Gonadotrophs (choice
C), which secrete gonadotrophins, even though many of these used to be categorized as "non-functional".
Corticotrophs (choice D) secrete ACTH, and thus cause Cushing syndrome. Thyrotrophs (choice E) secrete
thyroid-stimulating hormone (TSH) and thus lead to symptoms of hyperthyroidism.

A lactotroph is a tumour that secretes prolactin and thus also are called prolactinomas. These are the most
common, accounting for 60 percent of pituitary adenomas. Because of the prolactin secretions,
amenorrhoea is a common symptom in women.

74
A healthy 14 year old boy complains of anteriomedial right knee pain and limping for 6 weeks. The most
significant findings on examination consists of spontaneous external rotation of the right lower extremity
when the hip is flexed and wasting of the muscles of the right thigh. Examination of the right knee is
normal.

Which of the following is the most likely diagnosis?

Legg Calve Perthes disease


slipped capital femoral epiphysis
stress fracture of the distal femur
septic arthritis of the hip
ostomyelitis of the distal femur

Explanation

The correct answer is choice B

This patient has a chronic slipped capital femoral epiphysis (SCFE) of his right hip. A SCFE occurs when the
epiphysis of the femoral head displaces from its normal position. The most common presentation of a
chronic SCFE is an insidious onset of knee pain. The patient will be able to bear weight, but will have an
antalgic gait. It is essential that every knee pain be evaluated with a hip examination.

Choice A: Legg Calve Perthes Disease is a condition in which avascular necrosis develops in the hip.
Presentation can be similar with referred knee pain; however, the age group for initial presentation is
usually between 4 and 8 years old.

Choice C: Stress fracture of the distal femur will present with pain in the distal thigh or knee, however, hip
rotation will not be affected.

Choice D: Septic arthritis of the hip presents more acutely with sudden pain, inability to bear weight, and
fevers.

Choice E: Osteomyelitis of the distal femur will present with pain in the thigh or knee, however, hip
rotation will not be affected.

A 44 year old woman presents at your clinic on account of neck pain for the last five weeks. It is a
shooting pain with a 10/10 severity score, and for the last four days she has suffered from weakness of
the left arm as well. Her history is unremarkable for any medical conditions and her only surgery was an
elective caesarian section following an uneventful pregnancy at age 38. Physical Examination reveals
weakness on left shoulder abduction with a 4/5 score and left elbow flexion, also with a 4/5 score. With
the exception of the triceps, all reflexes in the left arm are absent, but there are no sensory findings, and
the rest of the examination is unremarkable.

Most likely, you will diagnose this patient with which of the following conditions?

75
frozen shoulder
radial nerve palsy
CVA
brachial neuritis
cervical myelopathy

Explanation

The correct answer is choice D.

The weakness of shoulder abduction is associated with spinal nerve C5, while the affect on elbow flexion
means C6. The lack of reflexes in the biceps reflex points to C5 and C6, in the brachioradialis and supinator
C6, while the presence of the triceps reflex means that C7 is unharmed. These findings, together with the
acute onset and extreme pain, are highly suggestive of brachial neuritis.

A frozen shoulder (choice A) manifests with stiffness and painful restriction of all movements of the
shoulder join. This is the result of adhesive capsulitis, and thus can be treated with intra-articular injection
of steroids, along with physical therapy. Palsy of the radial nerve (choice B) presents with wrist drop, due
to weakness of the wrist extensors, supinator, extensor carpi radialis, and finger extensors. Known
classically as "Saturday night palsy" resulting from falling asleep with one's arm hanging over the back of
chair), the condition results from compression of the radial nerve against the middle third of the humerus.
If the nerve is compressed in the axilla, however, the triceps reflex will be lost as well and the triceps will
weaken. A cerebrovascular accident accident (CVA, choice C), can be ruled out, since it would manifest
with upper motor neuron signs.

Cervical myelopathy (choice E) would be a possibility, if the lesion were located at C5-C6, but in this case
you would expect upper motor neuron signs below the level of the lesion. Due to compression on the nerve
root and on fibers continuing downward, a spinal cord lesion can manifest with lower motor neuron signs at
the level of the lesion, and upper motor neuron signs below the level of the lesion.

A 45 year-old woman noticed tinnitus in her left ear which progressed over 5 weeks to hearing loss in that
ear. On physical examination she is found to have a marked decrease in hearing on the left, with Rinne
test indicating air conduction better than bone conduction. The other cranial nerves I - VII and IX - XII are
intact. A brain MRI scan revealed a solitary, fairly discrete, 3 cm mass located in the region of the left
cerebellopontine angle.

Which of the following statements is most appropriate to tell the patient regarding these findings?

Due to the aggressive nature, survival is not expected for more than a year
Remissions and exacerbations are likely to occur in coming years
Other family members should undergo MR imaging of the brain
The lesion can be resected with a good prognosis
A test for HIV-1 is likely to be positive

Explanation

The correct answer is choice D.

76
The presentation and imaging suggests a diagnosis of acoustic neuroma, a benign tumour that develops
from Schwann cells of the cranial nerve VIII in the cerebellopontine angle. Although it also is known as
vestibular schwannoma, acoustic neuroma can involve either the cochlear or vestibular branch of the
nerve. Thus, it may present with symptoms related either to sensorineural hearing loss, or balance, or
both. In this case the hearing loss, tinnitus (ringing in the ear), and the Rinne test result indicating air
conduction better than bone conduction suggest that the acoustic nerve is the affected site. A Romberg
test, which involves balance and coordination with eyes closed, is part of a standard neurological
examination and should be performed on a patient such as this. If she is Romberg positive, then the
patient can be tested with electronystagmography (ENG) to explore any vestibular deficits.

Acoustic neuroma is a fairly common condition. With the increased use of MRI and high resolution CT
scanning in recent years, many small, asymptomatic acoustic neuromas has been elucidated as incidental
findings. In such cases, only regular monitoring may be required. In the case of this patient who has
symptoms, surgical removal or destruction of the tumour with radiation are the best treatment options.
The prognosis is very good, it is a slow-growing tumour, and thus choices A and B can be eliminated.

Acoustic neuroma has no associated with HIV and since occurrence of most acoustic neuromas is sporatic,
MRI screening of family members (choice C) has not been recommended as of 2010. Some acoustic
neuromas, however, develop in association with neurofibromatosis, which is categorizes as type 1 or type
2. Both types are associated with acoustic neuroma, but in type 2, often they are bilateral and in such
cases the inheritance appears to be autosomal dominant.

A 45 year old lady presents after a 6 week history of a painful neck. The pain is described as shooting in
nature and 10/10 on a severity score. Now she describes a 3 day history of right arm weakness.
Examination reveals weakness on right shoulder abduction (4/5) and right elbow flexion (4/5). Reflexes in
her right arm are all absent apart from triceps. There are no sensory findings. The remainder of the
examination is normal.

What is the most likely diagnosis?

Stroke
Cervical myelopathy
Brachial neuritis
Radial nerve palsy
Frozen shoulder

Explanation

The correct answer is choice C

Parsonage-Turner syndrome, also known as acute brachial neuropathy and acute brachial neuritis is a rare
set of symptoms resulting from inflammation of the lower motor neurons of the brachial plexus and/or
individual nerves or nerve branches. The brachial plexus is an arrangement of nerve fibers, running from
the spine, formed by the ventral rami of the lower four cervical and first thoracic nerve roots (C5-T1). It
proceeds through the neck, the axilla (armpit region), and into the arm. The brachial plexus is responsible
for cutaneous and muscular innervation of the entire upper limb, with two exceptions: the trapezius muscle
innervated by the spinal accessory nerve (CN XI) and an area of skin near the axilla innervated by the
intercostobrachial nerve. Lesions can lead to severe functional impairment.

In the idiopathic version, the pathophysiology is unknown, but the condition is generally thought to be an
immune system – mediated inflammatory reaction against nerve fibers of the brachial plexus. Axonopathy

77
with subsequent Wallerian degeneration appears to predominate.

Brachial neuritis is characterized by the acute onset of excruciating unilateral shoulder pain, followed by
flaccid paralysis of shoulder and parascapular muscles several days later. Affected muscles become weak
and atrophied, and in advanced cases, paralyzed. Despite its wasting and at times long-lasting effects,
most cases resolve themselves and recovery is occasionally complete. Nerve grafting or tendon transfers
may be considered for the few patients who do not achieve good recovery by 2 years. Surgery usually is
aimed at improving shoulder abduction.

Lacunar Stroke (choice A) that are purely motor are caused by infarcts at the posterior limb of the internal
capsule. It is marked by hemiparesis or hemiplegia that typically affects the face, arm, or leg of one side.
Dysarthria, dysphagia, and transient sensory symptoms may also be present. A lacunar stroke would not
explain brachial plexus deficits.

Cervical myelopathy (choice B) is caused usually by compression on the spinal cord in the neck.
Compression may be from bony overgrowth in the spine. Symptoms will depend how far down the
compression is with weakness and loss of sensation present from somewhere in the arms, all the way
down the chest and back to the feet. More specifically at the level of the lesion the muscles supplied by
that nerve root will display lower motor neurone signs (wasting, fasciluation, loss of tone and
decreased/absent reflexes) and all levels below will have upper motor neurone signs (spasticity, brisk
reflexes, clonus). Bladder and bowel function may be affected e.g. incontinence.

Radial nerve palsy (choice D), also known as Saturday night palsy, is a condition where a person can not
extend their wrist and it hangs flaccidly. Wrist extension is achieved by muscles in the forearm contracting,
pulling on tendons that attach distal to (beyond) the wrist. If the tendons, the muscles, or the nerves
supplying these muscles, are not working as they should be, wrist drop may occur.

The radial nerve is the terminal branch of the posterior cord of the brachial plexus. A stab wound may
damage the posterior cord and result in neurological deficits including an inability to abduct the shoulder
beyond 15 degrees, an inability to extend the forearm, reduced ability to supinate the hand, reduced ability
to abduct the thumb and sensory loss to the posterior surface of the arm and hand. The radial nerve can
be also be damaged if the humerus (the bone of the arm) is broken because it runs through the radial
groove on the lateral border of this bone.
Wrist drop is also associated with lead poisoning because of the effect of lead on the radial nerve.

Frozen shoulder (choice E) or adhesive capsuilitis is a disorder in which the shoulder capsule, the
connective tissue surrounding the glenohumeral joint of the shoulder, becomes inflamed and stiff, greatly
restricting motion and causing chronic pain. This condition, for which an exact cause is unknown, can last
from five months to three years or more and is thought in some cases to be caused by injury or trauma to
the area. It is believed that it may have an autoimmune component, with the body attacking healthy tissue
in the capsule. There is also a lack of fluid in the joint, further restricting movement. Frozen shoulder
would not explain this patient’s brachial nerve plexus deficits.

A 22 year old football player is brought to the emergency department 1 hour after he sustained a left leg
injury during a tackle. Physical examination shows mild tenderness and anterior instability of the tibia with
the knee in 90 degrees of flexion (positive drawer sign). Active range of motion of the left knee is limited
by pain.

Which of the following best explains these findings?

78
hemarthrosis
patellar fracture
tear of the anterior cruciate ligament
tear of the medial ligament
tear of the medial meniscus

Explanation

The correct answer is choice C

The anterior cruciate ligament (ACL) is a cruciate ligament which is one of the four major ligaments of the
human knee. The ACL originates from deep within the notch of the distal femur. Its proximal fibers fan out
along the medial wall of the lateral femoral condyle. There are two bundles of the ACL—the anteromedial
and the posterolateral, named according to where the bundles insert into the tibial plateau. The ACL
attaches in front of the intercondyloid eminence of the tibia, being blended with the anterior horn of the
lateral meniscus. These attachments allow it to resist anterior translation of the tibia, in relation to the
femur.

Anterior cruciate ligament injury is the most common knee ligament injury, especially in athletes.

Lateral rotational movements in sports like these are what cause the ACL to strain or tear. Strains can
sometimes be fixed through physical therapy and muscle strengthening, though tears almost always
require surgery. The most common method for repairing ACL injuries is arthroscopic surgery. Doctors will
either use a patient's own tendons, such as part of their hamstring, or ligaments from cadavers to
construct a new ACL. Other common injuries accompanying ACL tears are meniscus, MCL, and knee
cartilage tears.

The drawer test is a test used by providers to detect rupture of the cruciate ligaments in the knee. The
patient should be supine with the hips flexed to 45 degrees, the knees flexed to 90 degrees and the feet
flat on table. The examiner sits on the patient's feet and grasps the patient's tibia and pulls it forward
(anterior drawer test) or backward (posterior drawer test). If the tibia pulls forward or backward more than
normal, the test is considered positive. Excessive displacement of the tibia anteriorly indicates that the ACL
is likely torn, whereas excessive posterior displacement of the tibia indicates that the PCL is likely torn. The
Lachman test is a variation on this test in which the knee is in thirty degrees flexion.

hemarthrosis (choice A) is a bleeding into joint spaces. It usually follows injury but occurs mainly in
patients with a predisposition to hemorrhage such as those being treated with warfarin (or other
anticoagulants) and patients with hemophilia. In hemophilia it may occur spontaneously, and recurrent
hemarthroses are a major cause of disability in that patient group due to hemophilic arthropathy, requiring
synovectomy, joint replacement. Up to a quarter of all severe ligament or capsular knee injuries leading to
a haemarthrosis are associated with cartilage damage that can lead to progressive degenerative arthritis.

patellar fracture (choice B) is a fracture of the kneecap. It is usually the result of a hard blow to the front
of the knee. Treatment options for patella fracture include nonsurgical and surgical options, depending on
the type of fracture. An inverse fracture of the patella takes from 4-6 weeks to heal.

tear of the medial ligament (choice D) would not manifest as anterior instability of the tibia. The medial
collateral ligament (MCL) of the knee is one of the four major ligaments of the knee. A MCL injury can be
very painful and is caused by a valgus stress to a slightly bent knee, often when landing, bending or on
high impact. Depending on the grade of the injury, the lowest grade (grade 1) can take between 2 and 10
weeks for the injury to fully heal. Recovery times for grades 2 and 3 are difficult to predict because of the
amount of damage done can take weeks to several months. It is difficult to apply pressure on the injured
leg for at least a few days.

tear of the medial meniscus (choice E) would not manifest as anterior instability of the tibia. The medial
meniscus separates the tibia and femur to decrease the contact area between the bones, and serves as a
shock absorber reducing the peak contact force experienced. It also reduces friction between the two

79
bones to allow smooth movement in the knee and distribute load during movement.

Acute injury to the medial meniscus fairly often accompanies an injury to the ACL (anterior cruciate
ligament) or MCL (medial collateral ligament). A person occasionally injures the medial meniscus without
harming the ligaments. Healing of the medial meniscus is generally slow. Damage to the outer 1/3 of the
meniscus will often fully heal, but the inner 2/3 of the medial meniscus has a limited blood supply and thus
limited healing ability. Large tears to the meniscus may require surgical repair or removal. If the meniscus
has to be removed (menisectomy) because of injury (either because it cannot heal or because the damage
is too severe), the patient has an increased risk of developing osteoarthritis in the knee later in life. More
chronic injury occurs with osteoarthritis, made worse by obesity and high-impact activity. The medial
meniscus and the medial compartment are more commonly affected than the lateral compartment.

A 24 year old football player severely fractures his ankle after being tackled in a game. Upon inspection,
you see an open wound with bone fragments. He is in serve pain and the decision is made to take him to
the operating room for an open reduction internal fixation (ORIF) treatment.

Indications for ORIF treatment of an ankle fracture include all of the following except?

a fracture-dislocation
undisplaced fracture with grade II ATFL tear
trimalleolar fracture
unstable talar tilt
unable to maintain a closed reduction

Explanation

The correct answer is choice D

Internal fixation is an operation in orthopedics that involves the surgical implementation of implants for the
purpose of repairing a bone. An internal fixator may be made of stainless steel or titanium. Types of
internal fixators include bone screws and metal plates, pins, rods, Kirschner wires and intramedullary
devices such as the Kuntscher nail and interlocking nail.

Reduction is a medical procedure to restore a fracture or dislocation to the correct alignment. When a bone
fractures, the fragments lose their alignment in the form of displacement or angulation. For the fractured
bone to heal without any deformity the bony fragments must be re-aligned to their normal anatomical
position. Orthopedic surgeons attempt to recreate the normal anatomy of the fractured bone by reduction.

Reduction could be by Closed or Open methods.

 Open reduction refers to the method wherein the fracture fragments are exposed surgically by
dissecting the tissues.
 Closed reduction refers to manipulation of the bone fragments without surgical exposure of the
fragments.

Once the fragments are reduced, the reduction is maintained by application of casts, traction or held by
implants which may in turn be external or internal.

Open Reduction Internal Fixation (ORIF) is a medical procedure. Open reduction refers to open surgery to
set bones, as is necessary for some fractures. Internal fixation refers to fixation of screws and/or plates to

80
enable or facilitate healing. Rigid fixation prevents micromotion across lines of fracture to enable healing
and prevent infection, which happens when implants such as plates are used. Open Reduction Internal
Fixation techniques are often used in cases involving serious fractures such as comminuted or displaced
fractures. Risks and complications can include bacterial colonization of the bone, infection, stiffness and
loss of range of motion, non-union, malunion, damage to the muscles, nerve damage and palsy, arthritis,
tendonitis, chronic pain associated with plates, screws, and pins, compartment syndrome, deformity,
audible popping and snapping, and possible future surgeries to remove the hardware.

Unstable talar tilt occurs in inversion injuries to the ankle resulting in ankle sprain. Injury to the anterior
talo-fibular ligament is the most common and 97% of all ankle ligament ruptures occur on the antero-
lateral side. Injury to the calcaneo-fibular ligament occurs in more severe injuries and disruption of the
posterior talo-fibular ligament is rare. ORIF is not indicated for talar tilt intasbility.

a fracture-dislocation (choice A) would require open reduction internal fixation. Joint dislocation occurs
when bones in a joint become displaced or misaligned. It is often caused by a sudden impact to the joint.
The ligaments always become damaged as a result of a dislocation. A subluxation is a partial dislocation. It
is important the joint is reduced as soon as possible, as in the state of dislocation, the blood supply to the
joint (or distal anatomy) may be compromised. This is especially true in the case of a dislocated ankle, due
to the anatomy of the blood supply to the foot.

undisplaced fracture with grade II ATFL tear (choice B) would require open reduction internal fixation. The
anterior talofibular ligament (ATFL) passes from the anterior margin of the fibular malleolus, forward and
medially, to the talus bone, in front of its lateral articular facet.

It is the most commonly sprained ligament, as part of the lateral ligament of the ankle.

trimalleolar fracture (choice C) would require open reduction internal fixation. A trimalleolar fracture is a
fracture of the ankle that involves the lateral malleolus, medial malleolus and the distal posterior aspect of
the tibia, the posterior malleolus. The three aforementioned parts of bone articulate with the talus bone of
the foot. Surgical repair is required, and because there is no lateral restraint of the foot, the ankle cannot
bear any weight while the bone knits. This typically takes six weeks in an otherwise healthy person.

inability to maintain a closed reduction (choice E) would require open reduction internal fixation. Closed
treatment is frequently unsuccessful in maintaining a good position in adults, because there is frequently
comminution of the fracture. Re-displacement and deformity can reoccur with an unacceptable ultimate
result.

A 59-year-old man presents at your clinic on account of shooting pains, ataxia in the lower extremities,
and bladder dysfunction. Physical examination is remarkable for small pupils that fail to constrict when
exposed to light, but do constrict when the patient focuses on an object placed close to his face.
Additionally, your neurological examination reveals deficiencies in sensation of vibration, discriminative
touch, and proprioception. You order VDRL testing which returns with positive results.

Mostly likely, spinal cord imaging will reveal atrophy of which of the following structures?

lateral column
dorsal column
ventral column
dorsal horn
ventral horn

81
Explanation

The correct answer is choice B.

VDRL stands for Venereal Disease Research Laboratory and it is the standard assay used to test for
syphilis infection. Transmitted sexually (though it also can be transmitted via infected blood) and caused
by the spirochete Treponema pallidum, syphilis develops in four stages: primary, secondary, latent, and
tertiary, and diagnoses can be difficult, since the disease can mimic many other conditions. The primary
syphilic lesion is the chancre. Secondary syphilis can include many conditions, notably neurosyphilis,
while the late stage (tertiary), which is very rare manifests with cardiovascular sequelae. Although the
incidence of syphilis infection dropped significantly in the United States during mid 20th century, the
incidence rose again near the end of the same century and by 2007 the incidence was 3.7 cases per
100,000 population.

Once called "Prostitute's Pupils", pupils that fail to constrict when exposed to light, but do constrict with
accommodation are known as Argyll Robertson (AR) pupils and are a sign of neurosyphilis. Another
feature of this phase of syphilis is degeneration of the dorsal (posterior) column of the spinal cord. Known
as tabes dorsalis, this condition appears generally between 10 to 25 years following the primary phase of
the infection, if it has not been treated and manifests with sensory defects. The dorsal columns of the
spinal cord, which carries this sensory information, such as that of vibration, touch, and proprioception,
atrophies, which is thought to result from damage to cells of the dorsal root ganglia.

The lateral column (choice A) includes the spinocerebellar and spinothalamic tracts which ascend, and the
lateral corticospinal and rubrospinal tracts which descend. The ventral column (choice C) includes the
anterior corticospinal and tectospinal tracts which descend, and also part of the spinothalamic tract,
which ascends, while the ventral horn (choice E) carries motor (descending) neurons. If any of these
choices were correct, there would be notable motor deficiencies. Although the dorsal horn (choice D)
carries many types of sensory information, including touch, vibration, and proprioception, it is the dorsal
column that is affected in neurosyphilis.

Following an episode of severe head trauma incurred in a motor vehicle accident, a 16 year old female is
noted to have decerebrate posturing. Funduscopic examination reveals marked bilateral papilledema. A CT
scan of the head reveals marked diffuse cerebral edema with effacement of lateral ventricles.

The edema is most likely to be severest in which of the following parts of the brain?

White matter
Grey matter
Dura
Ependyma
Meninges

Explanation

The correct answer is choice A.

The greatest amount of salt and water increase with cerebral edema occurs within white matter. Edema as
seen in response to trauma results from a breakdown of tight endothelial junctions which make up the
blood-brain barrier. This allows normally excluded intravascular proteins and fluid to penetrate into
cerebral parenchymal extracellular space. Once plasma constituents cross the blood-brain barrier, the
edema spreads; this may be quite fast and widespread.

Cerebral edema is most marked in white matter, not grey matter (choice B). Enlargement of neuronal cell

82
bodies is most marked with storage diseases.

Neither the tough connective tissue of the dura (choice C) nor the ependymal lining (choice D) do not swell
significantly even when faced with increases in sodium and water content.

The thin layer of meninges (choice E) accounts for little brain swelling.

A 39 year-old Caucasian woman presents at your clinic complaining of tingling and numbness in her legs
and a recent episode in which she lost her balance. Her family history is remarkable for a sister who was
diagnosed recently with multiple sclerosis (MS), she is concerned that her symptoms are an indication that
she is developing the disease as well. During your neurological exam, you find no Lhermitte sign or
anything else relevant to MS. Nevertheless, you refer her to a neurologist who ends sending her for an
MRI of the brain and spinal cord. When the MRI result is ready, you are relieved to learn that no changes
due to MS were revealed. However, there is an incidental finding of an 9 mm pituitary mass.

The most likely diagnosis is which of the following?:

pituitary microadenoma
pituitary macroadenoma
craniopharyngioma
pituitary carcinoma
primitive neuroectodermal tumor

Explanation

The correct answer is choice A.

Derived from adenohypophyseal cells, tumours of the pituitary gland account for 15 percent of primary
intracranial neoplasms. With an incidence of 1 out of 10,000 annually, adenomas account for more than 90
percent of pituitary tumours. Pituitary adenomas are categorized as microadenomas if they are less than
10 mm in size, or macroadenomas (choice B) if they are 10 mm or bigger. Due to improving availability
and increasing resolution of brain imaging modalities such as CT and MRI, incidental findings of small
pituitary tumours are on the rise, and it is estimated that as much as 10 percent of the general population
has such asymptomatic tumors. For this reason, the term "incidentaloma" has been coined.

Very often microadenomas are asymptomatic and studies have revealed that most are stable and do not
grow. Thus, for those microadenomas greater than 4 mm, MRI followup is recommended annually for the
first 2 years. Then, if the tumour proves stable, the intervals between MRIs may be increased.

Craniopharyngioma (choice D) is the most common pituitary tumour in the pediatric population. Thus, it is
far less likely in this case than pituitary adenoma. Pituitary carcinoma (choice D) is unlikely, since
fortunately, such a tumour is extremely rare, accounting for less than 0.5 percent of symptomatic
tumours, and this one is not even symptomatic.

Primitive neuroectodermal tumors (choice E) represent a class of malignant embryonal neoplasms that
occur mostly in children and some in young adults, with no connection to the pituitary gland.

A 71-year-old man presents to his primary care provider complaining of nonradiating lower back pain over
the past several months. On review of symptoms, he mentions feeling shorter than previously. Physical
exam reveals a slight abnormal curve of the spine. Radiographic studies show anterior vertebral wedging
and vertebral end-plate irregularity. His personal medical history is unremarkable, but his mother died at

83
age 75 years due to complications of a hip fracture. His T score is –2.

Which of the following choices is the correct interpretation of this patient’s T score and treatment?

Normal BMD; calcium supplementation


Normal BMD; alendronate for prevention
Osteopenia; zinc supplementation for prevention
Osteopenia; alendronate for treatment
Osteoporosis; residronate for treatment

Explanation

The correct answer is choice D

Osteoporosis is a disease of bones that leads to an increased risk of fracture. Osteoporosis literally means
'porous bones'. In osteoporosis the bone mineral density (BMD) is reduced, bone microarchitecture is
disrupted, and the amount and variety of proteins in bone is altered. Osteoporosis is defined by the World
Health Organization (WHO) in women as a bone mineral density 2.5 standard deviations below peak bone
mass (20-year-old healthy female average) as measured by DXA. Osteoporosis can be prevented with
lifestyle changes and sometimes medication; in people with osteoporosis, treatment may involve both.
Lifestyle change includes exercise and preventing falls as well as reducing protein intake. Medication
includes calcium, vitamin D, and bisphosphonates.

Dual energy X-ray absorptiometry (DXA, previously DEXA) is a means of measuring bone mineral density
(BMD). Two X-ray beams with differing energy levels are aimed at the patient's bones. When soft tissue
absorption is subtracted out, the BMD can be determined from the absorption of each beam by bone. Bone
density (or BMD) is used in clinical medicine as an indirect indicator of osteoporosis and fracture risk.

Results are generally scored by two measures, the T-score and the Z-score. Scores indicate the amount
one's bone mineral density varies from the mean. Negative scores indicate lower bone density, and
positive scores indicate higher.

The T-score is a comparison of a patient's BMD to that of a healthy thirty-year-old of the same sex and
ethnicity. This value is used in post-menopausal women and men over aged 50 because it better predicts
risk of future fracture.

 Normal is a T-score of -1.0 or higher


 Osteopenia is defined as less than -1.0 and greater than -2.5
 Osteoporosis is defined as -2.5 or lower, meaning a bone density that is two and a half standard
deviations below the mean of a thirty year old woman.

The Z-score is the number of standard deviations a patient's BMD differs from the average BMD of their
age, sex, and ethnicity. This value is used in premenopausal women, men under the age of 50, and in
children.

Osteopenia is considered by many doctors to be a precursor to osteoporosis. When medical therapy is


pursued, treatment includes medications with a range of actions. Commonly used drugs are
bisphosphonates including alendronate, risedronate, and ibandronate; selective estrogen receptor
modulators (SERMs) such as raloxifene; estrogen; calcitonin; and teriparatide

Normal BMD; calcium supplementation (chioce A) is characterized by a T-score of -1.0 or higher.

Normal BMD; alendronate for prevention(choice B) is indicated for people with a T-score of -1.0 or higher

84
and to reduce the risk of osteoporotic fracture. Bisphosphonates like alendronate prevent the loss of bone
mass. Bone has constant turnover, and is kept in balance by osteoblasts creating bone and osteoclasts
digesting bone. Bisphosphonates inhibit the digestion of bone by osteoclasts.

Osteopenia; zinc supplementation for prevention (choice C) is characterized by a T-score of less than -1.0
and greater than -2.5. Preventative measures like Zinc supplementation are ineffective in halting the
progression of osteopenia into osteoporosis.

Osteoporosis; residronate for treatment (choice E) is defined as -2.5 or lower, meaning a bone density that
is two and a half standard deviations below the mean of a thirty year old woman. Bisphosphonates like
residronate prevent the loss of bone mass.

A six year old child presents with a four week history of early morning headache, vomitting and restlessness. The child
has been unable to work steadily in the last week. A CT scan reveals the presence of a mass in the posterior crainial
fossa.

What is the most likely diagnosis?

Craniopharygioma
Ependymoma
Medulloblastoma
Oligodendroglioma
Glioblastoma Multiforme

Explanation

The correct answer is Choice C

Medulloblastoma is the most common malignant intracranial tumour of childhood arising from embryonic
tissue in the posterior part of the cerebellar vermis. It constitutes about 30% of all pediatric intra- cranial
tumours. It is also one of the few intracranial tumours to metastasize, usually to the bone. It is more
common in males with a male to female ratio of 2:1. Clinically, the child presents with a history of raised
intra-cranial pressure (ICP), which manifest as early morning headaches, vomiting, restlessness and
reduced social interaction. Raised ICP is associated with hydrocephalus which results from occlusion of the
fourth ventricle. Late signs include double vision and frequent falls. On examination, a paralytic
strabismus, nystagmus, papilloedema and ataxia are frequently observed. There may also be neck stiffness
from tonsillar herniation.The investigation of choice is a CT scan which will demonstrate an isodense
midline lesion in the cerebellum compressing the fourth ventricle. MRI may also be useful. Definitive
treatment is maximum surgical excision with external beam irradiation. Pre-operative ventriculo-peritoneal
shunt may be useful in reducing hydrocephalus, but increases the risk of tumour seeding. Adjuvant
chemotherapy has been shown to improve child survival at 5 years.

Crainiopharyngioma (Choice A) is a benign cystic tumour which arises from embryonic remnants of the
fourth pharyngeal pouch. The tumour is located just above the sella turcica where it compresses the optic
chiasma. The condition is most common in children aged between the ages of 5 – 10 years and accounts
for 6 -10 % of intracranial tumours in children. Clinical presentation depends on the size. Most times the
features are insidious and may include; headaches, visual disturbances, papilloedema and hydrocephalus
from third ventricle outflow obstruction. Options for treatment include surgical excision and radiotherapy.
There is a high rate of recurrence.

Ependymoma (Choice B) are the third most common brain tumour in children. It constitutes about 10% of

85
all intra-cranial tumours occurring mostly within the first two decades of life with the mean age of
diagnosis being 5 -6 years. The tumour arises from ependymal cells lining the ventricular canals. About
70% of these tumours are found in the fourth ventricle. Clinical features include raised ICP, hydrocephalus,
ataxia, vertigo and vomiting. CT scan shows an isodense mass within the fourth ventricle with or without
calcification. Treatment is by surgical excision and postoperative radiotherapy.

Oligodendroma (choice D) and Glioblastoma Multiforme (Choice E) are malignant forms of brain gliomas
which are not common in children.

A 38 year-old man presents at your clinic with fatigue, fever, and episodes of epistaxis for the past four
months. On physical examination his temperature is 38.0 °C (normal 36.5–37.5). His occupational history
is remarkable for exposure to organic solvents. Laboratory studies show his Hgb is 9.2 g/dL (normal 13.2-
16.2), Hct 27.9 percent (normal 40-52), platelet count 95,000/microliter (normal 140,000-450,000), and
WBC count 20,000/microliter (normal 4,100 -10,900). A peripheral blood smear shows large cells with
some signs of maturity, some cells with rod-shaped inclusions and kidney-shaped nuclei.

The most likely diagnosis in this patient is:

L1 leukemia
L3 leukemia
M1 leukemia
M2 leukemia
M6 leukemia

Explanation

The correct answer is choice D.

A fever, together with anemia and an extremely high white blood cell count (WBC) occurs in various types
of leukemia. Anemia is a common effect, as is thrombocytopenia (the cause of the patient's epistaxis),
because the proliferation of leukemic cells in the bone marrow interferes with the production of other blood
cells. Exposure to benzene and other organic solvents is known to cause thrombocytopenia, either isolated
or as a component of acute myelogenous leukemia (AML).

Chronic leukemias are defined as proliferations of fairly mature white blood cells. Generally, these are
post-mitotic. In contrast, acute leukemias are characterized by a proliferation of immature white blood
cells, but the precise level of maturity varies among acute leukemia subtypes.

L1 (choice A) is the type of acute lymphoblastic leukemia (ALL) that most often develops in children. Cells
tend to be small, with round, homogeneous nuclei. L3 (choice B) is Burkitt-type ALL, a very rare form
whose cells are large, with nuclei that are round to oval.

While M1 (choice C) is a type of acute myeloblastic leukemia, its cells demonstrate no signs of maturity,
and while the cells are big, their nuclei are round. In contrast type M2, though a class of acute
myelogenous leukemia (AML) features very large cells with some signs of maturity and kidney-shaped
nuclei. Often they contain Auer bodies (or Auer rods), javelin-shaped marks made of fused lysosomes that
stain with bluish dye, azure.

M6 (choice E) is a acute erythroblastic leukemia. Cells are big and round to oval, with large, lobed nuclei.

86
A 30-year-old woman presents with acute onset of low back pain that radiates into the left leg. On physical examination,
she has sensory deficits of the posterior lower leg with weakness of plantar flexion and eversion of the foot. She also has
diminished ankle reflexes.

Which of the following is the most likely diagnosis?

L3 radiculopathy
L4 radiculopathy
L5 radiculopathy
S1 radiculopathy
S2 radiculopathy

Explanation

The correct answer is choice D

The patient presents with low back pain, secondary to an irritation or impingement of the S1 root. Causes
include: herniated disc, tumors. S1 radiculopathy manifests with weakness of foot eversion (peroneus
brevis and longus) and weakness of plantar flexors (gastrocnemius and soleus muscles). Because of this,
she would have difficulty walking on her toes. Radiation of pain is on the posterior thigh and leg.
Numbness is located at the posterior lower leg. With radiculopathy at this level, the ankle reflex is
diminished.

L3 radiculopathy manifest with pain in the anterolateral aspect of the thigh and weak hip adductors and
flexors. Reflexes are not affected.

L4 radiculopathy presents with pain in the lower back, hip, posterolateral thigh, and anterior leg. There
isnumbness on the antero-medial thigh and knee. There is weakness of the quadriceps muscle (knee
extension). The knee reflex is diminished.

L5 radiculopathy presents with pain on the hip, lateral thigh and leg, with numbness of lateral leg and first
3 toes. There is weakness of the dorsiflexors, with the patient unable to walk on the heels. Ankle reflex is
usual.

S2 Radiculopathy patients present with sphincter disturbances (urinary and bowel incontinence, urinary
frequency), anaesthesia on the posterior thigh (“saddle area”) and perianal area. Reflexes to be tested
include: anal wink and bulbocavernous reflex.

87
88

Das könnte Ihnen auch gefallen